You are on page 1of 197

CONTENTS I

SI. No.
Page No.
1.. Syllabus (i)
2 Unit - 1 1
Differential Calculus
3. Unit 2 31
Partial Differential Equations
4. Unit - 3 58
Matrices and Determinant
5. Unit - 4 91
Integeral Calculus
6. Unit 5 121
Probability and Statistic
7. Unit - 6 149
Numerical Methods
..........-.:
SYLLABUS
ADVANCE ENGINEERING MATHEMATICS- (AC/AA-l.l)
UNIT-I:
Differential Calculus: Introduction, functions of several variables, Partial
Differentiation, Homogeneous function, Euler's Theorem, Total derivatives,
Taylor's theorem for function of two variables, Maxima and Minima of function of
the variable, Lagrange's method of undetermined multipliers.
UNIT-II:
Partial Differential Equations: Introduction, formation of partial differential
equations, method of separation of variables differential equation
Of the first order and first degree. Application of differential equations of first and
higher degree.
UNIT-III:
Matrices: Definition, Properties of matrix, addition, subtraction and
multiplication. Inverse of a matrix, elementary transformation and theorem.
Rank of a matrix, Caley Hamilton Theorem.
UNIT-IV:
Integral Calculus: Integration of substitution , by parts and by partial fractions.
Integration of trigonometric and irrational functions. Reduction
L __
..:;,
Formulae for indefInite integrals involving powers of circular functions of x
and products of sin x and Cos x. Elementary ideas of definite integrals and their
calculations. Simpson rule .for approximate integration. Lengths of simple curves.
Volumes and surfaces of solids of revolution. Mean value and
root- means-square value. Double and triple integrals and their simple applications .
UNITV:
Probability and Statistic: Concept of probability t laws of probability, Binomial
poisson, Normal Distribution. The t-distribution t working rule.
Nature and purpose of Mathematical statistics, Tabular and Graphical
Representation sample mean and variance.
UNITVI:
NUmerical Methods: Introduction, Numerical analysis including solution of
equations graphically, iterational Newton Raphson's or successive substitution
methods- Rule of false position (Regula Fabi). Numerical integration and
differentiation.
ii
(
Unit - 1 : Differential Calculus
Multiple Choice Type Questions:
Q.l. If f = 1 ,then fxx + fyy + fzz is
~ x 2 + l + Z 2
(a) f (b) (c) (d) o
2 4
Q.Z.
If u - f [ ~ J. then :if. +Y.f, is
(a) 0 (b) 1 (c) -1 (d) None of the above
x+ Y au au
Q.3.
If u = ..Jx .JY ,then x-+ Y is
x+ Y ax ay
u u
(a) (b) (c) 0 (d) u
2 2
Q.4. The function f (x, y) = x
2
+ l +6x +14 attains the maximum value at the point
(a) (-3,0) (b) (3,0) (c) (0,-3) (d) (0,3)
Q.S. Let a function f (x, y) be a continuous and possesses first and second order partial
derivatives at a point ( XI, Yl) .If P(XI, Yl) is a critical point and
r=fxx(XI'YI),s=fxy(x"Yl),t fyy(XI'YI),then the point P is a point of relative
maxima if
(a) rt-s
2
>O,r >0
(b) rt-s
2
>O,r<O
(cl rt - S2 < 0, r >
(d) rt-s
2
<O,r<O
2
+v U =at
2
Q.6. If z =u
2
, , v == 2at ,then dz is
dt
(a) 2at
2
+4at (b) at
2
+2at
(c) 4a
2
t(t
2
+2) (d) ~ 2at+ 2a
1
Q.7.
If t(x,y,z)=O then(:],(:1(:) is
(al 1 (bl -1 y
(c) 0 (d) None of the above
Q.8. Which of the following is not a homogeneous function
2
+ l/2
(al
x
3
+4xy2 +3x
2
y (bl
X
3
/
x
3
+l
(cl
x+y J
(dl None of the above
log
(
..Jx +JY
I I log x -log Y .
Q.9. The function f (x, y) = 2."" + - + 2 ' IS homogeneous with degree
x xy x +y
(a) 0 (b) -2 (c) 2 (d) None of the above
Q.l0. If f (x, y) =x
4
l sin-1 ( ~ ) then the value of xfx +yfy is
(al 6z (b) 4z (c) 0 (d) 2z
Q.ll. If u = ~ 2 l 2 log (1.) and v =cos-1 ( 2 xy 2 Jand if z =u+v then x az + y az is
x +y x x - y ax ay
(a) 4v (b)4u (cl 2u (dl 4u +v
au au .
Q.12. If u=atanB,v=bcotB then -.- IS
aa ab
(al tan B (bl cot B (cl 1 (d) ab
Q.13. If f(x,y)=x
4
+3x
3
y+4xy3+/,thenthevalueof fxy atthepoint (1,0) is
(a) 9 (bl 12 (cl 1 (d) 0
(
2 2) az az.
Q.14. If z = y f x - Y Y- +x- IS equal to I
ax ay
xz (cl xy
(al xz
(b) (d) z
y z
Q.1S. If z is a homogeneous function of x,y of degree nand Z =f (u) then x au +y au is
ax ay
f(U) f'(u)
(a)
n f'(u) (b) nz (c) nf(u) (dl n f(u)
2
.J
Q.16. A function f (x, y) is homogeneous if and only if
(a)
f (x, y) =xnf ( ~ )
(b)
f(x,y}: n(;J
(c) f (AX, Ay)=Anf (x, y) (d) All of the above
r 0 r 0 ao .
Q.17. If X =e cos , y = e sm , then - IS
ax
y y
(a) (b)
x
2
+i x
2
+i
x x
(c) (d)
x
2
+i x
2
+i
Q.lS. The stationary point of the function f (x, y) = i +3x
2
+4xy + x
3
is
(a) (b) ( ~ , i ) ( - ~ ' i )
(c) (d) ( ~ ' - i ) ( - ~ ' - i )
az az.
Q.19. If z = log
(
eX +e
Y
)
, then -+-IS
ax ay
(a) 1 (b) -1 (c) o (d) eX +e
Y
Q.20. For the function f (x, y) = i - x
2
,the point (0,0) is a
(a) point of maxima (b) point of minima
(c) saddle point (d) . nothing can be said
x(y-2)
Q.21. The value of the lim ( . is
(x.Y)....(1,l) Y x- 2
(a) 1 (b) -1 (c) o (d) 2
4
Q.22. The value of lim -; + _ is
x....
oo
.Y....2 X +2y
(a) 1 (b) -1 (c) o (d) 2
3
az az.
Q.23. If z = I (x- y),then +-IS
. ax ay
(a) 0 (b) 1 (c) 2 (d)
z=l(x-y)
Q.24. The degree of the homogeneous function I (x, y) = ( ax-p +by-P)-1/Pis 1 if
(a) P= 0 (b) P= 1 (c) P= 2 (d) For any value of P
Key.
1. (d) 2. (a) 3. (a) 4. (a) 5. (b) 6. (c)
7. (b) 8. (c) 9. (b) 10. (a) 11. (c) 12. (c)
13. (a) 14. (b) 15. (a) 16. (d) 17. (a) 18. (c)
19. (a) 20. (c) 21. (a) 22. (c) 23. (a) 24. (d)
'<
.----------
.
1. Euler's theorem is applicable only for homogeneous function.
2. The degree of the homogeneous function is always an integer.
2
3. The degree of the homogeneous function I (x, y) =
x +
is zero.
xy
2 2
' f . f . bl d h a Z a Z . I
4. If z IS a unction 0 varia es x an y t en --= -- IS a ways true.
axay ayax
(
x4-l] au au 4
5. If u=log 3 3 ,then x-+ =-.
x +y ax ay 3
x
3
+ xy2 + l au au
6. If u = 2 2 ,then x-+ y- = u.
x+y ax ay
7. The conditions for I (x, y) to be maximum or minimum is
Ix = O,fy = O,fxx.fyy - (Ixy )
2
= 0 .
8. If l(x,y)=O,then dy =
dx Iy
9. A function I (x, y) is a homogeneous function if and only if I (AX, Ay) = A?I (x, y) .
Xl/4 + 1/4
10. The function I (x, y) = 1/5 y'l< is a homogeneous function.
x +y
4
Key
1. T 2. F 3. T 4. F 5. F 6. T
7. F 8. T 9. F 10. T
Short Answer Type Questions:
a
2
z
2
a
2
z
Q.l. If z=f(x+ay)+g(x-ay),showthat -2=a -
2

ay ax
Sol. z =f(x+ay)+ g (x-ay) .... (1)
Differentiating (1) partially with respect to 'x', we get
az =f'( x+ay) +g'( x-ay) .... (2)
ax
Differentiating (2) partially with respect to 'x', we get
2
-2
az
=
)"(
x-ay
f"(
x+ay +g
)
... (3)
ax
Differentiating (1) partially with respect to 'y, we get
az =af'(x+ay)-ag'(x-ay) ... (4)
ay
Differentiating (4) partially with respect to 'y, we get
~ : ; =a
2
f"(x+ay )+a
2
g"(x-ay)
~ : ; =a
2
[f"(x+ay)+g"(x-ay)] ... (5)
Using (3) in (5),we get
a
2
z 2 a
2
z
-=a
ay2 ax
2
.
5
L
. ,1 a ( 2au) au
Q.2. If U=tne 41 Ifmd what value ofn will make 2"-a r - =-,
r r ar at
Sol.
u= tne
... (1)
Differentiating (1) partially with respect to 't', we get
r
n1
r2
au =tne
(
4t2
2
J+nt - e-41
at
2
e-
4i
(2 J
=t
n
-
rZ
r +nt ..,(2)
4
Now, differentiating (1) partially with respect to 'r', we get
aU -- r2 () 2r 1 -- r2
n n 1
a; = t e 41 - 4t =- 2' t - r e 41
Now/consider left hand side
-
_ 1 a[. 2 [
--t
n-l
--
1 a (
r
2 au)
--- r --t
1 n-l
re
-frJ]-
---
1 a ( 1
re
3 -fr J
r2 ar ar r2 ar 2 r2 ar 2
1
t
n
--t
1 n-l -f, (
- -.!.(-.!.. .3r
2
- e r e
3 2r) J
- r2 2
2 4t
r2 r2
3 -- 1
=__tn-1e 41 __ r
4
t
n
-
1
e 41
,.. (3)
2 4
We are given that au) =au
r ar ar at
r2 r2 r2(2 J
::::>
_'!"r
4
t
n
-
1
e-
4i
=t
n
-
2
e-
4i
(using (2) and (3))
2 4 4
--
r2
3
Compairing the coefficients of tn-Ie 41 from both sides ,we get n = --.
. 2
6
1
au au
X3 -lJ
Q.3. If u =sec- .Show that x-+ y- =2cotu.
(
x+y ax cry
1 X3 - 3 J
Sol. u = sec- Y
(
x+y
X
3
-y
3J
~ secu=
(
x+y
x -y
3 3 J
Let z = sec u =
(
x + y .
x3 _
Here, z is a homogeneous function of degree 2 as for f (x, y) = y
x+y
2 3
f(ltx,lty)=Il?x
3
-Il?l = 1t (x -l) 2
Itx +Ity x + y =It f (x, y) .
By Euler's theorem,
az az
x-+y-=nz=2z
ax cry
Substituting the value of z in above equation,we get
au au
xsecu tanu ax + ysecu.tanu cry =2secu
au au secu
~ x-+y-=2 =2cotu
ax ay sec u. tan u
Hence proved.
au au
Q.4.
If u =log
Fx
Fx
-
+
JY
JY
J.Show that x-+y-=O.
(
ax ay
(
Fx-JYJ
Sol.
u = log Fx+JY
Fx-JY
~ e u =Fx+JY
7
L
3
-rx-JY U
Let z=e =-rx +JY .
Here, z is a homogeneous function of degree 0 as for f (x, y) =Jf- -1. ;
x+... y
f(AX,AY) = .JTx-JiY = ...fi(-rx -JY) 0
.JTx+JiY ...fi (-rx +JY) =A f (x, y) .
By Euler's theorem,
az az
x-+y-=nz=O.z=O
ax ay
substituting the value of z in above equation/we get
au au
xe
U
+ yeu-=O
ax ay
u (au au]
=>
e x ax + yay =0
au au
=>
xax +Yay =0.
Hence proved.
Q.S. If z is homogeneous function of degree n,show that
2
2 a
2
z a
2
z 2 az
x -+2xy-+y -=n(n-l)z.
ax
2
axay ay2
Sol. Since z is a homogeneous function of degree n,by Euler's theorem
az az
x-+y-=nz ...(1)
ax ay
Differentiating (1) partially with respect to 'x', we get
az a
2
z a
2
z az
-+x-+y--=n
ax ax
2
axay ax
8
a
2
z a
2
Z aZ
=> x-+ y-=(n-l)- ... (2)
aX
2
axay ax
Multiplying (2) by x ,we get
a
2
2 a
2
Z z ( ) az
x -+xy--= n-l x- ... (3)
ax
2
axay ax
Differentiating (1) partially with respect to 'V, we get
a
2
z az a
2
z az
x
aXay
+ay +Yal =nay
a
2
z a
2
z az
=>
x-+y-=(n-l)- ...(4)
axay ay2 ay
Multiplying (2) by Y,we get
a
2
z 2 a
2
z az
xy-+y -=(n-l)y- ...(5)
axay ay2 ay
Adding (3) and (5)/we get
2 a
2
z a
2
z
2
z ( )( az azJ ( ) 2 a
x -+2xy--+y -= n-l x-+ y- = n-l nz (from(l))
a
ax
2
axay ay2 ax oy
2
z a
2
z 02
Z
=> X2-2 +2XYa ~ . +l-a2 =n(n-l)z.
ox Xuy y
x
3
+l
Q.6. Verify Euler's theorem for the function z = ---"-
xy
x3 + 3
Sol.
Z= Y
xy
It can be easily verified that z is a homogeneous function of degree 1.
By Euler's theorem,
oz az
x-+y-=nz=1.z=z ...{1)
ax ay
Verification:
9
L
x
3
+y3 X2
z= =-+
...{2)
xy Y x
Differentiating (2) partially with respect to 'x', we get
az 2x l
... (3)
ax =-Y-7
Multiplying (3) by x,we get
az 2X2 y2
x-=--- ...{4)
ax y x
Differentiating (2) partially with respect to 'v', we get
2y
az =
x
2
+
... (5)
ay x
Multiplying (5) by Y,we get
az _
... (6)
y ay - y
x
Adding (4) and (6),we have
2 2
az az 2X2 l x 2l x
x-+ =-----+-= + =---'-
ax ay y x y x y x xy
=>
az az
x-+y =z.
ax ay
Q.7.
Sol.
Hence proved.
Examine f (x, y) = x
3
+l-3xy for maximum and minimum values.
f (x, y) = x
3
+l-3xy
f =3X2_3yf =3y2
x 'y
3x'f =6x'f =-3-f =6y
'xx 'xy 'yy
For maxima and minima, fx =0 , fy = 0
=> 3x
2
-3y =0 ,3l-3x=O
10
=> X
2
= Y , y2 = X
=> (X
2
t = x or X4 = x or X4 - x = 0 or x (x
3
-1) = 0
=> x(x-l)(x2 +x+l)=O or x=O,1
For x=0,y=(0)2:::::0 and for x=l,y=(1)2 =1
The two stationary points are (0,0) and (1,1).
(0,0) (1,1)
0 6
fxx =6x
-3 -3
fxy =-3
0 6
fyy -6y
27 -9
fxx.f
yy
-(fxyt
From the above table,we see that at the pOint (0,0), fxx.f
yy
_(fxy)2 <O.
Q.S.
Sol.
:. (0,0) is neither a point of maxima nor a point of minima.
At the point (1.,1), fxx.f
yy
-(fxy t >0 and fxx >0,:. (1,1) is a pOint of minima.
The minimum value is f (1,1) =1 +1-3 =-1.
Find the extreme values of the function f (x, y) = x
3
l (1- x - y) .
f (x, y) = x
3
l (1- x - y)
fx =x
3
l (-I)+(1-x- y).3x
2
l =x
2
l (-x+3-3x-3y) =x
2
l(-4x-3y+ 3)
fy =x
3
l (-1) +2yx
3
(1- x - y) = x
3
Y ( - Y +2- 2x - 2y ) =x
3
Y ( - 2x - 3y +2 )
For extreme values, fx = 0 , fy = 0
11
L
:::}
x
2
y2 (-4x-3y+3) = 0 and x
3
y(-2x-3Y+2)=0
:::}
-4x-3y+3=0 and -2x-3y+2=0
:::}
4x+3y=-3 and 2x+3y=-2
Solving above two equations for x and Y,we get
..... \ ;; ..... \
_1 _1 A.'2- - ; ~ ::. 7"
x-
Z
,y-3" " ./
. (.1 ..1)
The extreme value of the given function is ~ Z' 3" .
~ ~
Q.9. If (cx-az,cy-bz
)
=O,showthat ap+bq=c,where p=-&q=
ax dy .
Sol. let u = cx-az and v = cy -bz.
Then (u,v)=O ...(1)
Differentiating (i) partially with respect to 'K, we get
a au +a av == 0
au ax avax
a a
:::}
-(c-ap)+-(-bq)=O ... (2)
au av
Differentiating (1) partially with respect to 'V, we get
a au +a av =0
au dy av dy
:::} a (-ap)+ a(c-bq) =0
... (3)
au av
Eliminating a and a from (i) and (2),we get
. au av
c-ap
-bq 1=0
-ap c-bq
:::}
(c-ap)( c-bq)-abpq =0
12
:=}
c
2
-cbq-apc+abpq-abpq = 0
:=}
c
2
-cbq-apc =0
:=}
c ( c - bq - ap) =0
:=}
ap+bq=c (.: c;t:O)
Hence proved.
Q.l0.
Sol.
( )
au au au
If u =u y-z,z-x,x- y ,prove that -+-+-=0.
ax ay az
let a=y-z,b=z-x,c=x-y.
Then u=u(a, b, c) ...(1)
Differentiating (1) partially with respect to 'x, we get
au au aa au ab au ac
-=--+--+-
ax aa ax ab ax ac ax
:=}
au = au (0) + au (-1) + au (1) = _ au + au
ax aa ab ac ab ac
...(2)
Differentiating (1) partially with respect to 'y', we get
au au aa au ab au ac
-=--+--+-
ay aa ay ab ay ac ay
:=}
au = au (1)+ au (0)+ au (-1) = au _au
ay aa ab ac aa ac
...(3)
Differentiating (1) partially with respect to I Z', we get
au au aa au ab au ac
-=--+--+-
az aa az ab az ac az
:=}
au = au (-1) + au (1) + au (0) = _ au + au
az aa ab ac aa ab
...(4)
,
"
Adding (2),(3) and (4) ,we get
13
au + au +au =.
ax ay az
Hence Proved.
0.11. Expand eX cos y inpowers of x and y as far as terms of second degree.
Sol. I (x, y) =eX cos y
x=o,y=o
I (x, y)
eX cos y
1
. Ix (x,y)
i
eX cos y
1
Iy(x,y)
_ex siny 0
Ixx (x, y)
eX cos y
1
Ixy (x, y)
X .
-e smy
0
Iyy (x, y)
'- -
_ex cosy -1
.
By Taylors theorem,
a a) 1 (a a)2
l(x,y)=/(o,O)+ x-+y- 1(0,0)+- x-+y- 1(0,0)+ ...
(
ax dy 2! ax dy
2
x 2xy l
=I(O,O)+xf (0,0)+ yly (O,O)+-Ixx (O,O)+-I (O,O)+-I (0,0)+ ...
xy yy
x 2! 2! 2!
2
x 2xy l
=l+x(l)+ y(O)+-(l)+-(O)+-(-1)+ ...
2! 2! 2!
+...
2!
x
2
l
=l+x+---+...
2! 2!
14
Q.12. If f(x,y)= 2x-y ,show that Iim[Iimf(x,y)] * Iim[limf(x,y)].
2x+ y x-+o y-+O y-+O x-+o
2x-y
Sol. f(x,y)=-
2x+y
l.H.S
2x- y] Ii [2X-OJ Ii (1) 1
Iim
[Ii
mf
(x,y)] =Iim m = m -- = m =
x-+o y-+O _ x-+o
[Ii
y-+O 2x+y x-+O 2x + 0 x-+O
R.HS.
Iim[Iimf (x,y)] =Iim[Iim 2x- y] =Iim[O- y] =lim ( -1) :::;-1
y-+O x-+o y-+O x-+O 2x+ y x-+O 0+ y x-+o
l.H.S* R.H.S.
Hence proved.
Long Answer Type Questions:
Q.l. If Z =f (u) is a homogeneous function of x,y of degree n, then
d
2
U d
2
U d
2
U
x
2
dX
2
+ 2xy dXdy + l dy2 =g (U)( g' (U)-1)
f(u)
where, g ( u) =n f' (u)
Sol. Since z is a homogeneous function of degree n,by Euler's theorem
dZ dZ
x-+y-=nz ...(1)
dX dY
It is given that, z:::; f (u) .
dZ :::; f'(u) dU
and
dZ =f'(u) dU
... (2)
dX dX dy dy
Using (2) in (1),we have
I
x.f'(u) dU +y.f'(u) dU:::; nf(u)
dX dY
15
Lw
du du /(u)
=>
x-+y-=n-
dx dy f'(u)
du du
=>
x-+y-=g(u) ... (3)
dx dy
Differentiating (3) partially with respect to lx', we get
2
d2U du J d u , du
x-+-.l +y-=g (u)
(
dx
2
dx dxdy dx
d
2
u d
2
u I du
=>
X-+y-=(g (u)-l)- :..(4)
dx
2
dxdy dx
Multiplying (4) by x ,we get
2 d
2
u . d
2
u du
X -2+xy-=(g
I
(u)-l)x.- ...(5)
dx dxdy dx
Differentiating (3) partially with respect to 'V, we get
2 2
d u (d u du J ' du
x dXdy + y dy2 + dy.1 = g (U) dy
d
2
u d
2
u , du
=>
y-+X-=(g (u)-l)- ...(6)
dy
2
dXdy dy
Multiplying (6) by y ,we get
2 d
2
u d
2
u, du
y -+xy-=(g (u)-l)Y.- ... (7)
dy2 dxdy dy
Adding (5) and (7) ,we get
2 2 2
2 d u d u 2 d u , (dU du J
x -+2xy-+y -=(g (u)-l) x-+y
dx
2
dxdy dy2 dy dy
=g(u )(g'(u)-l).
Hence proved.
16
. +
Q.2. If u =sm 11 11 ,Prove that
X76 +y76
2 2 2
2a u a u 2a u 1 (2 )
X -2+2xy--+y -2 =-tanu tan u-ll ,
ax axdy dy 144
. -1 + yX; J
Sol. u =sm 11 11
X76 +y76
Let z =sin u =xX; +yX;
xl(; +yl(; .
1 xX; +
Here, z is a homogeneous function of degree - as for f (x, y) = l(; l(; ;
12 x 6 + Y 6
":-':'..,.
+(AY)X; AX; + yX;) +yX;)
f(Ax,AY) l(; 11 = =A){2 A){2f ( )
(AX) 6+ ( Ay)'6 Al(; (x,% + y'%) (x,% + y'%) = x, y .
We know that if z =f (u) is a homogeneous function of degree n,then
2 2 2
a u a u a u I
x
2
ax
2
+2xy axay +l dy2 =g(u)(g (u)-1) ..,(i)
f(u)
where, g (u) =n f' (u )
1
Here, n = 12
z = f (u ) = sin u
=> f' (u) = cos u
f(u) 1 sinu 1
=>
g(u)=n--=---= tanu (2)
f'(u) 12 cosu 12
Using (2) in (l},we get
17
L
2 2 2
2 a U a U 2 a U 1 ( 1 2 )
X -+2xy--+y -=-tanu -sec u-l
2
ax axdy dy2 12 12
2
1 (sec u-12J
=-tanu
12 12
2
1 (tan U-llJ
=-tanu
12 12
1
2
=144 tanu(tan u-ll)
2 2 2
-I [ X+ y J 2 a u a u a u
sinucosu
Q.3.
If u=sm ,Prove that x -2+2xy--+i-=
x- y ax axay ay
2
4cos
3
.
, -1 [ x+ y J
Sol.
u=sm
x+y
Let z =sin u= -.JY .
1 x+y
Here, z is a homogeneous function of degree - = as for f (x, y) .JY;
2 x- y
f(AX,AY)== AX+AY = A(X+y) =Ali x+y =Alif(x,y).
ffx-jiY
We know that if z == f (u) is a homogeneous function of degree n,then
2 2 2
2a U a u 2a U ()('() )
x -+2xy-+y -=g u g u -1 ...(1)
2
ax axay dy2 ,
f(u)
where, g (u) = nf' (u)
Here, n =2"
1
z =f (u) =sin u
18
=> f' (u) =cos u
f (u) _.!.. sin u
...(2}
=>
g(u)=n I'(u) - 2cosu
f(u) __1
=>
g'(u}=n I'(u) - 2cos
2
u
Using (2) in (1),we get
2 a
2
u 2 a
2
u 2 azu 1 sin u (1 1)
x -+ xy--+y -=---: --
2
ax axay al 2 cosu ,2cos
2
U
=.!.. sinu (1-2COS
2
u)
2 cosu 2cos
z
u
=.!.. sinu ("-COS2UJ
2 cosu 2cos
2
U
sinucosu
- 4cos
3
u
Hence proved.
Q.4.
3 ) h t (a u= ___9 ..
If u =log (x
3
+l +Z - 3xyz ,prove t a ax ay az (x + y +z)
Sol. u=1og(x
3
+l+z
3
-3XYz) ...(1)
Differentiating (1) partially with respect to ')(, we get
au 1 a(3 3 3 3) 3x
2
-3yz
-= .- x +y +z - xyz
ax x
3
+l +Z3 - 3xyz ax x
3
+l +Z3 - 3xyz
au 3x
2
-3yz
=>
...(2)
-
ax
=x
3
+ l +Z3 - 3xyz
Differentiating (1) partially with respect to 'V, we get
au 1 a (3 3 3 3) 3l-3xz
- = .- x +y +Z - xyz =--:-----''':---:--
ay x
3
+l +Z3 - 3xyz ay x
3
+l +Z3 - 3xyz
19
au 3l-3xz
=>
,.. (3)
ay = x
3
+ y3 +Z3 -3xyz
Differentiating (1) partially with respect to i Z', we get
au 1 a' 3z
2
-3xy
--::-_-:-----:-__._(x
3
+l + Z3 - 3xyz) =
az x
3
+ l + Z3 - 3xyz az x
3
+ l +Z3 - 3xyz
au 3z
2
-3xy
=>
... (3)
'az = x
3
+ l +Z3 -3xyz
Adding (1),(2) and (3),we get
2
au au au 3 ( x + l +Z2 - xy - yz - ZX)
- + - +-= ---=--....,.---..,,-....,.---_..:..
ax dy az x
3
+l +Z3 - 3xyz
3 ( x
2
+ y2 +Z2 - xy - yz - zx) 3
(x+y+z}(X
2
+y2+
Z
2_xy_yZ-ZX) = x+y+z
au au au 3
=>
-+-+-=-
ax dy az x+ y +z
Now,
(
a a aJ2 (a a aJ ,3
ax + ay + az u= ax + ay +az x+ y + z
=
3 3 3
- (x+y+zt (x+y+zt (x+y+zt
9
=--- 2'
(x+ y+ z)
Hence proved,
20
Q.5. Find the expansion for sin x sin y in powers of x,y upto fourth order terms.
Sol. Let f (x, y) = sin x sin y
i
x=O,y =0
0 sinxsiny
f(x, y)
0 cosxsin y
fx (x, y)
0 sinxcosy
fy (x, y)
0 -sin xsin y
fx2 (x, y)
cos xcos y
1
fxy(x,y)
-sin xsin y 0
fi (t,y)
-cosxsiny 0
. f ~ (x, y)
0 -sinxcos y
fX2y (x, y)
0 -cosxsin y
fxy2(X,y)
0 -sinxcos y
fydx,y)
0 sin x sin y
fx4 (x, y)
cosxcos Y
-1
fxJ
y
(x, y)
0 sin x sin y
fx2i (x,y)
cos x cos y
-1
fxy3(X,y)
0 sinxsin y
fl (x,y)
21
L
By Taylor's theorem,
I (x. y) =I (O,O)+;ifx (0,0)+ yly x
2
Ixx (0,0) +2xylxy (0,0) +llyy (0,0)J +
2. .
3
I
x
2
;![x 3(0,0) +3x yfx2y (0,0)+3xy2 Ixy2 (0,0) +lly3 (0,0)] +
[X4 IX4 (0,0) +4X3Ylx3y (0,0)+6x
2
l Ix2/ (0,0) +4xy3Ixy3 (0,0)+ lly4 (0,0)]+...
Substituting the values from above table,we get
sin x sin y =+x(0) +y (0) + [ x
2
(0) +2xy (1) +l (0)J +
;![x
3
(0)+3x
2
y(0)+3xy2 (0)+ l (O)J+
X4 (OJ+4x
3
y(_1)+6x
2
y2 (0)+4xy3 (-1)+ y4 (O)J+ ...
3
sin xsin y = xy -.!. (x y +xy3 )+'"
6
a.6. Expand x
2
y +3y - 2 in powers of x -1 and y +2 using Taylor's theorem.
Sol. I(x, y) =x
2
y+3Y-2
By Taylor's
theorem,
2 2 2
l(a+h,b+k)=/(a,b)+(h
al
+-.!..(h
2
a { +2hk aL+k
2
a {) +
ax OY)(a.b) 2! ax aXuy dy (a,b)
a
3 3
3 a
3
3 a
3
1 ( I 2 I 2 aI I )
- h -3+3h k:\ 2:\ +3hk :\ +k -::;-T +... ...(1)
3! l ax ox oy oXuy oy (a,b)
Here, a+h=xandh=x-l=>a=l
b+k =yand k = y+2=>b=-2
22
I(x,y)
Ix (x, y)
Iy(x,y)
Ixz(X, y)
:
Ixy (X, y)
Ii (x,y)
(x,y)

Ixyz(x,y)
Ii (x,y)
x
2
y+3y-2
2xy
x
2
+3
2y
2x
0
0
2
0
0
x=l,y =-2
1
-10
-4
4
-4
2
0
0
2
0
0
substituting the values from above table in (l),we get
x
2
y+3y - 2=-10+( (x-l)( -4) +(y +2)(4))+ ((X_l)2 (-4)+ 2( x-l)(y + 2)(2)+( y+ 2)2 (0))
(0)+3(x-l)2 (y+2)(2)+3(x-l)(y+2)2 (0)+(y+2)3 (0))+ ...
x
2
y +3y - 2 =-IO-4(x-l) +4(y + 2)-2(x-I)2 + 2(x-I)(y +2) +(x_I)2 (y + 2) +...
Q.7. Find the extreme values of I(x, y) = x
3
+3xy2 -l5x
2
-l5l +72x .Also find the
points of relative maxima and minima.
Sol. I(x, y) =x
3
+3xy2 -l5x
2
-l5l +72x
Ix = 3x
2
+3l-30x+72
Iy =6xy-30y
23
L
fxx =6x-30,fxy =6y,f
yy
=6x-30
For extreme values, fx = 0 , fy = 0
Now, fx = 0 gives
3x
2
+3y2 -30x+72 =0 ...(1)
Also, fy =0 gives
6xy-30y=0
~ 6y(x-5) = 0
~ y=0,x=5 ...(2)
substituting the value of y from (2) in (l),we get
3x
2
-30x+72=0
~ x
2
-lOx+24=0
~ x=4,x=6
:. The extreme values are (4,0),(5,0)and (6,0).
(4,0) (5,0) (6,0)
fxx =6x-30 -6
0 6
fxy = 6y
0 0 0
fyy =6x-30 -6
0 6
fxxfyy -(fxy t
36 0 36
From the table,we see that f xx.f yy - (fxy )2 =0for the point (5,0).
:. The point (5,0) is neither a point of maxinia nor a point of minima.
24
At the point (4,0), f:o: < 0, :. (4,0) is a point of maxima.
At the point (6,0) I f:o: > 0 , :. (6,0) is a point of minima.
3
b
3 3
a c
Q.8. If u =-2+-2+2" ,where x + y +z =1 .prove that the stationary value of uis given by
x y z
abc
x= ,y= ,z=--
a+b+c a+b+c a+b+c
3
b
3 3
a c
Sol. f=-+-+
x
2
l l
x+y+z=l => x+y+z-l=O
Take g = x+ y +z-1.
Let F=f+Ag
3
b
3 3
a c
=>
F =-+-+-+A(X+ y+z-l)
2
x l Z2
2a
3
2b
3
2c
3
=> F. =--3+A; Fy =--3+A; F
z
=--3+A;F,t =x+y+z-l
x Y z
For stationary values,
Fx = 0, Fy = 0, F
z
= 0, F,t = 0
For Fx =0,
3
2 x
3
2a +A= 0 => 1 = a ...(1)
- 3 /1, 3
X
For Fy =0,
3 2_L
3
2b +A=0 => 1 - b
3
...(2)
- 3 /1,
Y
For F =0 I
z
~ -
3 __
2 l
2c +A= 0 => ') - c ... (3)
- 3 /1, 3
Z
25
L
For FA, =0,
x+y+z-l=O x+y+z=l ... (4)
From (1),(2) and (3),we get
3 3 Z3
-=-3
Y --=k(say)
a
X
b
-
3
... (5)
Using (5) in (4),we get

(a +b +c}=l
__l
... (6)
(a +b +
substituting (6) in (5),we have
abc
x= ,y= ,Z=--.
(a+b+c) (a+b+c) (a+b+c)
Hence proved.
Q.9. Divide 24 into three parts such that the continued product of the first, the square of the
second and the cube of the third may be maximum.
Sol. Let x,y,z be the three numbers such that x +y +z = 24.
We need to find the maximum value of xy2Z3 given that x + y +z = 24.
Take J = xy2
Z
3 and g =x+y+z-24
Let F=J+Ag
F =xy2
Z
3 +A(X+ y+ z-24)
Fx =y2Z3 +A; Fy =2
XYZ
3 +A; F =3xy2Z2 +A; FA, =x + y +Z- 24
z
26
For stationary values,
Fx =O,F
y
=O,F
z
= 0, FA =0
For Fx =0,
lz3+ A=0
=>
A=_y2
Z
3
...(1)
For Fy =0 ,
2
XYZ
3 +,1,=0
=>
A=-2
xyz
3
...(2)
For F
z
=0,
3xy2
Z
2 +,1,=0
=>
A=-3xy2
z
2
...(3)
For FA =0,
x+ y+z-24=O => x+y+z=24 ... (4)
From (1) and (2),we get
y2
z
3 =2
XYZ
3
=>
y=2x ...(5)
From (1) and (3),we get
y2Z3 = 3xy2Z2
=> z=3x ... (6)
Using (5) and (6) in (4),we get
x+2x+3x=24 => x=4
x=4,y=8,z=12.
Q.l0. Using Lagrange's method of multipliers ,find the shortest distance of the point (1,2,2)
to the sphere x
2
+ l + Z2= 36.
Sol. Let P ( x, y, z) be any point on the given sphere.
The distance between the points (x, y, z) and (1,2,2) is
d =J(X-I)2 +(y_2)2 +(Z_2)2
27
L
Let J (x, y, z) =(x_l)2 +(y - 2)2 +( z- 2)2
And g(x, y, z) =x
2
+ y2 +Z2 - 36 .
Let F=J+Ag
~ F =(x_l)2 +(y_2)2 +(Z-2)2 +A(X2 + y2 +Z2 -36)
Fx =2(x-l)+Ax=(2+A)x-2
Fy =2(y - 2) +AY =(2 +A) y - 4
=2{z -2)+AZ =(2+A)Z-4 F
z
FA, =x
2
+ l +Z2 -36
For stationary values,
Fx =O,F
y
=O,F
z
=O,FA, =0
2
Now, Fx =0 ~ ... (1)
x=2+A
4
F
y
=0 ~ ...(2)
y=2+A
4
F =0 ~ ...(3)
z
Z=2+A
FA, =0 ~ x
2
+l +Z2 =36 ...(4)
substituting the values of x,y, and z from (1),(2) and (3) in (4),we get
2 )2 ( 4 )2 ( 4 )2
- +- +- =36
(
2+A 2+A 2+A
~ 4+16+16=36(2+A)2
~ 36=36{2+A)2
28
~ (2+Af =1
~ 2+2=1 or 2+2=-1
~ 2=-1 and 2=-3
For 2 = -1, x = 2, Y = 4, z = 4 .
For 2=-3, x=-2,y=-4,z=-4.
:. we obtain two points (2,4,4) and (-2,-4,-4).
The distance of the point (1,2,2) from the point (2,4,4) is 3 and the distance of the
point (1,2,2) from the point (-2,-4,-4) is9.
Hence the shortest distance of the point (1, 2, 2) from the given sphere is 3.
Q.ll. A rectangular box ,which is open at the top ,has a capacity of 256 cubic meter, Determine
the dimensions of the box such that the least material is required for the constructions of
the box. use Lagrange's method of multipliers to obtain the solution.
Sol. Let x,y,z be the length ,breadth and height of the box.
It is given that,volume of box is 256 cubic meter.
~ xyz =256
Let g = xyz - 256
Let 5 be the surface area of the box. Then
S =xy+2yz+2xz
Let F =S+2j.
~ F =xy+2yz+2xz+2(xyz-256)
~ Fx = y + 2z + 2yz ; Fy = x + 2z + 2xz ; F
z
= 2 y + 2x + 2X)' ; FA, =X)'Z - 256
For stationary values,
Fx =0, Fy =0, F
z
=0, FA, =0
Now, Fx =0 ~ y+2z+2yz=0 or y+2z=-2yz ... (1)
29
L
Fy =0 => X+2Z+AXZ=0 or X+2Z=-AXZ ...(2)
F
z
=0 => 2y+2x+AXY =0 or 2y+2x=-AXY ...(3)
FA = 0 => xyZ 256 = 0 or xyZ = 256 ... (4)
Multiplying (1) by x ,we get
xy +2xz = -AxyZ = -256 A ... (5)
Multiplying (2) by Y,we get
xy+2YZ=-AXYZ=-256A ...(6)
Multiplying (3) by Z ,we get
2zy +2xz = -AxyZ = -256 A ...(7)
From (5) and (6),
xy+2xz = xy+2yz => x= y
From (6) and (7),
xy+2yz = 2yz+2xz . => y = 2z
Substituting the values of x and y in (4),
y.y.[ ~ ] = 256
=> y=8
=> x=8,z=4
Hence length =8m, breadth=8m and height=4m.
30
Unit - 2 : Partial Differential Equations
Multiple Choice Type Questions:
Q.l. If the unknown function in a differential equation depends on more than one
independent variable then the differential equation is said to be
(a) ordinary differential equation
(b) partial differential equation
(c) homogeneous differential equation
(d) none of above
' I d'ff ' I . az az ,
Q.2. The or der 0 f t he partla I erentla equation y - - x - = Z IS
ay ax
(a) 1 (b) 2 (c) a (d) None of the above
2 2
' I d'ff ' I ,az az 3az ,
Q.3. The or der 0 f t he partla I erentla equation -2- -2+ - = xy IS
dy ax ax
"'.:.':<
(a) 1 (b) 2 (c) a (d) None of the above
Q.4. Which of the following equation partial differential equation is a linear in p and q
(a) pq=l (b) p(l+q)= qz
(c)zp+ yq =x (d) pq+ p+q=O
2
, f az 2 '
Q.S. The soIutlon 0 --= xy IS
axay
X
2
y
3
22)
(a) -+f(y)+g(x) (b) ~ + f(y)+g(x
6 6
3 3
32)
(c) ~ + f(y)+g(x) (d) ~ + f(y)+g(x
6 6
Q.6. The relation Z = (x +a) (y +b) represents the partial differential equation
(a)
z= p/q
(b) z= pq
(c) z=p-q (d) z=p+q
x
2
l
Q.7. The relation 2z = -2+-2 represents the partial differential equation
a b
(a)
z= p/q
(b)
z= p-q
(c) 2z == xp + yq (d) z=p+q
31
,-,
Q.8. The relation z = f (x
2
-l)represents the partial differential equation
(a) yp+xq=O (b) yp-xq=O
(c) xp+ yq=O (d) xp- yq =0
Q.9. Elimination of a function f from z= f ( ~ J. gives a partial differential equation
oz oz oz oz
(a) x-+-=O (b) x---=O
ox cry ox cry
oz oz oz oz
(c) y-+-=O (d) x-+y-=O
ox oy ox cry
Q.l0. The partial differential equation formed by eliminating arbitrary functions from
z=f(x+at)+g(x-at) is
02
Z
_a202
Z
=0 02
Z
+a202
Z
=0
(a) (b)
2 2 2
ot ax
2
ot ox
2
02Z
02Z_
02
Z
02
Z
2
(c) a -+--0 (d) -+-=a
2 2 2 2
ot ox ot ox
Q.ll. The solution of partial differential equation OZ + OZ = ~ is
ox cry a
(a) z =e
x
/
a
f (x + y) (b) Z = ex/af (x - y)
x a
(c) Z = e / (d) None of the above
Q.12. Which of the following is lagrange's linear equation(P,ctR are any functions of XN and z)
(a) Pp2+Qq=R (b)
pp2+Qq2 =R
(c) Pp+Qq=Rpq (d) Pp+Qq=R
Q.13. The partial differential equation l x oz +ZX2 OZ = xy2 is
ox cry
(a) f(X3+l,X2+l)=0 (b) f(x
3
-l,x
2
-l)=0
(c) f (X3 + l, x
2
-l) = 0 (d) f (X3 -l, x
2
+ l) =0
32
Q.14. The auxiliary equations for the Lagrange's equation Pp +Qq =Rare
dx == dy = dz
(b) dx = dy = dz
(a)
P Q R Q P R
dx dy dz
(d) dx == dy == dz
(c) -=-==
P R Q R Q P
Q.15. The relation z =(x+ a)2 + y represents the partial differential equation
(a)
z =.!.(az)2
(b)
z =.!.(azJ2
4 ax +y' 4 ax -y
(c) (d)
- .!. (az J2 +X
z- 4 ay
. az az.
2
Q.16. The general Integral of - == 3 - IS
ay
(
ax
)
(a) z=ax+3ay+c (b) z=ax+3a
2
y+c
(c) z =a
2
x+3ay+c (d) z=ax+3y+c
Q.17. The equation az
az
e
X
gives t
h
e genera
I I .
== so utlon
ax
ay
x
(a) z == ae +be
Y
b) z== i' +e
Y
(c) z == a ( eX +e
Y
) +b d) None of the above
2 2 2
a z a z a z
Q.18. The complementary of the partial differential equation -2- 4--+4-== 0
z
ax axay ay
(a) h(y+2x)+xlz(y+2x) (b) h(y-2x)+.if
z
(y 2x)
(c)
h (y +2x) +12 (y 2x)
(d) None of the above
. azu azu .
Q.19. The equation -2=c
2
IS a
at
(a) wave equation (b) heat equation
(c) Laplace equation (d) None of the above
33
Q.20. The relation z = ae
bt
sinbx gives the partial differential equation
(a)
a
2
z _ a
2
z=0
(b)
a
2
z+a
2
z=0
at
2
ax
2
at
2
ax
z
(c)
az + az =0
(d)
az _az =0
ax at ax at
Q.21. The general solution of the equation : : a ~ =0is
(a)
h(x)+ fz(Y)
(b) f(x- y)
(c) f(x+y) (d) None ofthe above
Key
1. (b) 2. (a) 3. (b) 4. (c) 5. (a) 6. (b)
7. (c) 8. (a) 9. (a) 10. (a) 11. (b) 12. (d)
13. (b) 14. (a) 15. (a) 16. (b) 17. (c) 18. (a)
19. (a) 20. (b) 21. (a)
True/False Type Questions:
1. A particular integral of a partial differential equation is a solution obtained from the
complete integral by assigning particular values to the arbitrary constants.
2. A partial differential equation is an equation involving one or more partial derivatives of an
unknown function of two or more independent variables.
3. Every partial differential equation is homogeneous.
4. The method of separation of variables is used to solve an ordinary differential equation.
5. A singular integral of partial differential equation is same as its particular integral.
6. A heat equation can be solved by using method of separation of variables.
a
2
z azz a
2
z
7.
Thesolutionof
axz
+4
aXaY
-Sal =0 is z=h(Y+x)+ fz(Y+Sx).
zy
a
z
z a
z
z e
X
+
8. The particular integral of
axz
- dy2 = e
X
+
zy
is --3-'
9. In a homogeneous partial differential equation with constant coefficients, all the terms
contain derivatives of the same order.
10. The complete solution of a partial differential equation is the sum of complementary
function and particular integral. .
34
11. The solution z(x, y) of the equation az =0 is z =f (y ) .
ax
12. The method of separation of variables converts a partial differential equation into a set of
one or more ordinary differential equations.
Key
1. T 2. T 3. F 4. F S. F 6. T
7. F 8. T 9. T 10. T 11. T 12. T
Short Answer Type Questions:
Q.l. Form the partial differential equation from (x - h)2 +(y - k)2 +Z2 = a
2
.
2
Sol.
(X-h)2 +(y_k)2 +Z2 =a
... (1)
Differentiating (1) partially with respect to lx', we get
az
2(x-h)+2z-=O
ax
az
=>
x-h=-z-=-zp ...(2)
ax
Differentiating (1) partially with respect to y, we get
2(y-k)+
az
=>
y-k =-z-=-zq ...(3}
ay
Using (2) and (3) in (1) ,we get
2
(_Zp)2 +(_Zq)2 +Z2 =a
2
=> Z2p2+Z2q2+Z2 =a
=> Z2 (p2 +q2 +1) =a
2
This is the required partial differential equation.
35
L
Q.2.
Sol.
..~ ': ,.<
Q.3.
Sol.
2 2
Form the partial differential equation from 2z =x2 +4 .,
a b
x2 2
2z=_+L
...(1)
2
b
2
a
Differentiating (1) partially with respect to 'x', we get
2
az
=2x
ax a
2
1 1 az
~ ...(2)
;;;= x ax
Differentiating (1) partially with respect to 'y', we get
2az =2y
ay a
2
1 1 az
~ ...(3)
b
2
=y dy
From (1),(2) and (3),we get
2z =.!. az x
2
+! az i
xax y dy
az az
~ 2z=x-+y-.
ax ay
2
'Id'ff '1 . az Y
SI o ve partla I erentla equation --= e cos x .
axdy
a
2
z
Y
...(1)
axay =e cos x
Integrating (1) w.r.t. 'x',we get
az = e
Y
sin x +f (y )
ay
Integrating above equation w.r.t. 'x', we get
. z = e
Y
sin x+ If(y) dy + g (x)
~ Z = e
Y
sin x + F(y)+ g (x) .
36
Q.4. Form the partial differential equation from z = f (x
2
+ l) .
Sol. Z=f(X2+l) ...(1)
Differentiating (1) partially with respect to 'x', we get
p =~ : =f'(x
2
+l).2x
..,(2)
q = ~ ~ = f'(x
2
+ l).2y
... (3)
Dividing (2) by (3),we get
p x
-=
q y
~ py=qx
~ py-qx=O.
Q.S. Solve l p - xyq = x ( Z- 2y) .
Sol. l p - xyq = x(z- 2y )
This is the Lagrange's linear equation.lts auxiliary equations are
dx dy dz
7= -xy =x(z-2y)
From first two ratio's ,we have
dx dy
-=
y -x
~ xdx=-ydy
Integrating both sides,we get
x
2
l
-=--+C
2 2
2
~ x +l =c
1
...(1)
From the last two ratio's,we get
dy dz
-y = (z-2y)
~ -zdy+2ydy =ydz
37
=> 2ydy =ydz +zdy =d (yz)
Integrating both sides,we get
yZ = yz+c
z
=>
yZ _ yz =c
z
...(2)
From (1) and (2),
xZ+l=f{l-yz}.
a.6. Solve p+3q=5z+tan(y-3x).
Sol. p+3q=5z+tan(y-3x)
This is the Lagral(lge's linear equation. Its auxiliary equations are
dx dy dz
"1="3 5z+tan(y-3x)
From first two ratio's ,we have
dx dy
-=
1 3
Integrating both sides,we get
1
=> x=-y+c
3
=>
y-3x=c
1
... (1)
From first and last ratio's,we have
dx dz
"1- 5z+tan(y-3x)
dx dz
=> -=-
1 5z+tanc
1
Integrating both sides,we get
1
x= -log( 5 z+ tan C
1
) + d
5
=> 5x = log(5z +tancl)+d
=> 5x-d =log(5z+tanc
l
) or e
5x
-
d
=5z+tanc\
38
5x
e
=C
2
=> SZ+tanc
1
5x
e
=>
=C
2
...(2)
Sz+tan(y-3x)
Form (1) and (2},we have
5x
e
f(y-3x) =Sz+tan(y-3x)
Q.7. Solve p(1+q)=qz.
Sol. p(1+q)=qZ
Let t=x+ay
at at
=> -=1-=a ...(1)
ax ()y
az az at dz
p=-=--=
ax at ax dt
az az at dz
q=-=--=a
()y at ()y dt
(1) reduces to
dZ( dZ) dz
- 1+a- =a-z
dt dt dt
dz
=> 1+a-=az
dt
dz az-1
adz =dt
=>
-=-- =>
dt a az-1
Integrating both sides,we get
log ( az -1) =t+c
=> log ( az -1) =x + ay + C
39
L
Q.8. Solve (D3 -6D
2
D' +12DDfl -8D'3)Z = o,where D = ~ , D ' = ~ .
ax dy
Sol. (D3 -6D
2
D' +12DD'2 -8D'3)z =0
Putting D = m and D' = lithe auxiliary equation is given by
m
3
-6m
2
+12m-8=0
=> (m-2)(m
2
-4m+4) =0
=> (m-2f =0
=> m=2,2,2
The solution is given by
Z= h (y +2x) +xft (y +2x) +x
2
h (y +2x) .
a
2
a
2
Z z x+2y
Q.9. Solve ----=e
ax2 ay2 .
Sol. The given equation can be written as
2y
a , a
(D2 _ D'2) z =e
x
+
where D =ax' D =dy
The auxiliary equation is
m
2
-1=0 => m=l
Complementary function is given by
G.F=h(Y+x)+ 12(Y-X)
Particular integral is given by
1
2 __ ex+2y
P.I.
_D'2
D
= 1 ex+2y = _.!..e
X
+
2Y
e-22 3
:. The complete solution is
2Y
Z=C.F.+ P.I.= h (y+x) +12 (y_x)_.!..e
X
+ .
3
40
d
2
d
2
d
2
Z z z .
Q.I0. Solve -2-2--:i:+-2 =smx.
dX dXvy dy
Sol. The given equation can be written as
d,d
(D2 - 2DD' +DI2 ) Z =sin x
where D= dX,D =dy
The auxiliary equation is
m2-2m+1=0
=> (m_1)2 =0 or m=1,1
Complementary function is given by
C.F. =.t; (y+x)+x f2 (y +x)
Particular integral is given by
P.l.= 2 1, . ~ sin x
D -2DD +D
1 . .
2 2 smx=-smx
(-1) -2(0)+(0)
:. The complete solution is
z=C.F.+ P.I.=.t; (y +X)+Xf2 (y+x)-sinx.
Q.ll. Eliminate the arbitrary constants a and b from Z =ae
bt
sin bx .
Sol. z=ae
bt
sinbx ...(1)
Differentiating (1) partially with respect to lX, we get
dZ b
dX =abe t cosbx
... (2)
Differentiating (2) partially with respect to 'x, we get
d
2
-
dX
Z
2
=-ab
2
l
t
sinbx ... (3)
Differentiating (1) partially with respect to It', we get
dZ =abe
bt
sinbx
... (4)
dt
Differentiating (4) partially with respect to It', we get
41
'l _
2
az = ab
2
e
bt
sin bx
... (5)
at
2
From (3) and (5),we get
a
2
z a
2
z
ax
2
=- at
2
a
2
z + a
2
z =0.
~
ax
2
at
2
Q.12. Form the partial differential equation from j (x +y +z, x
2
+l + ) =0 .
Sol. Let u = x + y +z, v = x
2
+l +Z2
Then j(u, v) =0 ... (1)
au az au az av az av az
Now -=1+- -=1+- -=2x+2z- -=2y+2z
, ax ax ' dy dy , ax ax ' ay dy
Differentiating (1) partially with respect to 'x', we get
aj au + aj av =0
au ax avax
aj ( az) aj ( az)
~ - 1+- +- 2x+2z- =0 ~ ..(2)
au ax av ax
Differentiating (1) partially with respect to 'V, we get
aj au + aj av =0
au dy avay
aj ( az \J aj ( az J
~ - 1+- +- 2y+2z- =0 ...(3)
au ay av ay
Eliminating aj and aj from (2) and (3),we get
au av
az azl
1+- 2x+2z-,
ax axl=o
az az
1+- 2y+2z
ay ay
l+p 2x+2zPI=0 az az
~
where P =-and q =:l..
l+q 2y+2zq ax v)'
42
::::} (1 + p)( y +qz) - (1 +q)( x +zp) = 0
::::} (y-z)p+(z-x)q=x-y.
d
2
Z 2d
2
Z 2
Q.13. Solve dx
2
- a dy2 = X .
Sol. The given equation can be written as
d,d
(D2 _ a
2
D/2 ) z = x
2
where D = dx ,D =dy
The auxiliary equation is
m
2
_a
2
=0
::::} m=a
Complementary function is given by
C.F. =.r. (y+ax)+ f2 (y-ax)
Particular integral is given by
1 2
P.I.= 2 2 . ~ x
D -a D
= 1
12 X
2
= -
1 (
1-a
2
_
D/2 J-
1
2 1 12
D' (I-a' ~ , J D' D' x =D' (I+a' ~ , Jx'
=_1(x2+a
2
_
1
(0))=_1x2 =1.(x
3
J= X4
D2 \. D2 D2 D 3 12
The complete solution is
z =C.F.+ P.l. =.r. (y+a x)+ f2 (y-ax)-..!..x
4
.
12
Long Answer Type Questions:
Q.l. Find the general solution of x( Z2 -l) dz +y(X2 - Z2) dz = z(l- X2) .
dx dy
Sol. We have
x Z
(
2
-y
2)dZ
-+y
(2
x -Z
2)dZ (22)
-=Z Y -x
dx dy
The auxiliary equations are
43
dx dy dz
...(1)
x( Z2 _ y2) y(X
2
_ Z2)
x(l-x
2
)
Using multipliers x,Y,z,we get
" <
r
m ~ ~ ~ + ~ + ~ ~ + ~ + ~
x(z2-l) y(;-l) x(l-;) ;(l-l)+l(;_Z2)+;(l-;) o
xdx+ ydy+ zdz = 0
Integrating both sides,we get
x
2
+ l + Z2 =c
1
... (2)
(1) can be rewritten as
dx dy
dz
dx dy dz dx dy dz
-+-+- -+-+
y
x Y z x Y z
x
.z
2 2
(Z2 _y2) (X2 _Z2) (l_X2) ( Z2 -l)+ ( x - Z2 ) +(y2 _x ) o
dx dy dz
=>
-+-+-=0
x y z
Integrating both sides,we get
log x +log y +log z = logc
2
=> xyz =c
2
... (3)
2
xyz =f (x + l +Z2 )
Q.2. Solve px (z - 2y2 )=(z - qy)( z -l-2x
3
)
Sol. The given equation can be rewritten as
px( z - 2l )+qy (z -l-2x
3
) =Z ( z -l-2x
3
)
The auxiliary equation is
dx
dy dz
...{1)
x(z-2l) y( z-l-2x
3
) z ( Z- y2 - 2x
3
)
From last two ratio's of (l),we get
dy dz
-=
y Z
Integrating,we get
44
log y = log z + log c
1
or
Y=C
1
Z
...(2)
From first and third ratio's,we get
dx dz
x(z-2l) z(z-l-2x
3
)
dx dz
=>
2
x (z - 2l) - z ( z - C
l
Z2 - 2x
3
)
=>
Zdx-C
I
2
Z
2
dx-2x
3
dx = xdz-2c
l
2
Xzdz
2dx
d
xdz-zdx 2 2xz z-z +2xdx=O
=>
2
C
j
A
x
Integrating,we get
2
Z 2 Z 2
--C
1
-+x =C
2
...(3)
x x
From (2) and (3),we get
L =f [!:"_C
I
2
+x
2
J.
Z X x
Q.3. Solve (X3 +3xy2)p+(y3 +3x
2
y)q = 2(X2 +l}z.
2
Sol. We have (X3 +3xy2) p+(l +3x
2
y)q =2( x + l)z
The auxiliary equation is
dx _ dy
dz
-:----;:-
...(1)
x
3
+3xy2 - l +3x
2
y
2(X2 + l)z
1 1 2
Using multiplier's -,-,--,we get
x y z
1 1 2 1 1 2
-dx+-dy- dz -dx+-dy--dz
dx dy dz x y
z x z
= = =
x
3
+3xl
y3 +3x2y
2(X2+ l)z x
2
+3l + l +3x
2
-4x
2
-4l 0
1 1 2
=>
-dx+-dy--dz =0
x y z
Integrating ,we get
logx+logy 2logz=logc
j
45
l
.xy =C
::::::>
2 1
... (2)
Z
From last two ratio's of (l),we get
dx _ dy _ dx+dy _ d(x+ y)
...(3)
x
3
+3.xy2 - l +3x
2
Y - x
3
+3.xy2 +y3 +3x
2
Y - (x + yl
dx dy dx-dy d(x-y)
Also, =--::----=----'::----::- ... (4)
x
3
+3.xy2 l+3x
2
y x
3
+3.xy2-l-3x
2
y (X_y)3
From (3) and (4),
d(x+y) d(X-y)
3 = 3
(x+y) (x-y)
Integrating,we get
1 _
1 c
+
2(X+y)2- 2(X_y)2 2
1 1
-2 2 =C
2
... (5)
(x+y) (x-y)
From (2) and (S),we have
.xy (1
7= 1 (x+ y)2
2 2 2
1 J
(x- y)2
az a z az
2x
.
Q.4. Solve -2-3-,:\-+2-
2
=e +
3y
+sm(x+2y).
ax (JXdy ay
Sol. The given equation can be written as
2x
a , a
(D2 -3DD' +2D'2)Z =e +
3y
+sin(x+2y)
where D = ax' D = dy .
The auxiliary equation is
m
2
-3m+2=0
::::::> (m-l)(m-2) =0 or m =1,2
Complementary function is given by
.-...
C.F. =.h (y+ x)+ 12 (y+2x)
Particular integral is given by
46
P.I.= 2 1 , f2 [e
2X
+
3Y
+sin(x+2y)]
D -3DD +2D
1 e2x+3y + 1 sin (x +2y)
D2 - 3DD' + 2D'2 D2 - 3DD' + 2D12
Put D =2, D' =3in first term and D2 =_1
2
, DD' =-2, D'2 =_22in sec ond
1 e2x+3y + 1 sin (x+ 2 )
2
2
-3.2.3+2.3
2
(-12)-3(-2)+2(-22) y .
=!e
2x
+
3Y
-.!.sin(x+ 2y)
4 3
:. The complete solution is
z = C.F.+P.I. =h (y+x)+ 12 (Y+2x)+!e
2X
+
3Y
-.!.sin(x+2Y)
4 3
Q.S. Solve (D3 - 7DD'2 - 6D'3 ) Z =cos ( x +2 y) .
Sol. (D3 -7DD'2 -6D
f3
) z=cos(x+2y)
The auxiliary equation is
m
3
-7m-6=O
=> (m+1)(m+2)(m-3)=O
=> m = -1,-2,3
Complementary function is given by
C.F. =h (y - x) +12 (y - 2x) +13 (y +3x)
Particular integral Is given by
P.l.= 1
D3 _ 7DDI2 _ 6Df3 cos (x +
2
y)
Put D2 =-e,DD'=-2,D'2 =_2
2
1 1
D(-1)-7D(-2
2
)-6D'(_22) cos(x+2y) = 27D+24D,cos(x+2
y
)
1 D
=3 9D2 +8DD,cos(x+2y)
47
=1 D 1
3 9( -1)+8(-2) cos(x+2y) =- 75 cos(x+2y)
Hence the complete solution is
1
Z= h(y-X)+ 12 (y-2x)+ 13 (y+3x)--cos(x+2y).
75
a
3
z a
2
z
Q.6. Solve -3- 2-
2
- =2e
2x
+3x/.
ax ax ay
Sol. The given equation can be written as
a , a
(D3 - 2D2D') z =2e
2x
+3x/
where D= ax,D = ay
The auxiliary equation is
m
3
-2m
2
=0
=> m
2
(m-2)=0 or m=0,0,2
Complementary function is given by
C.F.= h(y)+XI2 (y)+ 13 (y+ 2x)
Particular integral is given by
1 22x 3.
1
2
=> = e + xy
D3 -2D
2
D' D3 -2D
2
D'
1 2
1 22x+3 D')XY
2'-22'(0) e D'(1-2D"
')-1
1
2x
1 D 2
=-e +3- 1-2- xy
4 D3
(
D
1 1 ( (
+...
4 D3 D D
=_e
2x
+3- 1+2-+
D'
2-
D')2 J
xy2
1 1 ( D' ( D')2 J
=4"e
2x
+3 D3 xy2+21)(xy2)+ 21) (xy2)+ ...
1 2
1
D 2 1
2x
'2
=-e +3- xy +2-(2xy)+4-2(XY )+...
J
3
(
4 D D D
48
1
2x
+3-
1(2 2 1())
2x +... =-e
3
xy +2x y+4-
2
4 D D
=_e
2x
+3- 1 ( xy2 +2x2y+4.-
3
J 1 x
4 D
3
3
1 2x 1 (1 2 2 2 X4 J
=-e +3- -x y +-x
3
y+
4 D2 2 3 3
1 2x 1 (1 3 2 1 4 X
5
J
=-e +3- -x y +-x y+
4 Dl6 6 15
6
J 1
2x
+3
( 1 1 x
4 24 30 90
=_e -x
4
l +_x
5
y+_
The complete solution is
6
J ( 1
4
l +_x
5
y+_ Z =f, (y)+XI2 (y)+ 13 (y+2x)+_e
1
2X
+3 -' x
1 x
.
a
4 24 30 90
2
z a
2
z a
2
z
Q.7.
Solve -2+-a--6-
2
=ycosx.
ax axy ay
Sol. We have
a
2
z a
2
z a
2
z
-+---6-=ycosx
ax
2
axay al
a , a
(D2 +DD' -6D'2) z =ycosx
where D=- D =
ax' ay
The auxiliary equation is
m
2
+m-6=0
=> (m+3)(m-2) =0 or m = 2,-3
Complementary function is given by
C.F.= f, (y+2x)+ 12 (y-3x)
Particular integral is given by
=> P.l.= 1
D2 +DD' -6DI? ycosx
49
L_
1
=(D-2D')(D+3D') .ycosx
= D_12D' f(c+3x)cosxdx (Put y=c+3x)
= 1 ,[(c+3x)sinx+3cosx] = 1,[ysinx+3cosx]
D-2D D-2D
= f[(c-2x)sinx+3cosxJdx (Put Y =c - 2x )
= ( c- 2x) ( - cos x) - 2 sin x + 3 sin x = - y cos x + sin x
Hence the complete solution is
z =it (y +2x)+ 12 (y -3x)+sinx- ycosx.
Q.S. Using method of separation of variables/find the solution of the equation
au au . 4 -3x h 0
- +u =- If u = e w en t = .
ax at
au au
Sol. -+u=
..(1)
ax at
Let u =X ( x ) T (t )
...(2)
where Xis a function of x only and Tis a function of t only be a solution of the given equation.
Putting the value of u in (l)/we get
a(XT) a(XT)
ax +XT= at
Tax +XT= xaT
::::::>
ax at
dX +XT= XdT
::::::>
dx dt
::::::> T x' +XT XT'
::::::> x' + 1 = ~ =k (say)
X
Now,
x' +1= k
::::::>
x' =k-1
X X
Integrating both sides,we get
50
- (k-l)x (3)
log X = ( k -1) x +log a
=>
X -ae ..,
T'
Now, -=k
T
Integrating both sides,we get
logT =kt+loga
=>
T =ae
kt
...(4)
From (2),(3) and (4),we get
u =ab e(k-l)x e
kt
=abe(k-l)x+kt
...(5)
Given: u=4e-
3x
when t=O.
Using this in (5),
4e-
3x
= abe(k-l)X
Comparing the constant term and exponent of e on both sides,we have
ab=4 c-1=-3
=> ab=4. c=-2
Hence the solution is
u =4e-
3x
-
2t
du d
2
u. (0) .
Q.9.
Solve at = dx
2
If u x. = sm l[X
d
2
du u
Sol. .. (1)
at= dx
2
let u = X (x)T(t) ...(2)
where X is a function of x only and T is a function of t only be a solution of
the given
Putting the value of u in (l),we get
d(XT) d
2
(XT)
dt - dx
2
X dT = Td
2
X
=>
dt dx
2
d2X
X dT =T
=>
dx
2
dt
51
L_
:::::> XT'=TX"
x" _T' =k (say)
:::::> -X-T
There arises three cases:
Case-l: k =0
" T'
X --=0
:::::> -X-T
:::::> X" =T' =0
Integrating ,we have
X =ax +b , T =c ,where a,b,c are any arbitrary constants
:::::> u=(ax+b)c
It is given that, u =sin ttx at t =0 .
This is not true here.
Therefore this case is not possible.
Case-2: k = a
2
X" T' 2
:::::> -=-=a
X T
X" 2 T' 2
:::::> -=a -=a
X
'T
1 d
2
X _ 2 1 dT 2
2
:::::> ----a --=a or
d x =a
2
X dT =a
2
dt
dx
2
, T
X dx
2
'T dt
Integrating -
dT
=a
2
dt ,we get
T
I
logT =a
2
t+c
1
:::::>
T =be
a
2
d
2
x
2
Now,we shall solve -2- =a X
dx
This can be written as (D2 - a
2
) X =0
It's auxiliary equation is m
2
- a
2
= O.
:::::> m=a,-a
52
:. Solution is
x =aeax +pe
ax
:. u =(ae
ax
+pe
ax
)be
a2t
It is given that, U =sin trx at t =0 .
This is not true here.
Therefore this case is not possible.
Case-3: k =_a
2
=>
X" T' 2
-=-=-a
X T
=>
X" 2
--=-a
X
T' 2
-=-a
'T
=>
1 d
2
X
__..~ _ 2
X dx2 --a
..!... dT =_a
2
'T dt
or
d
2
X --a
2
X
dx
2
-
,
dT =-a
2
dt
T
. dT 2d
Integrating - =-a t ,we get
T
logT =-a
2
t +C
1 =>
T =be-
a
2
t
d
2
x
2
Now,we shall solve -2- =_a X
dx
2
This can be written as (D2 +a ) X =0
2
+a It's auxiliary equation is m
2
= 0 .
=> m =ai,-ai
:. Solution is
X =acos ax+p sin ax
U =(acosax+ psinax)be-
a2
/ ... (3)
It is given that, U =sin trx at t =0 .
=> sinJrx =(acosax+ pSinax)b =abcosax+ pbsinax
Comparing the terms on both sides,we get
ab =0, pb =1, a =Jr
53
Hence the solution is
u = sinJrx e-
a2t

Q.l0. The vibrations of an elastic string is governed by the partial differential equation
a 2 ~ = a 2 ~ .The length ofthe string is Jr and the ends are fixed.The inial velocity is zero
at ax
a
and the initial deflection is u(x,O) = 2( sinx+sin 3x) .Find the deflection u (x,t) of the
vibrating string for t >0 .
2
a
2
u u
Sol. ... (1)
at
2
= ax
2
Let u=X ( x)T (t ) ... (2)
where X is a function of x only and T is a function of t only be a solution of
the given equation.
Putting the value of u in (1" we get
a
2
(XT) _ a
2
(XT)
at
2
- ax
2
a
2
T Ta
2
X
=>
X-=-
at
2
ax
2
d
2
T d
2
X
=>
X-=T
dt
2
dx
2
=>
XT" =T X"
X" _T" =_a
2
(say)
Let
X T
X" T' 2
=>
_=_a
2
-=-a
X 'T
1 d
2
X
2
1 d
2
T 2
2 2
---=-a or
d X = _a2X d T -a
2
T
=>
X dx2 =_a
, T dt
2
dx
2
' dt
2
d
2
x
2
Now,we shall solve -2- =_a X
dx
This can be written as (D2 +a
2
)X =0
2
+a It's auxiliary equation is m
2
O.
54
=> m=ai,-ai
:. Solution is
x = a
l
cos ax + Asin ax
Similarly, T = a
z
cos at + pz sin at
-1
u =(a
l
cos ax + PI sinax)(a
z
cos at + pz sin at) ... (3)
Putting x = 0, u = 0 in (3),we get
o= a
l
( a
z
cos at + pz sin at)
=> at =0
Putting the value of a
l
in (3),we get
u = PI sin ax(a
z
cos at + pz sin at) ...(4)
Putting x =1(, U =0 in (4),we get
o= Asin a1((a
z
cos at + pz sin at)
=>
o= Asin a1( or sinatr=O
=> a1l =n1l or a =n, n is any integer
Now,(4) reduces to
U = PI sin nx( lXz cos nt + pz sin nt) ...(5)
Differentiating (5) w.r.t. tt',we get
du = PI sin nx(-na
z
sin nt + npz cos nt )
dt
du
Put -=Oandt=O.
dt
O=nPIPzsinnx => pz =0
(5) reduces to
u = azA sin nxcos nt ... (6)
Given: u(x,O) = 2(sin x+sin3x)
=> u = azPI sin nx= 2(sin x + sin 3x)
=> azA sin nx = 4sin 2xcos x
55
'
~ aill =4cosx and n=2
substituting these values in (6),we get the required solution as
u = 4cosxsin 2xcos2t.
Q.ll. A rod of length I with insulated sides is initially at a uniform temperature u.lts ends are
-- ..
suddenly cooled to 0C and are kept at that temperature. Prove that the temperature
2 2 2
00 c 1r n t
. ( ).. b ( ) ~ b . ntrx --/2
functIon U x,t IS given y U x,t = ~ n SlD--.e
n=1 I
where b is determined from the equation U0 =! b sin ntrx .
n n
n=1 I
Sol. let the equation for the conduction of heat be
au 2 a
2
u
-=c- ... (1)
ax
2
at
let U =X ( x) T (t ) ... (2)
where X is a function of x only and T is a function of t only be a solution of the above equation.
Putting the value of uin (l),we get
a(XT) 2 a
2
(XT)
----'---'- =C -----'-....:....
at ax
2
XaT =Tc
2
a
2
x
~
at ax
2
1 d
2
T 1 d
2
X
~
c
2
T dt
2
=X dx
2
1 d
2
T 1 d
2
X 2
let --=---=-a
c
2
T dt
2
X dx
2
1 d
2
X 1 dT 2 d
2
X dT 2 2T 2
--=-a or 2X -=-a c
X dx2 =_a
dx2 =-a , dt
'c
2
T dt
Solving these equations ,we get
b _a
2
c
2
t
X =acosax+psinax ,T
-
- e
~
U = (acosax+ psinax)be-a2c2t
... (3)
Putting x =O,u =0 in (3),we get
_a2c2t
O=ab e ~ a=O
56
(3) reduces to
. 2 2
U= /ibsinaxe-
a
c t
... (4)
Putting x = l,u = 0 in (4),we have
o=/ibsinal.e-a2c2t
=::}
sin al = Oor al = nli ,n is an integer
nli
=::}
a=
1
Hence (6) reduces to
2 2
c
2
n;
2
n
2
t c n; n
2
t
--2- --2- nlix nlix
U= Rbe I s1O--=b e I S1O-
jJ 1 n 1
This equation satisfies the given conditions for all integral values of n.
Hence taking n =1,2,3, ..... I the most general solution nis
U(x t) - ~ b . nlix
, - ... n S1O--.e
n=l I
By initial conditions u =U0 when t =0
Substituting this in above solution,we get
..
U - "b . nlix
0- ... n
S1O
-
n=l I
57
L
Unit - 3 : Matrices and Determinant
Multiple Choice Type Questions:
Q.l. The rank of the matrix (1 0 Jis
o -1
(a) 1 (b) 2 (c) 0 (d) 3
Q.2.
- [1 2-1]
The characteristic roots of the matrix 0 4 3 are
o 0 2
(a) 1,-1,0 (b) 1,2,4 (c) 1,2,2 (d) 0,1,2
Q.3.
Q.4.
Q.5.
If A is of order 5 x 3 and Bis of order 3X 4 then the order of (AB Yis
(a) 4x5 (b) 4x4 (c) 5x5 (d)
If A and B are two matrices of order 3 and AB =O,then
(a) A=O
(b) B 0
(c) detA = OordetB = 0 but A and B need not be zero matrices
(d) det A =det B =0 if A and Bare not zero matrices
The rank of the matrix 2 4 6 is equal to
[
1 2 3]
369
5x4
Q.6.
(a) 3 (b) 2 (c) 1 (d) o
Which of the following is not true ,in general
(a) A+B B+A (b)
A+(B+C)=(A+B)+C
(c) A(BC)=(AB)C (d) AB=BA
Q.7. Which of the following is true
(a)
(ABt =B-1A-
1
(b)
AadjA =JAIl
(c) ladjA! =!A!n-l
(d) All of the above
58
Q.B. If AAt =I ,then the matrix A is called
(a) Idempotent (b) Orthogonal
(c) Symmetric (d) Skew symmetric
Q.9. If
(a+b

J,the values of a and b are respectively
5
(a) a=2,b=4 (b) a=4,b=2
(c) a =2,b= 40ra =4,b=2 (d) None of the above
x-2 3
Q.l0. If I =O,the value of x is
x x+2
(a) -lor4 (b) -20r4
(c) -lor2 (d) -20r2
Q.ll. If are eigenvalues of a matrix A of order two, the eigenvalues of transpose of A are
(a)

(b)

(c)

(d) can't be determined
[1 4
Q.12. For what value of Ii.. ,the matrix Ii.. 3 is singular.
6 4
(a) 4 (b) 7 (c) -7 (d) 6
Q.13. The product of any mxn matrix A = (aij) by any scalar c is written as cA =
(a)
(c+aiJ
(b)
(c-aij )
(c)
(ca
ij
)
(d) None of the above
Q.14. A matrix A = ( aij ) is symmetric matrix if
"
(<if
aij =a
Ji
(b)
aij =-aji
(c) aij :: 0 'if i, j (d)
aij =2a
jj
Q.1S. If A,B and C are non singular nxn matrices ,then (ABet
(a) A-'B-'e-' (b) e-'A-'B-'
(c) A-'e-'B-' (d) B-'A-'e-'
59
Q.16.
If A=
(2
2 3
1)
,B=
(-3
2 )'then AS is
(a)
(b)

(c) (d)


0 5 9 7
-5 0 5 9
Q.17.
The value of the determinantl-9 -5
0 3
-7 -9 -3 0
-3
-7 -9 -5 0
(a)
an odd integer
(b)
an even integer
(c) zero
(d) 237
[0 1 OJ
Q.18. The inverse of the matrix 1 0 0 is
001
[0 -1
(a)
o 0
(b)
[0
-1 0
1 OJ
0
1 0
1 0 -1
(c) [1001 OJ
(d)
[0 1 OJ
100
010
001
Q.19.
If A isthe matrix G }then A' =
(a) (11 30J
(b)
15 41
G:7)
(c)
(d) (-1 0)
(! 1
8
1)
o -1
-
.
Q.20
For the matrix A = ( 3 1 ) Theorem gives that A
2
- 5A is
-1 2
(a) 0 (b) 71 (c) -71 (d)
7
60
Q.Z1. The system of linear equations AX =B exists if
(a) Rank A =Rank [A: B] (b) Rank A:;t: Rank [A:B]
(e) Rank A < Rank [ A:B] (d) Rank A> Rank [A:B]
(
1-IJ
Q.ZZ. The inverse of the matrix 1 1 is
(a)
(
~ - ~ J (b)
(
~ ~ J (e)
[
~ - ~ J (d) [ ~ ~ J
- ~ ~ - ~ ~
~ ~ ~ ~
Q.Z3. The system x+ay =0, y+az =0, z+ax =has infinitely many solutions when
(a) a=l (b) a=-l
(e) a=O (d) real values of a
Q.Z4. If a matrix A satisfies a relation A
3
+ A I =a ,then A-I is equal to
(a) A+I (b) A I
(e) A (d) I-A
Q.Z5. A matrix A is said to be idempotent if
(a)
A2 =1 (b)
A2 A
(e)
AAt =1 (d) A=A
t
Q.Z6. If A is an involuntary matrix, then determinant of A is
(a) 1 (b) -1
(e) lor-l (d) None of the above
2.) The inverse of an orthogonal matrix A is equal to
At
(a) A (b)
_At
(e) (d) -A
Q.Z% The diagonal entries of a skew symmetric matrix are
(a) real (b) pure imaginary
(e) zero (d) an integer
Q., The value ot[x Y l [ ~ :t] is
(a) ax
2
+hxy+bi (b)
ax
2
_ hxy +by2
2
(e) ax
2
+2hxy +bi (d)
ax - 2hxy +bi
61
Key
1. (b) 2. (b) 3. (a) (c) 5. (c) 6. (d)
'
4

7. (d) 8. (b) 9. (c) 10. (a) 11. (a) 12. (c)
13. (c) 14. (a) 15. (b) 16. (c) 17. (c) 18. (d)
19. (a) 20. (c) 21. (a) 22. (b) 23. (b) 24. (a)
25. (b) 26. (c) 27. (b) 28. (c) 29. (c)
True /False Type Questions :
1. The number of rows and columns of a square matrix are equal.
2. Every square matrix satisfies its characteristic equation.
3. Matrix multiplication is commutative.
4. The diagonal entries of a symmetric matrix are all zero.
,
5. The inverse of a matrix A exists if and only if A is a singular matrix.
6. A matrix of order Ix n is called a row matrix.
7. If A is of order 3x4 and B is of order 4x5 then AB and BA both exists.
8. Elementary row transformation does not alter the rank of a matrix.
9. Every square matrix can be uniquely expressed as a sum of a symmetric and a skew
symmetric matrix.
10. For any two matrices A and B, AB =BA.
11. For any two matrices A and B,(A + B)2 = A2 + 2AB+ B2 ..
12. The rank of an identity matrix of order n is n.
13. A matrix is a rectangular array of numbers enclosed in brackets.
14. A skew symmetric determinant of odd order is zero.
15. A skew symmetric determinant of even order is a perfect square.
16. The rank of a non singular square matrix of order n is n.
17. An identity matrix is involuntary but not idempotent.
18. Every matrix can be reduced to a triangular matrix by elementary row transformations.
Key
1. T 2. T 3. F 4. F 5. F 6. T
7. F 8. T 9. T 10. F 11. F 12. T
13. T 14. T 15. T 16. T 17. F 18. T
62
Short Answ.er Type Questions:
Q.l. Given 3(X y) =( x
6)
+
(4 x+ y)
.Find the values of x,y,z and w.
Z w -1 2w z+w 3
3(X y) (x 6) (4 x+ y)
Sol.
Z w = -1 2w + z+w 3
3X 3Y) (X+4 6+x+ Y)
=:>
(
3z 3w = -l+z+w 2w+3
Equating the corresponding entries of the above matrices,we get
3x= x+4 =:> 2x=4 or x=2 ...(1)
3y=x+y+6 =:> 2y=x+6 ...{2)
3z =-1+ z+w =:> 2z=w-1 ...(3)
3w=2w+3 =:> w=3 ...{4)
Substituting the value of x from (1) in {2),we get
2y=2+6 or y=4
Substituting the value of w from (4) in (3),we get
2z = 3-1 or z=l
x =2, y =4, z =1, w=3 .
[1 -2 3 J r02J
Q.2. If A= 2 3 -1 and B= 0 1 2 .Verifythat (ABY =BtAt.
-3 1 2 1 2 0
[1 -2 3r02J
Sol. AB= 2 3 -1 0 1 2
-3 1 2 1 2 0
[1+0+3 0-2+6 2-4+0 H4 4 -2J
= 2+0-1 0+3-2 4+6+0 = ,I 1 10
-3+0+2 0+1+4 -6+2+0 -1 5 -4
[4 1 -IJ
=:> (AB)' = 4 1 5
...{1)
-2 10 -4
63
l_
(I 0If 2-3J
Now, Bt At = 0 1 2 -2 3 1
2 2 0 3 -1 2
(1+0+3 2+0-1 -3+0+2J
:.::;;
= 0-2+6 0+3-2 0+1+4
2-4+0 4+6+0 -6+2+0
(4 I -IJ
= 4 1 5
... (2)
-2 10 -4
From (1) and {2),we get
(ABr = BtAt
3 4 1 1
2
4 . 3
6
0.3. Find the rank of the matrix
A=I
-1 -2 6 4
1 -1 2 -3
3 4 1 1
2 4 3 6
Sol.
A=I
-1 -2 6 4
1 -1 2 -3
Applying the row operation, R4 ~ Rl'we have
1 -1 2 -3
2 4 3 6
-1 -2 6 4
3 4 1 1
Applying the row operation, ~ ~ ~ - 2 ~ , R3 ~ ~ +R ,R
4
~ R4 - 3R ,
J j
1 -1
0 6
o -3
0 7
2
-1
8
-5
-3
12
1
10
64
Applying the row operation, R3 -t ~ +!R.z, R4 -t R4 _2R.z ,we have
2 6
1 -1 2 -3
0 6 -1 12
~
0 0 15/2 7
0 0 -23/6 --4
Applying the row operation, R4 -t R4 + 23 R
3
, we have
45
1 -1 2 -3
0 6 -1 12
0 0 15/2 7
0 0 0 -19/45
Rank=Numbers of non zero rows = 4
1 1 1 1
3 -2 1
Q.4. Find the rank of the matrix A = I
1
2 0 -3 2
3 3 0 3
1 1 1 1
A =1 1 3 -2 1
Sol.
2 0 -3 2
3 3 0 3
Apply the row operation, R2 -t R2 - R} , R3 -t R3 - 2RI ' R4 -t R4 - 3R
1
1 1 1 1
0 2 -3 0
o -2 -5 0
,
0 0 -3 0
Apply the row operation, R3 -t ~ +R.z
1 1 1 1
o 2 -3 0
o 0 -8 0
0.0 -3 0
"
65
Apply the row operation, R4 R4 - R3
8
1 1 1 1
0 2 -3 0
0 0 -8 0
0 0 0 0
Rank=Numbers of non zero rows = 3.
Q.S. Find the value of x such that [1 x 1][ .. = O.
15 3 2Jx
Sol. [1 x 1][ =0
15 3 2 x
1 3 2
[1 x 1] 2 5 (j
[ [
15 3 2
[1+2x+15' 3+5x+3
-
1
21=0
x

(2x+16).I+(5x+6).2+(x+4).x=0
2x+16+lOx+12+x2+4x=0
x
2
+16x+28=0
x
2
+14x+2x+28=0
x(x+14)+2(x+14)=0
(x+2)(x+14)=0
x=-2,-14.
66
4-2 1J
2t
7.L
Q.6. Find the inverse of the matrix A = 7 3 3.
z;J
[
201

j2 J]

4-2 1J
Sol. A= 7 3 3
[
201
..,. ... .,.,. '- 1)(2 t)
4 -2 C
IAI=lt f 3=4(3-0)+2,(7-6)+1(0-6)=12+2-6=8
201
If M
jj
denotes the minor of a
jj
,then the cofactor of a
jj
is
C;J =t-It
J

2
II _1-
2
1 =_9
-I
"3
0
31
1
=3,c21
=_1-
0 1
=2,c
31
- 3 3
1
Cll
C12
7 31 14 -21 14 -2
c13 =12 0 = -6 ,C23 = - 2 0 =-4, C33 = 7 3 1 =26
Now,
adjA = ( c
ij
r
[3 2-9J
adjA= -1 2 -5
-6 -4 26
Since,
A-I = adjA
[ 3 2
A-I = -1 2 -5.
-6 -4 26
67
L
2 1 3]
Q.7. If A= 2 0 1 ,verify that A(adjA)=IAII3 = (adjA)A.
(
-456
Sol. We have
2 1 3]
A= 2 0 1
(
-4 5 6
2 1 3
~ IAI=12 0 11=2(-5)-1(12+4)+-3(10)=-10-16+30=4
-4 5 6
If M
jj
denotes the minor of aij ,then the cofactor of aij is
j
Cij =(-lt Mij
=-5, = 9, C
31
=1 C
1l
C
21
=-16, C
22
=24, C
12
C
32
=4
=10, C
23
=-14, C
33
=-2 c
l3
Now,
adjA=(Cijr
(-5 9 1 ]
adjA= 16 24 4
10 -14 -2
(2 11-
5
9 I) (4 0OJ [1 0]
A (adjA) = 2 0 1 16 24 4 = 0 4 0 =4 0 1 0
-4 5 6 10 -14 -2 0 0 4" 0 0 1
~ A (adjA) =413 =IAII3
Similarly ,it can be proved that
(adjA)A =IAII3'
68
[2 0 1J
Q.8.
Let f(x)=x
2
-5x+6.Find f(A)if A= 2 1 3.
1 -1 0
[2 0 1J
Sol.
A= 2 1 3
1 -1 0
f(A)=A
2
-5A+61
~
f(A)=
[2
2
0
1 3
1J
-5
[2
2
0
1 3
1J
+6
[1
0
0
1 0
OJ
1 -1 0 1 -1 0 0 0 1
~ f(A)=
(2
2
0
1 3
If
2
0
1 3
1J
-5
(2
2
0
1 3
1J
+6
[1
0
0
1 0
OJ
1 -1 0 1 -1 0 1 -1 0 0 0 1
[4+0+
1
0+0-1 2+0+0J [10 0 5 J [
6
0 OJ
~ f(A)= 4+2+3 0+1-3 2+3+0 - 10 5 15 + 0 6 0
2-2+0 0-1+0 1-3+0 5 -5 0 0 0 6
(5 -1 2J [10 0 5J (6 0 OJ
~ f(A)= 9 -2 5 - 10 5 15 + 0 6 0
o -1 -2 5 -5 0 0 0 6
[5-10+6 -1+0+0 2-5+0 J
~ f(A)= 9-10+0 -2-5+6 5-15+0
0-5+0 -1+5+0 -2+0+6
[I' -3J -1
~ f(A)= -1 -1 -10 .
-5 4 4
69
0.9. Prove that the product of two matrices
2 2
COS 0 cos Osin OJ d( COS t/J
cost/Jsint/JJ is zero when 0 and t/J
2 A.
(
cos0 sin 0 sin
2
0 an cost/Jsint/J
sm ."
differ by an odd multiple of f( .
2
Sol. Consider,
COS
2
0 cos 0sin OJ [ cos
2
t/J
[
cos 0sin 0 sin
2
0 cost/J sin t/J
= (cos
2
Ocos
2
t/J+ cos osin Ocost/Jsint/J
cosOsinOcos
2
t/J+sin
2
Ocost/Jsint/J
= [cos0cost/J (cos 0cost/J + sin 0sin t/J)
cost/J sin 0(cos 0cos t/J + sin 0sin t/J)
"':': ...,.
cos t/J sin t/J]
..
sin
2
t/J
cos
2
Ocost/Jsint/J+cosOsinOsin
2
t/J]
cosOsinOcost/Jsint/J+sin
2
Osin
2
t/J
cos 0sin t/J (cos0cost/J + sin 0sin t/J)J
sin 0sin t/J (cos0cos t/J + sin 0sin t/J)
= [cos0cos t/J cos (0- t/J) cos 0sin t/J cos (
0
- t/J)J
...(1)
cost/JsinOcos(O-t/J) sin Osint/Jcos(O-t/J)
It is given that 0- t/J = ( 2n +1 ) f( .
2
=>
cos (O-t/J) = 0 . ...(2)
Using (2) in (l),we get
2 2
COS 0 cos 0sin OJ [ cos t/J cos t/J sin t/J]
[
cosOsinO sin
2
0 cost/Jsint/J sin
2
t/J =
cosocost/J(O) cososint/J(O)J=(O OJ = O.
(
cost/JsinO(O) sinOsint/J(O) 0 0
Hence proved.
-tanaJ (cos2a -sin2a]( )
0.10.
Proveth.U
,then I +A = I - A .
o sin 2a cos 2a
Sol. A-
0

(
tana
L.H.S.
70
I+A=(1 0)+( 0
o 1 tana
-tanaJ=( 1
0 tana
-tanaJ
1
R.H.S.
(
~ s 2a - sin 2aJ{I _ A) = ( c ~ s 2a - sin 2a)(. 1
sm2a cos2a sm2a cos2a -tana
(
COS 2a+tan a sin 2a tanacos2a- sin 2aJ
= sin 2a- tan a cos 2a tan a sin 2a+cos 2a
tan aJ
1
2 . 2 (sina)2' sina)( ).
cos a-sm a+ -- smacosa -- 2cos
2
a-I -2smacosa
(
cosa cosa
=
2smacosa-
.
-_.-
)
sina)2 . 2 2
(sina)(
2cos
2
a-I
-- smacosa+cos a-sm a
(
cos a cos a
=( cos
2
a-sin
2
a+2sin2 a 2sinacosa-tana-2sinacosa1
2sin acosa-2sin acosa+tana 2sin2 a+cos
2
a-sin
2
a )
ooS2 a+sin2 a -tana J
=
(
tan a cos
2
a+sin
2
a
=( 1 -tana)
tan a 1
:. L.H.S=R.HS.
Hence proved.
2 2J
Q.ll. Verify that the matrix A = 1:.( ~ 1 -2 is orthogonal.
3 -2
2 -1
Sol. A matrix A is called orthogonal if AAt =At A =I .
Consider,
(1 2 2 J[ (1 2 2 J J
AAt = ~ 2 1 -2 ~ 2 1 -2
-2 2 -1 -2 2 -1
1
(1 2 2 r 2 -2J
=- 2 1 -2 2 1 2
9 -2 2 -1 2 -2 1
71
l.
(1+4+4 2+2-4 -2+4-2) (9 00) (1 00)
=i 2+2-4 4+1+4 -4+2+2=i 0 9 0 = 0 1 0
-2+4-2 -4+2+2 4+4+1 0 0 9 0 0 1
AAt =1
=>
Now, consider
[ (1 2 2 )]f 2 2)
AtA = -} -} 2 1 -2 2 1 -2
-2 2 -1 -2 2 -1
1
(1 2 -2f 2 2)
=- 2 1 2 2 1 -2
9 2 -2 1 -2 2 -1
[+4+4 2+2-4 2-4+2) (9 00) (1 00)
=i 2+2-4 4+1+4 4-2-2 =i 0 9 0 = 0 1 0
2-4+2 4-2-2 4+4+1 0 0 9 0 0 1
AtA=I
..
AAt =AtA=I
=>
Hence A is an orthogonal matrix.
Q.12. Find the characteristic equation and characteristic roots of the matrix
1 2 -2)
A= 1 1 1 .
(
1 3 -1
Sol. The characteristic equation of a matrix a is given by IA- .MI = 0 .
(1 2-2)
A= 1 1 1
1 3 -1
(1 2-2) (1 00) (H 2 -2)
-
A- Al = 1 1 1 - A 0 1 0 = 1 1- A 1
. ,
1 3 -1 0 0 1 1 3 -I-A
72
I-A 2 -2
=> IA-AII=11 I-A
1 \=_..1,3+..1,2+4..1,-4
1 3 -I-A
The characteristic equation is given by
_..1,3 +..1,2 +4..1,-4=0
.;;:,
=>
..1,3-..1,2-4..1,+4=0
=> (..1,-1)(..1,2-4)=0
=> (..1,-1)(..1,+2)(..1,-2)=0
=> ..1,=1,2,-2
The characteristic roots are 1, 2 and 2 .
I:,.}
Q.13.
[
COSX
If f (x) = sin x
o
-sinx
cos x
0
OJ [COSY
0 ,& fy = sin y
1 0
-siny
cos y
0
OJ
0 then show that
1
f(x)f(y) = f(x+ y) .Hence show that f(xt = f( -x).
Sol. We have
COSX -sinx 0J[COS y -sin y OJ
[
f(x)f(y)= sinx
o
cos x
0
0
1
siny
0
cosy
0
0
1
COS x cos y-sinxsin y -cosxsin y-sinxcos y OJ
[
= sinxcosy+cosxsiny -sinxsiny+cosxcosy 0
001
COS(x+
y
) -sin(x+y) OJ
(
= sin(x+y) cos(x+y) 0
001
f(x)f(y)=f(x+y) ..(1)
Putting y = -x in (1)
f(x)f (-x) = f(x-x) = f(O) ...(2)
73
(=0 -smO 0) (I 0OJ
Now, 1(0)= sinO cosO 0 = 0 L 0 =1
3
, ...(3)
o 0 1 0 0 1
From (2) and (3),
l(x)/(-x)=13
Hence 1(-x) is the inverse of 1(x) .i.e., 1(xt = 1(-x).
Long Answer Type Questions:
2 0 -lJ
Q.l. Find the inverse of the matrix 5 lOusing elementary row operations only.
(
o 1 3
Sol. Consider the identity,
A=IA
2 0 -lJ [1 0 OJ
510=010A
(
o 1 3 0 0 1
Applying the row operation Rz ~ Rz - -
5
RI on the matrix on the right as well as
2
on the pre-factor on the right,we have
2 0 0
-Ilr 1
0
0 1 1 OIA
~ = -0;
0 1 0 1
Applying the row operation R3 ~ ~ - Rz on the matrix on the right as well as
on the pre-factor on the right, we have
2 0
0 1
o 0
-1
5
2
5
2
:1
1 0 0
5
=
1 OIA
2
5
-1 1
2
74
Applying the row operation Rl --? Rl +2R3 , ~ --? ~ - 5~ on the matrix on the right as
well as on the pre-factor on the-right,we have
2 0 0
0 1 0
' < ~ :
1
I
0 0
2
=
')
6 -2 2
-15 6
-5 111
5
-1 1
2
1
Applying the row operation ~ --? - R
1
, ~ --? 2 ~ on the matrix on the right as well as
2
on the pre-factor on the right,we have
1 0 OJ [3 -1 1 J
o 1 0 = -15 6 -5 A
[
o 0 1 5 -2 2
From above identity,the inverse of A is
-1 1 J
6 -5.
A-'=[+
-2 2
2 4 3 2
3 6 5 2
Q.2. Find the inverse of the matrix A = I
.
Sol. Consider the identity,
2 4 3 2
3 6 5 2
2 5 2 -3
4 5 14 14
=
2
5 2 -3
4 5 14 14
"
A=IA
1 0 o 0
0 1
o 0IA
o 0 1 0
0 o 0 1
1
Applying the row operation Rl --? - R\ on the matrix on the right as well as on the pre
2
factor on the right, we have
75
1 2 ~ 1
2
3 6 5 2 1=
2 5 2 -3
4 5 14 14
1 000
o 1 0 0IA
o 0 1 0
000 1
Applying the row operation 1S -+ 1S - 3R
t
, ~ ----t ~ - 2R
t
, R4 -+ R4 - 4R, on the matrix
on the right as well as on the pre-factor on the right,we have
1 2
3
2
1
o 0
1
-
2
-11=
o 1 -1 -5
o -3 8 10
~
2
0 0 0
3
2
-1
1
0
0
1
OIA
0
-2 0 0 1
Applying the row operation 1S ~ ~ on the matrix on the right as well as on the pre
factor on the right,we have
1 2
3
2
1
o 1 -1 -5
o 0
1
2
-1
o -3 8 10
~ 0 0 0
2
-1 0 1 O'A
=
~ 100
2
-2 0 0 1
Applying the row operation R, ----t R, - 21S , ~ ----t 2 ~ , R4 ----t R4 +31S on the matrix on
the right as well as on the pre-factor on the right,we have
o -2 0
1 0 7 11 I
5
2
o 1 -51 = -1 -1 0
1
IA
o 0 1 -2
-3 2 o o
o 0 5 -5
-5 0 3 1
7
Applying the row operation R\ ----t ~ -- - ~ ,1S ----t 1S +~ ,R
4
----t R4 - 5~ on the matrix
2
on the right as well as on the pre-factor on the right,we have
76
13 -7 -2 0 1 0 0 18
0 1 o -7 -4 2 1
=
OrA
-3 2 0 0 o 0 1 -2
o 0 0 5 10 -10 3 1
1
Applying the row operation R4 -7 - R4 on the matrix on the right as well as on the pre
5
factor on the right, we have
13 -7 -2 0
1 0 0 18
-4 2 1 0
0 1 0 -7
= -3 2 0
oIA
0 0 1 -2
3 1
o 0 0 1 2 -2
5 5
Applying the row operation RI -7 R
J
-18R
4
, R.z -7 R.z +7R
4
, ~ -7~ +2R4 on the
matrix on the right as well as on the pre-factor on the right,we have
1 0 0 0
0 1 0 0
0 0 1 0
0 0 0 1
64 18
-23 29
5 5
26 7
10 -12
=
5 5
IA
6 2
1 -2

5 5
3 1
2 -2
5 5
From above identity, the inverse of A is
-23 29
64 18
5 5
10 -12
26 7
A-I =1
5 5
1 -2
6 2
5 5
2 -2
3 1
-
5 5
77
l
(1 2-IJ
0.3. Verify (A+B)2 =A
2
+AB+BA+B2forthe matrices A= 2 0 3
o 1 2
(3 -1 1)
and B= 0 0 2.
4 -3 2
(1 2-IJ (3 -1 IJ (4 1 OJ
Sol. We have A+ B= 2 0 3 + 0 0 2 = 2 0 5
o 1 2 4 -3 2 4 -2 4
(4 1 0J(4 1 0)
=>
(A+B)2=(A+B)(A+B)= 2 0 5 2 0 5
4 -2 4 4 -2 4
(18 4 5 J
=>
(A+B)2 = 28 -8 20
28 -4 6
Now,
(1 2-IJ(1 2-1) (5 13)
A2 = 2 0 3 2 0 3 = 2 7 4
012012227
(1 2-r -1 1) (-1 2 3)
AB = 2 0 3 0 0 2 = 18 -11 8
o 1 2 4 -3 2 8 -6 6
(3 -1 T2-1) (1 7-4)
BA = 0 0 2 2 0 3 = 0 2 4
4 -3 2 0 1 2 -2 10 -9
(3 -1 If -1 IJ (13 ~ 3J
B2 = 0 0 2 0 0 2 = 8 -6 4
.
.,
,
4 -3 2 4 -3 2 20 -10 2
78
5
0 2 4
7 -4)
+
[13

2 2 7 8 -6 6 -2 10 -9 20 -10 2
A'+AB+BA+B' =
[
2 7 4
13J
+
C
18 -11
2 3
8
J
+
r
8
3
4
J
[18 4 5 )
= 28 -8 20
28 6 -4
Hence (A+Bl =A2 +AB+BA+B2.
-2 -4)
Q.4.
Express the matrix A= [
3 4 as the sum of a symmetric and a skew
-2 -3
symmetric matrix.

Sol. We have
2 -1 1 )
At = -2 3 -2 .
[
-4 4 -3
Now A =P +Q I where P is a symmetric matrix and Q, a skew symmetric matrix are
given as
P='!'(A+A
t
) and Q=.!.(A-A
1
)
2 2
l[ 2 -2 -4) [2 -1 1J] [4 -3 -3)
-2 3 4 + -2 3 -2 -3 6 2
-4 -2 -3 -4 4 -3 -3 2 -6
[[ 2 -2 -4) [2 -1 1)] [0 -1 -5)
-2 3 4 - -2 3 -2 1 0 6.
-4 -2 -3 -4 4 -3 5 -6 0
Verification:
4-3 -3)
pI -3 6 2 =P
-
[
-3 2 -6
P is a symmetric matrix.
79
0-1 -5]
Qt =.!. (A - AI ) =.!. 1 0 6
2 2 5
(
-6 0
Q is a skew symmetric matrix.
~
Q.S. If A=( 1 2],expressA
6
-4A
5
+8A
4
-12A
3
+14A
2
as a linear polynomial in A.
-1 3
Sol. The characteristic equation of A is
1-1 2
3-1
1
=0,
-1
12 -41+5=0.
By division algorithm, we have
1
6
-41
5
+81
4
-121
3
+1412 =(12 -41+5)(14 +31
2
-1)-41+5 ...(1)
From (l),we have
A
6
-4A
5
+8A
4
-12A
3
+14A2 =(A2 -4A+5)(A4 +3A
2
-1)-4A+5
Since by Cayley-Hamilton theorem,
A2 -4A+51 =0
Therefore, the right hand side of (2)equals -4A+51 .
Hence A
6
-4A
5
+8A
4
-12A
3
+14A2 =-4A+51
Q.6. Using Cramer's rule, solve the system of equations
x+y+z=1
1.x+2y+3z=k
12 x+ 22 y+3
2
z =k
2
Sol. We have
1 1 1
D=ll 2 31=(1-2)(2-3)(3-1)=2
3
2
12 22
Since D ':f:. 0 ,by Cramers rule solution is given by
D D D
x=;,y=;,z=;
80
(2-k)(3-k) 2(1-k)(k-3) (1-k)(2-k)
x= ,y= ,z=
2 2 . 2
(2-k)(3-k) (1-k)(2-k)
Or x= ,y=(1-k)(k-3),z=
Q.7. Solve the system of equations
x+ y+z =7,x+2y+3z =16,x+3y+4z =22.
Sol. The given equations may be written as AX = B,where
A=(: i
Then
[:
Applying elementary row operations Rz Rz - R), R3 - R) ,we have
l
81
Applying elementary row operations ~ ~ ~ - 2 ~ ,we have
1 IIJ._(XJ ( 7J
o 1 2 y = 9
(
o 0 -1_ z -3
Thus we have reduced the co-efficient matrix A to triangular form and the system of
equations get reduced to
x+ y+z = 7,y+2z =9,-z =-3
=> z =3,y =9-2z =3and x= 7- y =1.
Q.8. Using matrices ,solve the system of equations
x+3y+2z =0,2x- y+3z =0,3x-5y+4z =0,x+17y+4z =0.
Sol. The given equations can be written as
1 3
3 -5
2 -1
~ m=
1 17
0
0
0
0
Applying elementary row operations ~ ~ ~ - 2R
1
, R3 ~ R3 - 3, R4 ~ R4 - R
1
,
we have
1 3
0 -7 -3
2 0
0
n
o -14
-: ~ = ~
0 14
Applying elementary row operations R4 ~ R4 +~ , we have
1 3 2
0 -7 -3
0 -14 -2
0 0 0
m=
0
0
0
0
/
Applying elementa ry row operations R3 ~ ~ - 2 ~ , we have
82
I
1 3 2
0 -7 -3
0 0 4
0 0 0
[ ~ } =
0
0
0
0
/
:. The system of equations reduces to
x+3y+2z =0,-7y-3z =0,4z =0
=> x=O,y =O,z=O
Hence x = 0, y = 0, z= 0 is the only solution.
Q.9. Find for what values of k the set of equations
2x-3y+6z-5t =3, y-4z+t = 1,4x-5y+8z-9t = k
Has (1)no solution (2) infinite number of solutions.
Sol. The given system of equations can be written as AX = B
2-3 6-5] [3]
o 1 -4 1 X= 1
[
4 -5 8 -9 k
The augmented matrix is given by
2 -3 6 -5
. 3]
[A:B]= 0 1 -4 1
. 1
[
4 -5 8 -9 k
Applying elementary row operations R3 ~ R3 - 2R
1
,we have
2 -3 6 -5
[A:B] 0 1 -4 1
[
o 1 -4 1 k ~ J
Applying elementary row operations R3 ~ ~ - ~ ,we have
2 -3 6 -5 3 ]
[A:B] 0 1 -4 1 . 1
[
o 0 0 0 : k-7
83
L
(i) There is no solution if rankA '# rank [A: B]
If k-7 '#0 or k '# 7 ,then rankA =2 , rank [A:B] =3
=> The given system has no solution if k '# 7
(ii)There are infinite number of solutions if rankA = rank [A+B] .
We can easily see that rankA = 2 and rank [A: B] = 2 will be equal to 2 if
k-7=0 or k =7.
For k = 7 ,the given system of equations reduces to
2-3 6 -5] [3] x
3 1 -4 1 Y = 1
[
o 0 0 0 Z 0
t
This can also be written as
2x-3y+6z-5t =3,y-4z+t =1 ... (1)
As r =rankA =rank [A: B] =2 and the no. of equations, n =4,:. to obtain the
solution n - r = 2 variables must be taken as constants.
So ,let z=kl ' t =k2
...(2)
From (1) and (2),we get
x =3+3k
l
+2k2 ' Y =1+4kl - k2 , Z =kl ' t =k2 .
This is the required solution.
Q.10. Determine ,without actually attempting to solve, whether the following system
is consistent or not.
x-3y+ z =-1,2x+ y-4z =-1,6x-7y+8z =7.
Sol. The given equations may be written as .AX = B .
1-3 1] (-1]
2 1 -4 X = -1
(
6 -7 8 7
84
The augmented matrix is given by
1 -3 1 :
-1]
[A:B]= 2 1 -4 -1
[
6 7 8 7
Applying elementary row operations ~ ~ ~ - 2Rl ~ ~ ~ - 6Rl'we have
1 -3 1 -1]
[A:B] 0 7 -6 1
[
o 11 2 : 13
Applyi ng elementary row operations R3 ~ R3 -.!..!. ~ I we have
7
1 -3 1 -1
[A:B] 10 7 -6 1
80 80
o 0
7 7
Applying elementary row operations ~ ~ ~ R3/we have
80
[1 -3 1 : -1]
[A:B] 0 7 -6 : 1
o 0 1 : 1
1 -3 1 1 -3 -1
=7 :;I!:Oand We have 10 7 -6 0 7
11=7:;1!:O.
0 0 1 0 0 1
rankA = rank [A:B] = 3
Hence the given system of equations is consistent.
Q.ll. Show that only value of Afor which the following system of linear equations has non
zero solution is 6 and then solve the equations.
Sol. The given equation can be written as
~
(I-A)X+2
Y
+3Z=0}
3x+(I-A) y+2z =0 ...(1)
2x+3y+(I-A)Z = 0
85
s
The system of equations AX = 0 has a non zero solution if the rank of the coefficient
matrix A is less than 3 i.e., IAI = O.
I-A 2 3
.
. .
I 3
I-A
2 1=0
2 3 I-A
I
=> (6- ..1)(..1
2
+3..1+3) =0
-3H
=>
A=60r ..1=-
2
Hence 6 is the only real value of Afor which the given system of equations has a non
zero solution.
Substituting ..1=6 in (l),we get
(7
Applying elementary row operations -t + ,we have
(
-:
5 -2 -3 z 0
Applying elementary row operations R2 -t + R" R3 -t + R, ,we have
5
; _ [XJ-[OJ
5 5 Y - 0
o 0 0 z 0
=>
-5x+2y+3z=0
19 19
--y+-z=O
5 5
O.z=O
Equation (3) is true for all values of z,so let
z =k , k be any number
...(1)
... (2)
...(3)
86
Putting the value of z in (1) and (2),we have
-5x+2y=-3k and y=k
=> x=k,y=k,z=k
This is the required solution.
~
[1 -2 3 J
Q.12. Verify that A(BC)=(AB)C for the matrices A= 2 3 -1'
-3 1 2
[1 02J [1 1 1 J
B = 0 1 2 and C = 1 2 -3.
1 2 0 2 -1 3
[1 -2 3 f 0 2J [1-0+3 0-2+6 2-4+0 J
Sol. AB = 2 3 -1 0 1 2 = 2+0-1 0+3-2 4+6-0
-3 1 2 1 2 0 -3+0+2 0+1+4 -6+2+0
[4 4-2J
=> AB= 1 1 10
-1 5 -4
Consider
[
4
4-2f 1 1 J [4+4-4 4+8+2 4-12-6 J
(AB)C = 1 1 10 1 2 -3 = 1+1+20 1+2-10 1-3+30
-1 5 -4 2 -1 3 -1+5-8 -1+10+4 -1-15-12
[4 14 -14J
=> (AB)C = 22 -7 28
-4 13 -28
Now
(1 02f 1 1 J [+0+4 1+0-2 1+0+6J
BC= 0 1 2 1 2 -3 = 0+1+4 0+2-2 0-3+6
1 ,2 0 2 -1 3 1+2+0 1+4+0 1-6+0
[5 -1 7 J
',' => BC= 5 0 3
3 5 -5
87
Consider
7 J (5-10+9 -1+0+15 7-6-15 J
-2 3](5 -1
3 -1 5 0 3 = 10+15-3 -2+0-5 14+9+5

-3 1 2 3 5 -5 -15+5+6 3+0+10 -21+3-10
4 14 -14J
=> A(BC) = 22 -7 28
[
-4 13 -28
Hence A(BC)=(AB)C.

=
1 IJ
Q.13. Verify Cayley-hamilton theorem for the matrix A 1 0 .
1 2
Sol. The characteristic equation of a matrix a is given by IA - 211 =O.
2-2 1 1
=>
I 0
1-2 o 1=0
1 1 2-2
=> 23_522+72-3=0.
According to Cayley-hamilton theorem
A3-5A2+7A-3I=O
Verification:
[2 I If I IJ [5 4 4J
A2 = 0 1 0 0 1 0 = 0 1 0
112112445
A
3 [5 4 4f I IJ (14 13 13J
= 0 1 0 0 1 0 = 0 1 0
4 4 5 1 1 2 13 13 14
[14 13 13J [5 4 4J [2 I IJ (1 0 OJ
A3-5A2+7A-3I= 0 1 0 -5 0 1 0 +7 0 1 o -3 0 1 0
13 13 14 4 4 5 1 1 200 1
88
14-25+14-3 13-20+7+0
13-20+7+0J
= 0+0+0+0 1-5+7-3 0-0+0-0
[
13-20+7+0 13-20+7-0 14-25+14-3

000
Hence verified.
r'
o 2]
Obtain the characteristic equation of the matrix A = [ 2 1 and hence
o 3
Calculate its inverse.
Sol. The characteristic equation of a matrix a is given by IA- All =0 .
I-A 0 2
=>
2-A 1 =0
I 0
2 0 3-A
=> -A
3
+6A
2
-7A-2=0.
=> A
3
-6A? +7A+2=0
According to Cayley-hamilton theorem
A
3
-6A2+7A+21=O ...(1)
Multiplying (1) by A-I ,we have
A-
I
A
3
-6A-
1
A
2
+7A-
1
A+2A-
1
I = A-tO
=> A2 -6Al +7+2A-
1
= 0

-I 1 2 3 7
=> A =--A + A--I ... (2)
2 3
Now
[1 02f 02] [5 08]
A2 = 0 2 1 0 2 1 = 2 4 5 ...(3)
2032038013
89
06
{
0 z
z z
-
{
-
{
{
= I-V
<:=
~ ~
Z;
0
[ ~
0
{
0Jf [f
z
0
<J [1
v
0
~
8)<
i-= I_V <:=
0
~ Z- ~ ~ + :
0
0
Unit - 4 : Integeral Calculus
Multiple Choice Type Questions:
Q.l. The value of the integral Jsin x dx is
(a) cosx+c (b) -cosx+c
(e) sinx+c (d) -sinx+c
Q.2. The value ofthe integral Je
3X
dx is
3e
3x
+c (a) (b) e
3x
+c
3x
e
(c) -+c (d) None of the above
3
Q.3. The area bounded by x-axis and the curve representing f (x) = sin x between x = 0
and X=7r is
(a) 2 (b) 1
(c) 3 (d)
Yz
Q.4. The area bounded by the region R={ ( x, y) :-1 ~ x ~ 1, -1 ~ Y~ I} is
(a) 1 (b) 2 (c) 3 (d) 4
Q.S. The area bounded by the straight line t y = 2x +3the x-axis, the line x = 1 and the
linex= 4 is
(a) 12 (b) 24 (c) 72 (d) 6
Q.6. The value of the integral f2 x
3
dx is
(a) o (b) 1 (c) 42 (d) 13
Q.7. The value ofthe integral J2dx ~ is
a +x
l 1 1
(a)
tan- (;)
(b)
~ tan-I (;)
(c)
COC (;) (d) ~ COC (;)
Q.8. The value of the integral Jsin x cos x dx is
(a) sinx+c (b) sin
2
x+c
(c) .!.sin
2
x+c (d) None of the above
2
91
Q.9. The integral is rf (x) dx equivalentto
(a)
rf(x}dx
(b)
rf(x}dx -
(e)
- [0 f(x)dx
(d) None of the above
J1-tanx
Q.l0. The value of dx is
1+tan x
(a) log(1 + tan x) (b) log ( sin x+cos x)
(e) log(l-tanx) (d) log ( sin x- cos x)
Q.ll. If all values of x between a and b are equally probable,the mean value of x is
a-b
a+b (d)
(a) a-b (b) a+b (e)
-
2 2
Q.12. The value of the integral J (1 ) dx is
x x+a
1 1
(a) -(logx-log(x+ a))+ c (b) - (log ( x +a) -logx) + c
a a
1 1
(e) -(log x+log(x+a))+c (d) -(logx(x+a))+c
a a
Q.13. The value of the integral J 1.[; dx is
. 1+ x
(a) 2.[;+tg(I+.[;)+C (b) 2.[; - /tOg ( 1 + .[;) + c
f\
~
(e) log(l+.[;)+c
(d) None of the above
4
Q.14. The value ofthe integral t sin
7
x cos X dx is
16 Jr 17 Jr
(a) (b)
1155'Z 1155'Z
17 16
(d)
c;;-
(e) -
. , 1155 1155
92
"
Q.1S.
Q.16.
Q.17.
Q.18.
..~ -.... Q.19
Q.ZO.
Q.Z1.
Q.ZZ.
Q.Z3.
The length of the arc of the curve y = f (x) between the points x = a and x = b is
(0) d1+(:)' dx (b) ~ 1 + ( : ) ' dx (e) 1+(:)'dx
(d) !1+(:)'tb;
The anti derivative of the-function cosx is
(a) sin x (b) -sin x (c) cos x (d) -cosx
The volume ofthe solid of revolution obtained by revolving about .......... The area bounded
by a curve y = f (x) ,the lines x = a, x = b and x-axis is ! 1l'l tb; .
(a) x-axis (b) y-axis
(c) the line y =-x (d) the line y = x
The surface area of a sphere of radius a is
(a) 21l'a (b) 1l'a
2
(c) 41l'a
2
(d) 21l'a
2
The value of the integral ! ! (x + y) tb; dy is
(a) 1 (b) 0 (c) -1 (d) 2
The value of the integral IIxy tb; dy over the region in the first quadrant for which
x+ y$1is
1 1
(a) (b) 12 (c) (d) 24
12 24
2
The area between the curves x
2
+ l =a and x+ y = a in the first quadrant is
1l' 2 a
2
1l' 2 a
2
(a) -a -- (b) -a +
4 2 4 2
1l' 2 a
2
1l' 2 a
2
(c) --a +- (d) --a -
4 2 4 2
2
The volume of the region bounded by the surface y = x , X = l and the planes
z=O,z =3is
(a) 1 (b) 2 (c) 3 (d) 4
The value of II f2 3 tb; dy dz is
(a) 12 (b) 36 (c) 48 (d) 60
93
Q.24. The value of the integral IIeX (log sin x + cot x) dx is
(a) eX log(sinx)+c (b) eX cotx+c
(c) eX sinx+c (d) None of the above
e s i n ~ l X
Q.25. The value of the integral J~ dx is
l-x
2
. -I
e
sm
x2eSin-Ix +c
(a) X +c (b)
1
(c) None of the above
,Jl_xZ+C (d)
Q.26. When, ) ~ ) is split up into partial fractions, the coefficient of _1_ is
x+l x-2 x+l
1 2 1 2
(a) (b) (c) (d)
3 3 3 3
Q.27. When x:-
7
. is split up into partial fractions, the coefficient of _1_ is
(x+l) (x-2) x-2
1 1 1
(a) - (b) (c)
-
(d) 1
3 9 27
Q.28.
The value ofthe integral f21xldx is
(a) 2 (b) 4 (c) 0 (d) 1
Q.29. If f (x) is an even function ,then f;r f (x) sin xdx is
(a) 0 (b) 1 (c) 2 (d) can't be determined
94
96
(e)
"6Z
(q)
"8Z
(l)
"LZ (e) '9Z
(e)
'5Z
(e)
"tZ
(l)
"EZ (e)
'zz
(e)
'U
(l) "OZ
(e)
"6t
(l)
'8t
(e)
'Lt (e)
'9t
(q)
'5t
(p) ott (q)
"Et
-
(e)
'Zt
(l)
Ott
(q)
'Ot
(q)
'6
(l) '8
(q)
'L
"",.
(e)
"9
(q)
'5
(p)
't
(e) "E (l)
'z
(q)
't
Ae)l
True /false Type Questions:
1. The value of the integral rf (x) dx vanishes if f (x) is an even function.
2. A function f (x) has infinite number of anti derivatives.
3. The area bounded by curves can be obtained by simple integration or double integration.
4. A definite integral can be expressed as the limit of the sum of a finite number of terms,
when the number of terms tends to infinity and each term tends to zero.
5. The process of determining the area of a plane region is called rectification.
6. The process of determining the length of a plane curve is called quadrature.
7. The process of finding the integral of product of two functions is called integration by
parts.
8. The product of two odd functions is an odd function.
9. The root mean square value of xi' x
2
' ....,xn is the square root of mean of the squares of
XpX2''xn
10. The method of partial fraction consists of an algebraic procedure for turning a difficult
integrand into a sum of two or more easier functions.
11. The value ofthe integral Jsec
2
xdx is tanx+c.
12. If F(x) is an anti derivative of a function f (x) ,then rf (x)dx= F (b)- F(a).
Key
1. T 2. F 3. F 4. T 5. F 6. T
7. F 8. T 9. T 10. F
96
Short Answer Type Questions:
Q.1. Evaluate
Ia
2
sin
2
X ~ b 2 cos
2
l
Sol. let 1= I dx
a
2
sin
2
x+b
2
cos
2
x
Dividing both numerator and denominator by cos
2
x ,we get
2
sec x dx
...(1)
I =I--2-+-a-
2
-tan-
2
:-x
b
Put t=tanx.
~ dt=sec
2
xdx
Substituting the value of t and dt iii (l),we get
1 dt
1= Ib2 +a2 e
~ I = ~ I 1 2 dt = ~ J 1 ~ dt
a 2 b a 2
t +2 t +l
a a
1 a _l(at) 1 -l(atanxJ
~ I =-.-tan - +c=-tan +c
a
2
b .b ab b
2
Q.2.
Evaluate Jtan,[; sec ,[;
,[; dx.
2
let I = J
tan
,[; sec ,[;
Sol. ~ dx ...(1)
Put t =tan,[; .
2
~ dt=sec ,[;
2,[; dx
~ 2dt =sec
2
dx
97
Substituting the value of t and dt in (l),we get
I =2 It dt =t
2
+C =tan
2
,J; +C
1
Q.3. Evaluate dx .
J
cos(x-a)sin( x-b)
Sol. We have
cos ( a-b) =cos[( x-b)-(x-a)]
=cos(x-b)cos(x-a)+sin ( x-b)sin( x-a)
Dividing both sides by cos (x - a) sin (x - b) ,we get
cos(a-b) cos ( x - b) cos ( x- a) +sin ( x- b) sin ( x - a)
cos ( x - a) sin ( x - b) cos(x-a)sin(x-b)
cos(a-b)
=>
cos( x-a)sin( x-b\ cot(x-b)+tan(x-a)
Integrating both sides w.r.t x,we get
cos(a-b) J ( ~ . ( \dx =fcot(x-b )dx+ ftan (x-a)dx
~ )
cos x-a sm x-b
4. l"
=log sin ( x - b) +log sec ( x- a) +C
r": ~ b ~
~
=>
J 1 1
cos( x-a)sin( x-b) dx =cos ( a-b\ logsin(x-
b
l+log sec ( x-a)+c
1 dx 1 I [sin(X-b)]
= og Ct.
J
cos(x-a)sin(x-b) cos(a-b) cos(x-a)
98
x
2
+1
a.4. Evaluate I ~ eXdx .
(x+l)
x
2
+1
Sol. Let 1 = I ~ eXdx
(x+!)
2
1 =Ix +
1
+1-1 eXdx =IXl -1 +
2
eXdx = l x -1 + 2 ] eX dx
(x+l)2 (x+l)2 Jl
x
+l (x+lt
1
I
X
I - 2
=> 1= -exdx+ 2exdx
x+l (x+l)
Integrating the first term in above integral by parts,we get
x-I X I(X+l).I-(X-l).1 xdx+J 2 ~ e x d x + c
I--e - 2 e ()
- x+l (x+l) x+l
x-I x I 2 . I 2 .
1 =_e - eX dx+ eX dx+c
=>
x+1 (x+1)2 (x+ 1)2
x-I
=>
I=--ex+c.
x+l
a.s. Evaluate J 21( . dx .
(x+l) x
2
+1
I Sol. 1 dx ...(1)
(x + 1)2 ( x
2
+ 1 )
1 A B Cx+D
Let = - + +----,:---- ...(2)
(x+l)2(x2+1) x+l (x+l)2 x
2
+1
2
2
=> 1= A(x +1) (X2 +1) + B (x +1) + ( C x + D) ( x +1)
Putting x =1 we get B =.!:. .
2
3 2
Compairing the coefficients of x ,x ,x respectively,we get
A+ C = 0, A + B + 2C + D =0, A+ C + 2D =0
99
'--
Solving these equations ,we get
.. 1. 1 ..
A=- c=-- D=O
2' 2'
Substi!lJ!i"'S these valuesil'l
1 11111 x
---:-....,-----,- = - - +- -- -
(x+l)2(x2+1) 2'x+l 2'(x+l)2 2'x
2
+1
Integrating on both sides ,we get
I III 1 II 1 1 x
dx dx
(X+l)2(x2+1)dx="2 x+l +2 (X+l)2 -"2 Ix2+1dx
1 1 1 1 2
=-log(x+l)------log(x +l)+c
2 2(x+l) 4
2
Q.6.
x dx.
Evaluate
I

Sol. We have
J,- _,dx ...(1)
Putting x
2
= y ,we get
x
2
_ Y
(X2 +a
2
)( x
2
+b
2
) - (y+a
2
)(y +b
2
)
We shall simplify this by partial fractions.
y A B
Let =--+-
2
(y+a
2
)(y+b
2
) y+a y+b
2
=>
y = A(y+b
2
)+ B(y+a
2
) ...(2)
Putting y = _a
2
in (2),we get
100
2
Q.7.
Sol.
l
2 2
a
2
_a = A(_a
2
+b
2
)+ B(_a +a
2
)
:::::> A=-2-b
a
Putting y = _b
2
in (2)Jwe get
b
2
":'b
2
== A("":b
2
+b
2
)+ B(::::b
2
+ii)
:::::>B= a2 _b2
2 2 2
b
2
x a x x
dx= dx- dx J J
J
(x2+a
2
)(x2+b
2
) a
2
_b
2
(x2+a
2
) a
2
_b
2
(x
2
+b
2
)
2 2
1 b -I (x)
---tan
a -1 (x)
-
1
tan - +c
2
b
2 2
a - a a a - b
2
b b
-
a
tan-
-I (x)
b _I(X)
2-2tan - +c.
J
- a
2
_b
2
a
a -b b
1 dx.
Evaluate '3x2 +4x +5
We have
2 4 5) ( 4 5 2 2)
3x
2
+4x+5=3
(
x +-x+- =3 x
2
+-x+---+
3 3 3 3 9 9
=3[(x+%)' +~ J=3((x+j)' +[ ~ J ' )
substituting this value in the given integralJwe have
1 dx __
1
J 1 dx
f"3X'+4x+5 -.J3 tx+%)' +[ ~ J
1 .nh-
1
[X+2/3]
= .JjSl .J1i/3 +c
=-SI
1 .nh-
1
(3X+
--
2)
+c .
.Jj .J1i
101
J
x+2
Q.8.
Evaluate J dx .
x
2
+3x+l
2
Sol. let 1 =Jj x+ dx ..(1)
let)iaralpbetwotohstantssuchthar
+3x+l)+,u
dx
X+2=A(2x+3)+,u.
Comparing the coefficients of x and constant terms on both sides, we have
1 11_1..
A='2''-- 2
1 1
x+2=-(2x+3)+- ...(2)
2 2
Using (2) in (1},we get
1 1
-(2x+3)+- 1 (2x+3) 1 1
1= J2 2 dx =- J dx+- J dx
+3x+l . 2 +3x+l 2 +3x+l
=1
1
+/
2
...(3)
1 J(2x+3) 1 J. 1
where, 11 =- I dx and 12 =- J dx.
2 ... x
2
+3x+l 2 x
2
+3x+l
Consider
1 (2x+3) dx
11 ='2
Put t =x
2
+3x+1
dt=(2x+3)dx
2
11 =1.. J =.Jx +3x+l+c
1
2 'It
102
Now,
1, = ~ JJi:3x+1<h = iJ T(x + ~ J ~ ( ~ J
1 _ (2X+3)
= 2 cosh 1 J5 +C
2
Putting the value of I) '/2 in (3),we get
2
I = ..Ix + 3x + 1 + ~ cosh-1 ( 2 ~
3
) + C
r x sin-I x dx
Q.9.
Evaluate .b ..11- x2
Sol. We have
-I
r
xsm
x dx
...(1)
.b .Jl-x2
Put x=sin8
::::? dx=cos8d8
When x =O,sin 8 =0 ::::? 8=0.
When x = I, sin 8 = 1 ::::?
8=1C
2'
. rxsin-I x r
/2
sin 8.8. cos 8 d
il
r
/2
.
dx
_
. .b r:--; - 17 = 8sm 8d8
'\11- x
2
.Jl-sin
2
8
Integrating by parts,we get
-1
rxsm x 1 tr/2
/2
r
.b ..11- x
2
dx =-8cos Blo + cos 8d8 = 1-
8
cos 81:/
2
+Isin 81:/
2
1C
--.0+1-0=1.
2
103
L
Q.l0.
Sol.
r
/2

Evaluate r:-: r-- dx .
vsmx+vcosx
We know ,by property of definite integral,that
Jf(x)dx= [t(a-x)dx ...(1)
let . ...(2)

(1& )
1& sm --x
=> f --x)= 2 ,--
2 v cos x CfJ
(
-x) ,Jcosx+,Jsinx ...(3)
Using (1M2), and (3),we get
r
/2
r
/2
.Jsin x dx .Jcos x dx I (
= = say)
.Jsin x +.Jcos x .Jcos x + .Jsin x
21= dx= r/21dx=lxl:/2 =1&
=>
.Jsmx +.Jcosx 2
1= 1&
=>
4'
r
/2
dx-1&
Hence. - .
.Jsm x + .Jcos x 4
2
Q.ll. Find the circumference of the circle x
2
+l = a
2
Sol. We have x
2
+ l =a
Differentiating both sides w.r.t. 'x,we get
dy
2x+2y-=O
dx
dy =_x dy =+__x
=> =>
dx y dx - .Ja
2
+X2
104
B
A
Circumference of the circle = 4 X length of arc AB.
Length of the arc AB = r~ 1 +(:)'dx
=r 11+( x J2 dx= r a dx=a r--;==l=dx
.h \ ~ a 2 +X2 .h '_2. __2 .h;
a
-1 X Ir
= aIsm
(
-;;
)I
0 = a"2
:. Circumference ofthe circle = 4xa Ir = 2lra.
2
Q.12. Evaluate IIxy dx dy where Ais the domain bounded by x-axis,ordinate x = 2a and the
A
curve x
2
= 4ay .
Sol. The area of integration is OAB.
2
The limits of integration are 0 to 2a for x and 0 to ~ for y.
4a
:. IIxydxdy =fa (;{a xydxdy =f
axdx
(;{a y dy
A
105
-
x
2
/
. a I2\ /4a r X4 1 r
= f xdxL =
a
xdx.--=--
a
rdx
.b 2 32a
2
32a
2
o
6 2a 4
1 X 1 6 a
--- =-- 2a -0-
t ...
I
6..... 192a
2
((-) )-3.'
0 ..
III(x-2
y
+z)dxd
y
dz where R:O::S;x::S;l,O::S; y::S; x
2
,0::S; z::S; x+ y.
Sol. III(x-2y+z)dxdydZ
R
r r
x2
rx+
Y
=.b.b .b (x-2y+z)dzdydx
= L r2Ixz_2YZ+\x+y dydx
2 0
=1t[x(x+ y)-2y(z)x+y+ (x+
Y
)']dylh
2
2
2[
2
1 1 J
= L r x +xy-2yx-2l +2'x +2'l +xy dydx
3
2
J 3rrX2[ 2 2 2J
=.b.b 2'x -2'y d
y
dx=2'.b.b x -y dydx

L[X2y_.i.]X2 L[x
4
_ X6]X2 _ X7]1
2 3 0 2 3 0 2 5 21 0
"2l.
:
1

2 5 21 35 t1 ::.. 0
'l( - ), (-....---;)
Q.14. Find the volume of the region bounded by the surface y = x
2
,x'= l and the planes
. .
z =O,z =3.

2
Sol. Solving the two equations y =x ,x =l,we get x =0,1.
2
:. ,z varies from 0 to 3;y varies from x to J; and x varies from 0 to 1.
Required volume = L t 1dz dy dx
106
= ! rdydx
=3!lyl:: dx=3!(-X2}4x
2 ... 1 21
=3
(
3
i
3
/
2
-3
X3
)
1
0 =3
(
3-3
)
=1.
Long Answer Type Questions:
x
2
+1
Q.l.
Evaluate J 2;):?? ar
(x+l) (x-2)
x
2
+1 ABC D
Sol. Let 2 =--+ 2 + 3 +--,
...(1)
(x+l x-2) x+l (x+l) (x+l) x-2
=> x
2
+1= A(x+l)2 (x-2}+B(x+ 1)(x-2)+C(x-2)+D(x+l)3 ...(2)
Putting x = 2 in (2),we get
(2)2 + 1 = A{2+1)2 (2-2)+B(2+1)(2-2)+C{2-2)+ D{2+1)3
5
D=-.
=> 27
Putting x = -I in (2),we get
{_1)2 + 1 = A(-1 + 1)2 (-1-2)+B{-1+1)(-1-2)+C(-1-2)+ D{ -1 +If
=> C=_3.
3'
Putting x = 0 in (2),we get
(2t +1 = A(0+1)2 (0-2)+ B(O+1)(0-2)+C(0-2)+D(0+1)3
4 5 14
=> 2A+2B=-2C+D-l=-+--I=
3 27 27
7
...(3)
=> A+B= 27'
107
Putting x =1 in (2),we get
(1)2 +1 = A(1+1)\1-2)+8(1+1)(1-2)+C(1-2)+D(1+1)3
4
=> 4A+2B=-C+8D-2=
27
2
=> 2A+B=-, ..,(4)
27
Solving (3) and (4),we get
5 4
A=-- B=
27' 9
(1) becomes
x
2
+ 1 5 1 4 1 (2) 1 5 1
(x+lt(x-2) =- 27' x+l + 9' (x+l)2 + -3" (x+lf + 27' x-2
":': ,.E
Integrating both sides ,we get
x2+1 1 1
J
(X+1)2(X-2) 27 x+l 9 (X+1)2 3 (x+l)3 27 x-2
5 4 1 5
=--log(x+l)- + +-log(x-2)
27 9(x+l) 3(x+l)2 27
4 1
5
=-log
(X-2)
-- -+- ,
27 x+l 9(x+l) 3(x+l)2
Q.2. Evaluate JSin
m
xcos
n
xdx,mand n being positive integers,
Sol. We have
JSin
m
xcos
n
xdx= Jsin
m
-
1
xsinx( cos
n
x)dx
Integrating by parts ,we get
cos
n
+! xsin
m
-! x + m-l Jcosn+1xsin
m
- 2xcosxdx
JSin
m
xcos
n
xdx =
n+l n+l
108
n 1 m1
_ COS + xsin - x + m-l Jcos
n
xsin
m
-
2
XCOS
2
xdx
- n+l n+l
n m1
=_ cos +! xsin - x + m-l Jcos
n
xsin
m
-
2
x(l-sin
2
x)dx
n+l n+l
n+l m-I X m-1 J . m-2 dx
_ cos xsm + __ cos
n
xsm x
- n+l n+l
m-l J
--- sin
m
xcos
n
xdx
n+l
n 1 m1
cos + xsin - x + m-l Jcos
n
xsin
m
-
2
xdx
:::::::> (1+ m-l) JSin
m
xcos
n
xdx=
n+l n+l n+l
m+n) . cosn+1xsinm-1x m-l ._
m
:::::::> -- Jsmm xcos
n
xdx= - +-- Jcos
n
xsm 2xdx
(
n+l n+l n+l
n+! m-I X m-1 J n m-2 dx
cos xsm + __ cos xsm x .
:::::::> JSin
m
XCOS
n
xdx =
m+n m+n
Q.3. Evaluate Jcos
m
xcosnxdx,mand n being positive integers.
Sol. We have
Jcos
m
xcosnxdx
Integrating by parts,we get
m I
J cos
m
xcos nxdx = cos x sin nx + m J cos
m
- X sin x sin nxdx
n n
We know that
cos (n -1) x =cos nxcos x + sin nxsin x
:::::::> sinnxsinx =cos(n-l)x-cosnxcosx
m
Jcos
m
xcosnxdx = cos x sin nx + m Jcos
m
-! x(cos(n-l}x-cosnxcosx)dx
n n
m
sin nx m J -I m J
=cos x--+- cos
m
xcos(n-l)xdx-- cos
m
xcosnxdx
n n n
109
. ( m) I
m
sin nx m I
m

=> cos
m
xcosnxdx =cos x--+- cos 1 xcos(n-1)xdx
n n n
m+n) I sinnx m I
m

=> -n- cos


m
xcosnxdx=cos
m
x--+-; cos 1 xcos(n-l)xdx
(
n
I
sinnx m m-1 I
=> COS
m
xcosnxdx =COS
m
X--+-- COS xcos(n-l)xdx
m+n m+n
Q.4.
/x+2
Evaluate IV2x+"3 x
Sol.
/x+2
We have IV2x+"3 x
x+2 2
Put --=t
2x+3
2
=> x= 3t -2
1
2t
=> dx= dt
(1- 2t
2
)
A
/x+2 1dx - 2 It 1- 2t
2
t dt
I
V2x+"3-; -- '3t
2
-2" (1-2t
2
f
2 2
t t
=-2I( 2 \{ 2\dt=2I - .dt
__ ." 1
- 2I dt I dt
- - (2t2 -1) -4 (3t2 _ 2) (Using partial fractions)

110
1 (t-*)
( 1)

v2 t+-
Ji
___ (Jit-l) (.J3t-Ji) 1
- log log (r:::; +C
v2 v2t+l 3 v3t+v2
J
dx
Q.5. Evaluate .
(x+2)../x
2
+3x+4
1 1 1-2t
Sol. Put x+2=- => dx=--dt and x=-
t
2
t t
dx dt --J
f(X+2)N+3X+4
. 1 dt
dt =-
=-J../
2t
2
-t+
1 v2 t 2 --+-2
2
1 dt
=--JF-Rl 1 1
Ji t2 __+_+___
2 244
=- h
1
f '(t-H
dt
":J
t-
1 . -1 4
[
1]
=- hsIDh ": +c
1 . _1[4t-l]
=- Jismh .fi +C
111
1 4(_1)
=_ x+2 -1
v2 47 l+c
- ----= __Sinh-;[-2=X
J2 J7(2+x)
4a
Q.6. Show thatthe length ofthe loop of the curve 3a/ = x( x- a)2 is -,
J3
Sol. The curve is symmetrical about x-axis and it passes through the origin. Its point of
intersection with the x-axis is A (a, 0) .
y
o
The length of the loop =2r .
2 )2.4a
Wehave 3ay =x(x-a IS J3
y_ 1 r
=> - Favx(x-a)
=>
: = k(..fx+
1+(d
y
)2 =1+ (3x-a)2 = (3x+a)2
=>
dx 12ax 12ax
112
(3x+a) dx
The length of the loop = 2 r~
1 art a J
= 5a ~ l 3 . . J x +..Jx dx
------------_....
1
=5a(2a
3
/
2
+2a.al/
2
)
4a
=.J3'
2 2
b
2
Q.7. The ellipse b
2
x +a
2
i =a is divided into two parts by the line x= a and the
2
smaller part is rotated through four right angles about this line.Prove that the volume
2
generated is Jm b( %.J3 - ; ) .
Sol.
The shaded portion is rotated about the line x = E:. which is parallel to y-axis.
2
The points of intersection of the line x = a with the given ellipse is given by
2
2
a
2
b
2
b
2
-+a
2
i =a
4
113
=>
y=..[3b
2
is
2
"
J3b (
a
V=2127& x- a
)2
dy
where
x
2
=-r(b
2
-l)
Jy=!) 2
b
J3b (
2
a2)2
=2..(: 7& x -ax+
4
dy
J3b(a
2
aZ a
2
)2
=27&"(: dy
a
2
a
2 2
J3b
a
2
y(b
2
-l) b -1 Y
J3b j
l"2 J3b
= 27& -
2
b
2
y-- +-sm --
2
+-IYI2
b 3 b 2 2 b 4 0
r
o 0
=27&[a
b
2
2
(..[3 b2 _3..[3 b
2
_7&)+ ..[3 aZb]
2 24 4 4 2 2 3 8
-: -
Q.8. Find the surface of revolution of the solid generated by revolviong the
parabola yZ =4ax bounded by its latus rectum about x-axis.
Sol. Required surface area is
s=
We have
yZ =4ax
dy J"d
=>
dx=.rx
S =r27&J"d.rxJl +:dx
114
arc of the
...~ ~ ' . f
=4f(fa r.Ja +xdx
=4f(fa ~ l ( a +xt121:
8f( r[(2 )3
/
2 3
/
2J
=-"a a -a
3
=*f(a
2
[2.J2 -1].
Q.9. Changing the order of integration of r r e-xy sin nxdx dy ,show that rsin nx dx = f( .
x 2
Sol. We have r r e-XY sinnxdx dy
= rdy re-XY sinnxdx ...(1)
First we shall find the integral re-xy sinnx dx .
-xy -xy
Je-XY sinnxdx = -sinnx-=--- J--=--.ncosnxdx
y y
-xy
. e n J-xy dx
=-Slllnx--+- e cosnx
y y
e-XY n [ e-XY Je-XY ]
=-sinnx--+- ---cosnx- ---.nsinnxdx
y y y y
-xy 2
n dx
=-slllnx--..:..-e
. e - ~
-, cosnx--
n J- ~ .
e -, smnx
y l l
n2) _. . e-xy n_
=> 1+-
2
Je xy slllnxdx=-Slllnx----
2
e xy cosnx
(
y y y
=> Je-XYsinnxdx= 2 y 2e-XYsinnx- 2 n _e-XYcosnx
n +y n +y
115
-xy
Ie-XY sinnxdx=: 2 (-ysinnx-e-XY cosnx) ...(2)
n +y
Using (2) in (l),we get
r r e-XY sinnxdxdy = r d
Y
[ :-xy 2
n +y 0
= rdY[O+ 2n 2]= r[ 2n 2]dY
n +y n +y

f(
=Itan-
I
Yo =2"
...(3)
On changing the order of integration,we get
r re-XYsinnxdxdy= rsinnxdxre-XYdy
= r = r sin nxdx(-0+1)
o
=r sinnxdx
. (4)
From (3) and (4),we get
sinnx
r dx- f(
-x- -2"'
Q.I0. Evaluate IJI( x
2
+y2 +z2)dxdydz R denotes the region bounded by
R
X =0, y =0, z =0, and x +y+ z =a ,a >.
Sol. We have
x+y+z=a z=a-x-y
Upper limit of z =a-x- y.
Onx-yplane x+y+z=a becomes x+y=a y=a-x.
Upper limit of y =a - x
116
Upper limit of x =a
JIJ( X
2
+l + z2)dxdydZ
R
= rr-x r-X-Y(x2+l+z2)dxdydz
X
Y
""',
= rdxr-
x
dyr- - (x
2
+l+z
2
)dz
= r dx r-x dy r->:-y (X2 +y2 +Z2 )dz
=' dx r dy(x'z+ /z+ ;r-'
= ' dx r dy(x' (a-x- y)+ y' (a-x- y)+ y)' J

2
(
a-x-y
)
3)
= rdx r X
2
(a - x) - x Y + ( a- x) l-l + 3 dy
(
2 3 4 ( 4 )a-x
= dx x
2
(a-x)y-x
2L
+(a-x)L_L+ a-x- y)
. 2 3 4 12
r
(
o
= rdx(x2(a_x)2 _x2(a_x)2 +(a-x) (a_x)3 (a-xt +(a-x-(a-x)f]
2 3 4 12
='( (a-x)' + r= ' Ma'x' -2a.'+x' +
2
51
=Iia - a: + _(a;;)
a
=: -: + + = .
o
Q.11. Find the volume bounded by the cylinder x
2
+l =4 and the planes y +Z = 3 and
z=O.
117
-xy
=> Ie-XYsinnxdx=: 2 (-ysinnx-e-XYcosnx) ...(2)
n +y
Using (2) in (l),we get
r r e-XY sinnxdxdy =rd
Y
[ :-xy 2 (-YSinnx-ncosnx)]OC
n +y 0
= rdY[O+ 2n 2]= r[ 2n 2]dY
n +y n +y

tr
=ltan-
1
Yo =2"
...(3)
On changing the order of integration,we get
r r e-xy sin nxdxdy = rsin nxdx re-xy dy
=r =rsinnxdx{-O+l)
o
=rsinnxdx
.. (4)
From (3) and (4),we get
rsinnx dx= tr .
x 2
Q.l0. Evaluate III( x
2
+l +Z2) dx dy dz R denotes the region bounded by
R
X =0, y =0, z =0, and x + y+z =a, a >.
Sol. We have
x+y+z=a =>
z=a-x-y
Upper limit of z = a-x- y.
Onx-yplane x+y+z=a becomes x+y=a => y=a-x.
Upper limit of y =a - x
116
2
Sol. We have x + l =4 => y=.J4-x
2
y+z=3
=>
z=3-y
z=o.
:. zvaries from 0 to 3-y,yvariesfrom y=-.J4-x
2
to y=.J4-x
2

On the x-y plane, x
2
+ l = 4,:. x varies from -2 to 2.
f P r'-Y
:. Required volume =12 dx iMdy.b dz
=f
2
dx =t
dx
C; (3- y)dy
'2 I L,
2'
M
= Ldx3y- 21_M
= l,,+J4-x' +3J4-x' +
= 6 r.J4- x
2
dx = x
2

2 2 2 2 -2
= 6[2 sin-l %- 2 sin-l = 12[ + = 12K.
0.12. Evaluate JJ( x
2
+ l )dx dy throughout the area enclosed by the curves
y =4x, x +y =3, y = 0, y = 2 .
Sol. let OC represents y =4x ;AB x+ y =3 ;OA y = 0 and CB y =2.
The given integral is to be evaluated over the area of the trapezium OCBA.
Area OCBA consists of area OCD,area DCBE,and area EBA.
The co-ordinates of C,D and Bare 2). (1,2) and (3,0) respectively.
118
y 2
8(1,2)
o
E(I,O) A(3,O)
JJ(X
Z
+ l)dxdy= JJ (XZ + l)dxdy+ JJ (XZ + l)dxdy+ JJ(X
Z
+ l)dxdy
OCD DCBE EBA
=1'Z rX(xZ+l)dydx+ tzl(x
2
+
l
)dydx+ f r-X(x2+l)dydx
= I, +12 +13 (say)
Now, I, = rnx'+y')dydx= r(x'y+ dx
= r(x'(4x)+(4;)']dx
= r/276 x3dx =761x4\
1/2
=.!2.
0 48 .b3 34
I, = !,f(hy')dydx= L(x'y+ dx
= t,(x'(2)+

3 J3-X
13 = f rX(x2 + l)dydx= f
(
x
2
y+ 0 dx
119
\
= r(x'y+ dx=r(x' (3-x)+ Jdx
= r[3X' -x' + -: I
22
3
II( x
2
+ l)dx dy = 19 + 23 + 22 = 463
48 12 3 48'
120
l
\
I
Unit - 5 : Probability and Statistic
Multiple Choice Type Questions:
Q.l. Two dice,one is green and one is blue are thrown. What is the probability that both the dice
show 5 is
1 1 5 1
(a) (b) (c) (d)
18 36 36 12
Q.2. A number is chosen from the first 90 natural numbers.The probability that the number chosen
is a multiple of 5 or 15 is
1 1 .
(a)
.!. (b) !
(c) (d)
5 8 6 10
Q.3. A problem in mathematics is given to three students A,B and C whose chances of solving it are
respectively .!.,!and.!.. The probability that the problem is solved is
2 3 4
1 1 3 1
(a) (b) (c) (d)
2 4 4 24
Q.4. If A,B,C are three independent events,then P( AnBn C) is
(a) P(A)+P(B)+P(C) (b) P(A)P(B)P(C)
(c) P(AUBUC) (d) None ofthe above
Q.S. If A and AI are complementary events in a sample space S,then
(a) P{A)+P(AI)=O (b) P{A}+P(A
I
)=1
(c) P(A)-P(AI)=O (d) P(A)-P(AI) =1
Q.6. In tossing a fair dice,the probability of getting an odd number or a number less than 6 is
(a) 1 (b) 3. (c) 1 (d) 1
3 3 2 6
Q.7. The probability that a non-leap year will contain 53 Sundays is
1 (b) 2 (c) 3 (d) 4
(a)
7 7 7 7
Q.8. The probability that at least one of the events A and B occurring is 0.8 and the probability
that both the events occur simultaneously is 0.25.The probability P(A) +P(AI) is
(a) 1.65 (b) 1.05 (c) 1.85 (d) 0.95
121
4
Q.9. The mean and variance of Binomial distribution are 4and - respectively.The value of n is
3
(a) 12 (b) 4 (c) 5 (d) 6
Q.l0. The probability of having at least one tail in 4 throws with a coin is
15 1 1
(a) (d) 1
16 (b) 16 (c) "4
Q.ll. let X be a Poisson random variable, such that P ( X =0) =P (X =1). The standard
deviation of X is
(a) 4 (b) 2 (c) J2 (d) 1
Q.12. A manufacturer of steel blades found 5% of its blade defective.He sells blades in packets
each containing 5 blades.The probability that a packet contains one defective blade is
(a) 0.25
e
-O
25
(b) 0.5 (c) e-O
25
(d) 0.25
e-
X
x ~ O
Q.13. Given a probability density function f (x) = ' .The probability P(1 S; xS; 2) is
{
o ,x<O
1
+e-
2 1
e-
2 -1 -2
(a) e- (b) e- (c) (d)
e -e
Q.14. Students of a class were given an aptitude test.Their marks were found to be normally
distributed with mean 60 and standard deviation 5.The percentage of students who scored
more than 60 marks is
(a) 40% (b) 50% (c) 60% (d) 70%
Q.1S. let X be a random variable having normal distribution. If p( X <0) = p( X> 1) ,the mean
value of X is
(a) 0 (b) 1 (c) 1.5 (d) 2
Q.16. The mean deviation about the mean for a normal distribution with standard deviation (j is
423
(a) -(j (b) -(j (c) -(1 (d) (j
555
Q.17. An urn contains 10 black and 10 white balls. Find the probability of drawing 2 balls of the
same colour.
8 9 10 11
(al (b) (c) (d)
19 19 19 19
Q.18. A speaks truth in 75% and B is 80% of the cases. The percentage of cases in which they are
likely to contradict each other narrating the same incident is
(a) 25% (b) 30% (c) 35% (d) 40%
122
0.19. Six boys and six girls sit at random in a row; The probability that the girls are all together is
(a) ~ (b) 9 ~ 4 (c) 1 ~ 2 (d) 1 ~ 4
Q.20. The probability of getting 4 heads in 6 tosses of a fair coin is
(a) .!!. (b) Q (c) ~ (d) .!2
64 64 64 64
Q.21. For any two events A and B , P( A I B) is
(a) P(A)
(b) P(B)
p(AnB)
(c)
P(B)
p(AnB)
(d)
P(A)
Q.22. The mean and variance of Poisson variate are same.This statement is
(a) always true (b) always false (c) partially true (d) never true
1 1 1
Q.23. For two events A and B,let P(A) =-,P(B) =-,P(AUB) =-then P(AI B) is
3 4 2
(a)
.!. (b) .!. (c).!. (d) 1
3 4 2 12
0.24. Which of the following is not true for independent events
(a) P(AIB)=P(A) (b) P(BIA}=P(B)
(c) p(AnB) =P(A)P(B) (d)
P(AUB) =P(A)+P(B)
Q.25. Which of the following can serve as probability distribution?
x-2 ~
(a) I(x)=-; x=I,2,3,4 (b) I(x)=-; x=I,2,3,4
2 ~
x-2 1
(c)
l(x)=-5-; x=I,2,3,4 (d) 1(x) ="4 ; x=1,2,3,4
123
Q.26. Which of the following can not serve as probability distributiOn?
(a) f(x}=-
1
; x=1,2,3,4,5,6 (b) f(x}=2 ; x=1,2
6
1 1

(d)
f ; x=1,2
Q.27. Let A and B be two independent events with probability P(A) : 0 < P(A) < t
P(B) : 0 < P(B) < 1then
(a) A and B' are independent (b) P(AUB)=l
(c) P(A)+P(B)Sl
(d) A and B are mutually exclusive
Key
1. (b) 2. (a) 3. (c) 4. (b) 5. (b) 6. (a)
7. (a) 8. (b) 9. (d) 10. (a) 11. (d) 12. (a)
13. (d) 14. (b) 15. (b) 16. (a) 17. (b) 18. (c)
19. (c) 20. (c) 21. (c) 22. (a) 23. (a) 24. (d)
25. (d) 26. (d) 27. (a)
True/False Type Questions:
1. The mean and variance of a poisson variate are same.
2. The normal curve is symmetrical about x-axis.
3. The area under a normal curve is unity.
4. Normal distribution is the limiting form of Binomial distribution for large values of n
and p,q are not very small.
5. Poisson distribution is the limiting form of Binomial distribution when p is very large
and n is small.
6. For a function f (x) to be a probability density function, f (x) should be positive
in the entire domain.
7. If nand p are the parameters of a Binomial distribution, the standard deviation is npq.
8. If P(AUB) = P(A)+ P(B) ,the events A and B are mutually exclusive.
124
9. A husband and wife appear in a interview for two vacancies in the same position .The
probability of husband's selection is 1/7 and that of wife's selection is 1/5 and both of
them will be selected is 1/35 .
10. the probabilltyl'>footalrilrig-an even numoerlsl/3. ._
11. For any two events A and B, P(A)+ P(B)::; p(AnB).
12. Normal distribution is a discrete distribution.
13. Binomial distribution is a continuous distribution.
Key
1. T 2. F 3. T 4. T 5. F 6. T
7. T 8. T 9. T 10. F 11. F 12. F
13. F
Short answer type questions:
Q.l. Three machines 1,11 and III manufacture respectively 0.4,0.5 and 0.1 of the total
'\ \) production. The percentage of defective items produced by 1,11 and III is 2,4and 1 percent
espectively. For an item chosen at random, what is the probability it is defective?
, 0.4x2 0.8
Sol. The defective item produced by machme I = =
The defective item produced by machine II
.
The defective item produced by machine III
'I . 0.8 2
100 100
0.5x4 2
= --= - .
100 100
0.1x1 0.1
=--= - .
100 100
0.1 0 29
h - + - + - =.0 . T e reqUired probabl ity IS
100 100 100
125
- -
,- - .
Q.2. Three groups of children contains respectively 3 girls and 1 boy
/
2 girls and 2 boys and 1
girl and 3 boys.One child is selected at random from each group. Show that the chance
that the three selected consists of 1 girl and 2 boys is 13/32.
Sol. We can select 1 girl and 2 boys in the following mutually exclusive ways:
1
st
2
nd
3
rd
group group group
Girl Boy Boy
II Boy Girl Boy
III Boy Boy Girl
By theorem of total probabilityl
Required probability =P(I) +P(II) +P(III)
The probability of selecting a girl from the first group is 3/4 lof selecting a boy from the
second group is 2/4
1
and of selecting a boy from the third group is 3/4.
Since three events of selecting children from three groups are independent of each
other
l
therefore
3 2 3 _ ~
P(l)=4
x
4
X
4- 32
1 2 3 _ ~ I
SimiiarlYI
P(ll) =4
x
4
x
4 - 32
1 2 1 _ 1
P(l1l)=4
x
4
x
4- 32
9 3 1 13
Required probability =32 + 32 + 32 =32
Q.3. Two dice are thrown simultaneously .Find the probability of getting
i. An odd number on the first die
ii. An ace on atleast one of the die
Sol. The total number of possible outcomes when two dice are thrown is 36.
=> n(S ) =36 I where S denotes the sample space
(a) Let A be the event of occurring an odd number on the first die.
Then A ={(x, y): x=1,3,5;y =1,2,3,4,5,6}
126
n(A)=3x6=18
P(A)= n(A) =.!!_l
n(S) 36-"2
(b) Let A bethe-event of occurring anaceon
Then A= {{I,x): x = 1,2,3,4,5,6}U{{x,I): x = I, 2,3,4,5,6}n(I,I)
n{A)=6+6-1=11
\
0'
y,
o

A can hit a shots,B 2 times in 5 shots and C three times in 4 shots,AII of them
Q.4.
fire one shot each simultaneously at the target,What is the probability that 2 shots hit?
Sol. Let A,B and C be the events that A,B and C hits the target ,respectively,
323
Then peA) = -,PCB) =-,P(C) =
5 5 4
- 2 - 3 - 1
peA) = -, PCB) = -, P(C) =
554
Probability that 2 shots hit the target is
P(A)P(B)P(E)+P{A)P(B)P{C)+P(X)P{B)P(C)
321 333 223 6+27+12
=5'5'4+5'5'4+5'5' 4 =--10-0
45 9
=-=
100 20
Q.5. A,B and C in order toss a coin .The first one to throw a head wins.What are the chances
that A wins if A starts the game and assuming that the game may continue indefinitely?
Sol. The probability that A,B and C get a head in a toss are
1
peA) =PCB) =P(C) =
2
- - - 1 1
P(A)=P(B)=P(C) 1--=
2 2
127

If A begins,he can win in the following mutually exclusive ways:
A, ARCA, ARCABCA, ...
By theorem of total probability, probability that A wins is
...
=+GJ+(H+... =[1+()' +GJ' +...]
1 1 4
= 2'1- (1/8) = 7'
Q.6. In a book of 520 pages, 390 typographical errors occur. What is the probability
that random sample of 4 pages will contain no error?
Sol. The average number of errors is
A=390
520 4
Let X=number of error per page
P(X =x) =e-l AX
x!
=e-3/4 (3/4Y
xl
_.. P(X =0)=e-
3
!4 (3/4/ = e-
3
/
4

O!
Probabilty that random sample of 4 pages will contain no error is
[P(X =O)t =[e-
3
!4J =e-
3
.
Q.7. Given P(A)=0.5and P(AUB)=O.6,find P(B)if
i. A and B are mutually exclusive.
..
ii. A and B are independent.
Sol. We know P(AUB) = P(A)+P(B)-p(AnB) ...(1)
(a) If A and B are mutually exclusive,
128
AnB=0
=> p(AnB)=p(0)=0
:. (1) reduces to
P(AUB) =P(A)+P(B)
=> P(B)=P(AUB)-P(A)=0.6-0.5=0.1
=> P(B) =0.1
(b) If A and Bare independent,then
p(AnB) = P(A)P(B)
:. (1) reduces to
P(AU B) = P(A)+ P(B)- P(A)P(B)
=> P(AUB) = P(A) +P(B)(l- P(A))
=> 0.6 = 0.5 + p( B)(1-0.5) =0.5 + p( B)(0.5)
=>
0.1=P(B)(0.5)
1
=> P(B)=-.
5
1
Q.8. let E and F be independent events.The probability that both E and F happen is - and the
12
1
probability that neither E nor F happen is- .Then find prE) and P(F).
2
1 - - 1
Sol. Given: peE nF) = - and peE nF) = .
12 2
Now, p(EnF)=!
=>
P(EUF)=!
2 2
=> 1-P(EUF)=-
1
=> P(EUF)=-.
1
2 2
We know P(EUF)=P(E)+P(F)-p(EnF) ..(1)
Since A and B are independent events,then
p(EnF) =P(E)P(F)
129
1
=> P{E)P{F)=12
... (2)
:. (1) reduces to
P(EUF) = P(E)+ P{F)-p(EnF)
1 1
=>
-=P{E)+P{F)-
2 12
1 1 7
=>
P(E)+P(F)=2+ 12 = 12
...(3)
(2) and (3), => p( E) and p( F) are the roots of the equation
2 7 1
x --x+-=O
12 12
=>
12x2 - 7x +1=0
=>
12x2 -4x-3x+l=O
=>
3x(4x-l)-I(4x-l) =0
=>
(3x-l)( 4x-l) =0
1 1
=>
x=- x=
3' 4
1 1 1 1
.. P(E)=-,P{F)=- or P(E)=-,P(F)=- .
3 4 4 3
0.9. If X is a Poisson variate such that p(X = 2) =9P(X =4)+90P(X =6). Find
E(X) and P{X <2).
Sol. We have P(X=2)=9P(X=4)+90P(X=6) ...(1)
P(X = x) = e-A.It:'
We know ...(2)
x!
Using (2) in (1),we get
130
e-A-A? e-A-J4 e-A-J6
--=9--+90-
2! 4! 6!
1 J2 J4
=>
-=9-+90
2! 4! 6!
Simplifying the above equation, we get
J4 +3J
2
-4=0
=> (J
2
+4)(J
2
-1)=0
=> J=1 ::::::> J=1 (':J>O)
=> E(X)=J=l.
Now, P(X<2)=P(X=0)+P(X=I)
e-
I
(1)0 e-
I
(1)1 _ 2 -1
- + - e .
- O! I!
a.l0. Ten percent of screws produced in a certain factory turn out to be defective. Find the
probability that in a sample of 10 screws chosen at random, exactly two will be defective.
Sol. Let X be the number of defective screws in a sample of 10 screws chosen at random.Then
p{probability or success) =10% =J... and q{probability of failure) = I-J... = 9 ,
. 10 10 10
Probability of x defective screws in sample of 10 screws is
P(X = x) =nCxpxqn-x
Probability of 2 defective screws in sample of 10 screws is
P(X = 2) =lOC
2
( 1 ~ J(:0 )10-2
1)2(9)8
1
(9)9
=45
(
10 10 ="2' 10 =0.1937.
a.11. Using Poisson distribution ,find the probability that ace of spades will be drawn from a
pack of well-shuffled cards at least once n 104 consecutive trials.
131
1
Sol. Probability ofthe ace of spades = p = ,n =104 .
52
1
=>
A=np=I04.-=2.
52
Probability of occurring of x ,ace of spades in 104 trials is
2 x
P(X =x) =e- 2
xl
Required probability=P(X =1)+P(X =2)+... +P(X =104)=1-P(X =0)
P( X = 0) = e-22 _ -2
Now,
O! -e
:. Required probability =1-P(X =0) =1-e-
2
=1-(2.718t =0.865.
Q.12. In a sample of 1000 cases, the mean of a certain test is 14 and standard deviation is
2.5.Assuming the distribution to be normal, find how many students score between 12
and 15?
Sol. Here n =1000,,ll =14,0' =2.5 .
X -,ll
If X is the normal variate,then Z =--is the standard normal variate.
0'
If X denotes the marks of students in a certain test,then
P(12::;; X::;; 15) =p(12-,ll ::;;::;; 15 -,ll) = p(12-14 < X -14 < 15-14)
0' 0' 2.5 2.5 - 2.5
= P(-0.8::;; Z::;; 0.4)= P( -0.8::;; Z ::;;0)+ P(O::;;Z ::;;0.4)
= 0.2881 + 0.1554 =0.4435 (from the normal table)
The required number of students = 1000 x 0.4435 = 443.5 =444.
Q.13. A group of scientific men reported 1705 sons and 1527 daughters. Do these figures
1
conform to the hypotheses that the sex ratio is - .
2
Sol. He number of sons is 1705.
The total number of observations = 1705 + 1527 = 3232 .
132
1705
The observed male ratio --= 0.5175.
3232
1
In the given hypotheses male ratio is - = 0.5 .
2
Thus,the and theoretical ratio
=0.5275 -0.5 =0.0275
1 1
Here p=-,q=-,n=3232
2 2
[il
l
" The standard deviation of the proportion is s = Ipq = 2"' 2" =0.0088 .
n 3232
The difference is more than three times of standard deviation.
Hence,it can be definitely said that the figures do not conform to the given hypotheses.
Long answer type questions
Q.1. A bag contains 6 white and 9 black balls. Four balls are drawn at a time. Find the
probability for the first draw to give 4 white and the second to give 4 black balls in each
of the following cases:
(1) The balls are not replaced before the second draw.
(2) The balls are replaced before the second draw.
Sol:. (1) We can draw 4 balls from a bag containing 6 white and 9black balls in 15 C
4
ways.
Let A be the event that the first draw gives 4 white balls and B be the event that the
second draw gives 4 black balls. We can draw 4 white balls out of 6 white balls in 6C
4
ways.
133
6C
P[A]= 15C
4
4
Now if the drawn balls are not replaced, we can draw the other 4 balls in II C
4
ways. The
event B that the second draw result in 4 black balls (on the assumption that the first
draw has given 4 white balls) has 9 C
4
favorable cases.
9C
P[B/A] =IIC4
4
6C 9C 3
Hence p( AB} =p( A)xP( AI B} = x IIC: =715
6C
(2) We have
C
4
Since the balls are replaced after the first draw, the probability of drawing 4 black
9C
balls in the second draw is
C
4
. p( AB) =p( A).P(B}, as A and B are independent events.
6 9C 3
4
Hence P(AB}= lSCC X lsd =2963'
4 4
Q.2. Three urns AI' A , contain respectively 3 red, 4 white, I blue; Ired, 2 white, 3 blue; 4
2
red, 3 white, 2 blue balls. One urn is chosen at random and a ball is withdrawn. It is
found to be red. Find the probability that it came from urn and .
Sol. If denotes the ith urn chosen and R denotes the event of withdrawing the red D1 ball,
then
Now P(RI
By Baye's theorem,we have
AI)
P I R) -
-
134

- =0.38028

P( I R) -
-

=0.1690

P( I R) -
-
=
=0.4507
Q.3. In a Normal distribution 7% of the .itemsare under 35 and 89 are under 63.What are the
mean and standard deviation of the distribution.
Sol. If X N(.u, (J'2) I then we are given
P(X < 63) =0.86and P(X < 35) =0.07
=:} P(.u < X < 63) = 0.86-0.5 = 0.39 and
P(35 < X <.u) = 0.5 -0.07 = 0.43
X - .u 63 - .u)
=:} P 0 < -;;:- < (J' = 0.39 and
(
< X (J' < 0) =0.43
135
63
::::;..
= 1.23 and
35 - P = 1.48 (from Normal table)
(J' (J'
Adding above two equations /we get
28 = 2.71
(J'
28
::::;..
(J'=-=10.33
2.71
and P = 35 +10.33(J' = 35 +1.48.10.33 = 50.3
Q.4. If X is Normally distributed with mean 11 and standard deviation 1.S/find the number
xosuch that
(a) p(X >x
o
)=O.3 (b) P(X > x
o
) =0.09.
Sol. Given: p = 11, (J' =1.5
(a) We have P(X > x
o
) =0.3
::::;.. 0.5-P(11<X <x
o
)=O.3
::::;.. p(l1<X <x
o
)=0.5-0.3=0.2
p(
l1- P X-p xO- P)-02
::::;.. --<--<-- - .
(J' (J' (J'
::::;.. p(0<X-11<X
o
-11)=0.2
1.5 1.5
X -11)
::::;.. P 0 < Z < 0 = 0.2 ,where Zis the Standard normal variate.
(
1.5
-11
::::;.. (from Normal table)
15=0.52
::::;.. Xo =11.78
(b) We have p( X > xo) =0.09
::::;.. 0.5 - P(11 < X < Xo) = 0.09
136
=> P(l1<X <x
o
)=0.5-0.09=0.41
=> p(l1-.u < x
(1 (1
< X
o
-
li
)=0.41
(1
=> p(O< X -Ii < X
o
-11) =0.41
1.5 1.5
=> P 0 < Z < 0 = 0.41, where Z is the Standard normal variate.
(
X -11)
1.5
=>
-11 =1.34
1.5
(from Normal table)
=> Xo = 13.01.
Q.5. Compute the student's t for the following values in a sample of eight:
-4,-2,-2,0,2,2,3,3 taking the mean of universe to be zero.
Sol. Given: Ii =0 .
S.No. x
I - 1
x-x=x-
4
(X_X)2 = ( x - ~ J
1
-4 17
-
4
289
16
2 -2 9
-
4
81
-
16
137
--
--
3 -2 9 81
-
4 16
4 0
1 1
-
4 16
I
2 5 7 49
-
4 16
6 2 7 49
-
4 16
I
7 3 11 121
- -
4 16
3 8 11 121
- -
4 16
I
n=8
I:X=2
2)x-xf= 792
I
16
I
__ LX 2 1
x--=-=
n 8 4
J ' ~ ) x - x ) ' =t
92
=../7.07 =2.66
S.D.=s = n-1 16.7
1
-0
=_4_____ 2.83
In 2.66 - 10.64 = 0.266 .
Q.6. In a certain factory turning out razor blades, there is a small chance 1/500 for any blade
to be defective. The blades are supplied in packets of 10 .Use Poisson's distribution to
calculate the approximate number of packets containing no defective, one defective and
02
two defective blades respectively-in a consignment of 10000 packets. (e
4l
= 0.9802)
Sol. Here p =1/5oo,n =lO,B =lOOOO.
The average number of defective blades is A=np =_l_. lO =2.- =0.02
500 50
Probability that packets contains x defective blades is
138
P(X =x)= e-AX" = e-{)02(0.02f
xl x!
Number of packets containing x defective blades
= N.P( X = x) = e-{)02 (0.02f = (0.9802)(0.02f
xl xl
Number of packets containing 0 defective blades
(
0.9802) (0.02)0
=N.P(X=O)=loooo. 9802
O!
Number of packets containing 1 defective blades
(0.9802)(0.02l
=N.P(X =1 ) =1. 0000 =196
I!
Number of packets containing 2 defective blades
(
0.9802) (0.02)2
=N.P(X =2)=10000. =1.9604=2
21
Q.7. Given the bivariate data:
X: 1 5 3 2 1 1 7 3
Y: 6 1 o o 1 2 1 5
Fit a regression line of X on Y and then predict X if Y=2.S.
Sol.
X
d
x
=X-3
d 2 Y
X
1 -2
4 6
5 2 4 1
1
3 0 0 0
_. ,
d
y
=Y-2
4
-1
-2
d 2
y
16
1
4
139
--
1
0
4 -2 ,2 I -1
4 1 1 1 . -1
1-
2
I
4 2 1 0 0
l -2
16
1. - ~ 7
/4
1
! -1
0 0 5 3 3 9
I
Ld
x
=-1 Ld
x
2
=33 Ld/=36
LY=16 I Ld
y
=0
ILX=23
I
- LX = 23 =2.875
X = ~ - - 8
n
- _LY =16 =2
Y
- n 8
Regression Equation of X on Y :
X - X = b
XY
(Y - f) ... (1)
_ n Ldxd
y
- Ld
x
Ld
y
Where,
b
XY
- " 2 (" )2
nL.Jdy - L.Jdy
8.(-10}-(-2)(0} = _ ~ = -0.278
b
XY
= 8(36)-(O}2 288
Substititing these values in (l),we get
X -2.875 =-O.278(Y-2)
=> X =3.431-0.278Y
If Y =2.5 ,then
X =3.431-0.278(2.5)=2.736.
Q.8. There are 6 positive and 8 negative numbers. Four numbers are chosen at random,
without replacement/and multiplied. What is the probability that the product is a
positive number?
140
Sol. To get from the product of four number, the possible combinations are as follows:
S.no Out of 6 positive
numbers
Out of 8 negative
numbers
Positive numbers
-1. 4 0
6 8 6xS
C
4
x Co ==--xl==IS
lx2 I
i
2.
!
I
: 3.
I
I
I
2
0
2
4
6C x
8
C == 6xS x 8x7 == 420
2 2 lx2 lx2
6C x
8
C
4
== Ix 8x7x6xS == 70
o lx2x3x4
Total =505
I
6C 8 C 6 8
6 8
Probability == 4 X 0 + C2 X C2 + Co x C
14 4
C
4
IS+420+ 70 SOSx4x3x2xl SOS,
== 14x13x12xll == 14x13x12xll == 1001
lx2x3x4
Q.9. A skilled typist,on routine work, kept a record of mistakes made per day during 300
working days.
Mistakes 0 1 2 3 4 5 6
I per day
No of 143 90 42 12
:9
3 1
days
I
Fit a poisson distribution to the above data and hence calculate the theoretical
frequencies.
Sol. The mean number of mistakes
141
1
= -(143xO+90x1 + 42x2+12x3+9x4+3x5+1x6)
300 .
1 267
=-(90+84+36+36+15+6) =-=0.89
300 300
: Number of Theoretical frequency
eO.
89
X (0.89 r
I
Probability P ( r )
Imistakes
r!
1
0 0.411x300 = 123.3 = 123
eO.
89
X(0.89t
= 0.411
O!
I
1
eO.
89
X(0.89)1
I 0.365x300=109.5=110
=0.365
11
2
eO.
89
x(0.89)2
0.163x300 = 48.9 z 49
=0.163
2!
3
eO.
89
X (0.89 f
0.048x300 = 14.4 z 14
. =0.048
i 3!
:4 0.011x300 = 3.3 "'" 3
:
---'--"--' = 0.011
I
4!
5
! eO.
89
X (0.89 )5 = 0.002
0.002x300 = 0.6 "'" 1
5!
6
eO.
89
X(0.89)6 =0.0003
0.0003x300 =0.09 z 0
I
6!
Q.10. If a random variable.X has a binomial distribution, then show that
E(X) = np, var(X) = npq
Sol. We have
E(X)
n n
L.if(x) =Lx.nCtpxqn-x
x=o x=o
n en-I 2 n C 2 n-2 n C n
= Ipq +. 2P q + ... +n. nP
142
= np(
pq
n-l +(n-l) p2
q
n-2 +... + pn-l)
n-l
=np(q+p) =np (':p+q=l)
E(X)=np
n n
E(X2)= :l>2f(x) = L(x(x-l)+xf

II n
= Lx(x-
1
rCxpxqn-x + LX.nCx:pXqll-X
x=1 x=0
=2.1
n
C
2p
2qn-2 +3.2
n
C
3p
3qn-3 +...+n(n-lr Cllpn + E(X)
=n(n-l)p2 [qll-2 +(n-2) pqn-3 +... + pn-2J+np
=n(n-l) p2 (q+ Pr-
2
+np = n(n-l) p2 +np
Now, var(X)=E(X)2 _(E(X)}2
=n(n-l) p2 +np_(np)2 =np-np2 =np(l- p) =npq
Var(X) =npq.
Q.ll. The life of army shoes is 'normally distributed with mean 8 months and standard
deviation 2 months. If 5000 pairs are issued how many pairs would be expected to need
replacement after 12 months?(Given that P(Z > 2) = 0.0228).
Sol. Given:
Mean = 11 =8 , standard deviation =(1 = 2
Number of pair of shoes =5000
Tofind:P(X >12)
p(X > 12) =p( X > =p( X;8 > 12;8) =P(X >12)
=p( X >
143
L
12-8) X-/1
= P Z >-2- I where Z =----;- is the standard normal variate
(
=P ( Z > 2) = 0.0228
Nurriberofpafrs ofshoes whose life is morethan 1imonths
=5000xP(X >12)=5000xO.0228=114
Number of pairs of shoes that need replacement after 12 months is
5000 -114 = 4886 .
Q.12. Calculate Karl Pearson's coefficient of correlation from the following data:
6 8 12 15 18 20 24 28 31
Y 10 12 15 15 18 25 22 26 28
Sol.
IX
x=X-18
x
2 y
y=y
l
xy
6
-12
144 10
-9
81
+108
8
-10
100 12
-7
49
+70
12
-6
36 15
-4
16
.
+24
,.
144
15 15 16 -3
9
-4 +12
18 0 0 18 1
-1 0
I I
2 4 25 36
+6 +12
\ 20
I
g--.
24 6 36 22
+3 +18
28 10 100 26 49
+7 +70
31 13 ! 169 28 +117 81
+9
I
,LX =162 ,Ly=o. ,Ll=338! ,Lxy=431
LX =0 I Lx' =59!
,LY=17
I _ ~ ... _ ..... L __I
i
r= ,LXY
~ , L x 2 , L l
,Lxy=431,,Lx
2
=598,,Ll =338
431 431
r = 'JF=59=8=x3=3=8
449.582 =0.959.
Q.13. Given the following data:
Variance of X is 9
Regression equations: 4X -5Y+33 =0;20X -9Y -107 =O.
Find (i) the mean values of X and Y
(ii) the coefficient of correlation between X and Y
(iii) the standard deviation of Y.
Sol. We have 4X -5Y +33 =0 ...(1)
20X -9Y -107 =0 ...(2)
145
(i) Since the two regression lines always passes through the point (X, r) ,the mean
values of X and Y is given by solving the given regression equations.
Solving 4X -5Y +33 =0;20X -9Y -107 =0 ,for the values of X and Y ,we get
X =13,Y=17.
(ii) Let (1) be the regression equation of Yon X and (2) be of X on Y.Then
(1) can be written as
4 33
Y==-X+
5 5
4
::::> byx = - =0.8
5
(2) can be written as
X ~ y + 1 0 7
20 20
9
::::> bxy == = 0.45
20
:. I the coefficient of correlation is given by
r=,Jbxybyx
::::> r:::; .J0.45xO.8 0.6
O'y
(iii) We know that b =r
yx O'x
::::>
0.8=0.6 O'y
3
::::> O'y =4.
Q.14. Suppose that an instructor assumes that a student's final score is the value of a normally
distributed random variable. If the instructor decides to award a grade of A to those
students whose score exceeds 11 +0' ,B to those students whose score falls between
11 and 11 +0' ,a C if a score falls between 11- 0' and 11 and a 0 if a score falls between
11- 20' and 11- 0' .Find the percentage of students of each grade.
146
~ o l . I-or grace A, we nave
P(X >.u+0")=1-P(X '5:.u+0")
X -.u )
=1-P --0:-'5:1
(
!
=1-P(Z'5:1),
(where Z =X .u is the standard normal variate)
0"
= 1-(0.5+ p(O '5: Z '5: 1)) =0.5-0.3413 =0.1587
Hence the percentage of students to receive grade A is 15.87%.
For grade B,we have
p(.u< X <.u+O")=p(O< X ~ . u <1)
=P(0<Z<1),
X-.u
(where Z = --is the standard normal variate)
0"
=0.3413
Hence the percentage of students to receive grade B is 34.13%.
For grade C,we have
P(.u-O"<X <.u)=p( -1< X ~ . u <0)
=P(-1 < Z <0) =p(O<Z <I),
X-.u
(where Z =--is the standard normal variate)
0"
=0.3413
Hence the percentage of students to receive grade C is 34.13%.
147
For grade D,we have
P(,u-2a< X < ,u-a) =p(-2< X~ , u < -lJ
=P(-2<Z <-l)=P(l<Z<2),
X-,u
(where Z = --is the standard normal variate)
a
=p(O < Z < 2)- P(O < Z < 1) =0.4772-0.3413 =0.1359
Hence the percentage of students to receive grade 0 is 13.59%.
148
Unit - 6 : Numerical Methods
Multiple Choice Type Questions:
Q.1. The iteration formula for the Newton Raphson method is
f(xn)
(a) (b)
Xn+l = X + f (xn )
xn+l =xn - f' (xn )
n f'(xn)
f'(x
n
)
(c) (d)
Xn+l =X + f'(x
n
)
xn+l =xn - f (xn )
n
f(xn)
Q.2. The Newton Raphson iteration formula for finding the value of :ifN, N E is
2
X
3_N
_ n 2 x ~ + N
(a) (b)
xn+! - 3X2 xn+! = 3X2
n n
x _ 3X3 +N
x _ 3
X
3_N
(c) n+l - _n_ (d)
n+! n
2X2 2X2
n n
Q.3. If f is a continous function and the roots of the equation f (x) =0 lies between a and b then
(a) f(a).f(b)<O (b)
f(a).f(bO
(c) f (a).f (b) = 0 (d) None of the above
Q.4. The real root of the equation f (x) =0 is the abscissa where the curve cuts the
(a) x-axis (b) y-axis
(c) neither (a) nor (b) (d) both (a) and (b)
Q.5. The iteration method for solving an equation gives only root(s) at a time
(a) 1 (b) 2 (c) 3 (d) None of the above
Q.6. The rate of convergence of Newton Raphson method is
(a) 1 (b) 2 (c) 2.5 (d) 3
Q.7. Starting with Xo = 3,a root of X3 - 3x - 5 = 0 after first iteration is
(a) 1.4536 (b) 2.1567 (c) 2.4583 (d) 0.5425
Q.8. A root of the equation X3 2x - 5 = 0 may lie in the interval
(a) (0,1) (b) (1,2) (c) (2,3) (d) (2.5,3)
Q.9. A root of the equation can not lie on the interval
(a) (0,1) (b) (0.5,1) (cl (0,2) (d) (1,2)
149
L
Q.10. If a root of the equation j (x) =0 lies in the interval (a, b) ,then an approximate value of this
equation ,using Regula Falsi method is given by
(a)
aj(b)+bj(a)
x = ----'-'------':-'
j(b)- j(a)
(b)
aj(b)-bj(a)
x = --....:........'-'------:.-'
j(b)- j(a)
(c)
aj(b)-bj(ar
x = - - - - ' : ~ ~ - ' -
j(b)+ j(a)
(d)
iiJ(b)+bj(a)
x=
j(b)+ j(a)
Q.11. Newton Raphson method is also called
(a) Successive Substitution method (b) Successive Approximation method
(c) Successive Distribution method (d) None of the above
Q.12. Given the data x: 3 4 5
y: 9 16 25
The value of d
Y
! is approximately equal to
dx =3
(a) 4.0215 (b) 5.0006 (c) 6.1602 (d) 7
Q.13. An integral is approximated using Numerical integration if
the integrand function does not posses a usable antiderivative function
the integrand function is not known exactly but is represented approximately
by a set of data of points.
(a) only (i) is true (b) only (ii) is true
(c) both (i) and (ii) are true (d) neither 0) is true nor Oi) is true
Q.14.
f! ydx = !:[Yo +4(YI + Y3)+2Y2 + Y4]
J.<o 3
The above rule for finding integral is called
l)rd
(a) Trapezoidal rule (b) Simpson's "3 rule
(
3)th
(c) Simpson's '8 rule
(d) None of the above
(
Q.15. If the rate of convergence of Newton-Raphson method and False position method are rand 5
respectively,then
(a) r<s (b) r>s (c) r =s (d) r'5:s
150
Q.16. 'An equation that can be solved using Newton-Raphson method may not be solved by False
position method'This statement is
(a) always true (b) always false
(c) partially true (d) partially false
Q.17. The negative root ofthe equation can be determined using
(a) Newton-Raphson method (b) False position method
(c) both (a) and (b) (d) neither (a) nor (b)
Q.18. The Numerical differentiation formula can be derived by diffentiating polynomial
(a) linear (b) quadratic (c) cubic (d) interpolating
Q.19. The numerical method for solving an equation gives an
(a) exact root
(b) approximated root
(c) approximated root that converges to the exact solution as number of the
iterations tends to infinity
(d) approximated root that do not converge to the exact solution as number of the
iterations tends to infinity
Key
1. (a) 2. (b) 3. (a) 4. (a) 5. (a) 6. (b)
7. (c) 8. (c) 9. (d) 10. (b) 11. (a) 12. (c)
13. (c) 14. (b) 15. (b) 16. (b) 17. (c) 18. (d)
19. (c)
151
True/False Type Questions:
1. In Newton-Raphson method/we approximate the graph of function by suitable tangents.
2. Piecewise quadratic approximation of a function led to the Trapezoidal rule.
1),d
3. Piecewise quadratic function led to the Simpson's 3" rule.
(
4. Numerical differentiation is the computation of values of the derivative of a fUnction which is
represented approximately by a set of data points.
5. Newton-Raphson method is used for solving algebraic equations but not transcendental
equations.
6. The method of False position can be represented graphicany.
(
1)'d
7. Simpson's 3" rule requires the division of the whole range into an odd number of
subintervals of equal length.
3
8. A real root of the equation x - x -1 = 0 lies between 1 and 2.
9. The rate of convergence of Newton-Raphson method is greater than False position method.
10. The truncation error in Numerical differentiation is caused by replacing the tabulated
function by means of an interpolating polynomial.
Key
1. T 2. F 3. F 4. T s. F 6.T
7. F . 8. T 9. T 10. F
152
Short Answer Type Questions:
2 2
Q.l. Given that the equation X . - 69 has a root between Sand 8. Use the method of regula
falsi to determine it.
Sol. Let I(X)=X
2
.
2
-69.
=> f( 5) =-34.50675840 and 1(8) =28.00586026:
Since I (5) 1(8) < 0 ,the root lies between Sand 8.
Let Xl =5 and =8 x
2
The regula falsi iteration formula is
XJ (X2) -
x
2
1 (Xl)
X3 =
I(X2)- I ( ~ )
5(28.00586026) -8(-34.50675846)
=>
X = =6.655990062.
3 28.00586026 +34.50675846
Now, I (X:J) =-4.275625415 and therefore, the root lies between 6.655990062 and
8.0. We obtain
Now,taking ~ =6.655990062 and =8 ,we get x
2
(6.655990062) (28.00586026) -8(-4.275625415)
X = = 6.83400179
3 28.00586026 +-4.275625415
Proceeding in the same manner, we get next approximate root as 6.850669653.
Q.2. Use the Newton- Raphson method to find a root of the equation X3 - 2x - 5 =0 .
Sol. Here I(x) =X3 -2x-5 and f'(x) =3X2 -2.
By Newton-Raphson Method, X+1 =X - IiXn)
n n I I Xn)
X3 -2
=>
xn+l =X
n
_ n xn - 5
... (1)
3x2n - 2
Choosing Xo =2, we obtain I (xo) =-1 and f' (xo) =10.
Putting n =0 in (1), we obtain
Xl =2 - ( - 1 ~ ) =2.1
153
Now,
f (Xl) = (2.1)3 - 2(2.1) -5 = 0.061,
and
f = 3(2.1/ -2 = 11.23.
Hence
X = 2.1- 0.061 = 2.094568.
2 11.23
Q.3. Find a real root of the equation X = e -x using the Newton- Raphson method.
Sol. We have X = e-
x

:::;> xe
x
= 1
Let f ( X ) = xe
x
-1 = 0
:::;> f' ( x) = ( X + 1) eX
Then
f(x
n
)
xn+l = xn - f! (x
n
) = xn
Let Xo == 1.
Xl = e +
1
= 0.6839397
2e
X eX. +e
xn
n
:::;> x
2
+1
(Xl + = 0.5774545 ..
Proceeding in this way. we obtain
=0.5672297 and x
4
=0.5671433.
Q.4. Solve graphically the equation x-I = sin x.
Sol. x-I == sinx
X 2e
x
+ 1
n
(xn + 1) eX.
We take two equations y = X -1 and y = sin x.
Let us find out the abscissa of the point of intersection of the line y = X -1 and the
curve y == sin X and give a rough estimate of the root.
154
x
For the straight line y = x -1, we have the table:
x 3Jr 14
~ 1
y=x-1 1.4
~ o
For the curve y =sin x t we have the following table:
x 0 Jrl4 Jr 12 3Jrl4 Jr
Y =sinx
0
,
0.71 1.00 0.71 0
On the same axes
t
and with the same scale constructing the graphs of y =x -1 and
y =sin x twe get x =1.95 radians approximately.
155
2
Q.5. Determine the root of X4 +x
3
-7x - x +5 = 0 which lies between 2 and 3 correct to
three decimal places using Newton -Raphson method.
Sol. We have I(x) =X4 +X3 -7x
2
-x+5 =0
=> 1 (2) =16 + 8 - 28 - 2 + 5 =-1
and 1 (3) =81 +27 -63-3+5 =+47.
Since 1 (2) 1 (3) < O,the root lies between 2 and 3.
Now, f'(x) =4x
3
+3x
2
-14x-l

Taking Xi =2 as first approximate root,we have
I(Xi) =2- 1(2)
x
2
=Xi
f'(Xi) 1'(2)
(-1) 1
=>
x = 2--= 2+- = 2.067 [..' 1 '(2) =32+12-28-1 =15J
2 15 15
and
x =x_I (x
2
) =2.067 _ _1--,-(2_.0_67--,-)
3 2 f'(x
2
) 1'(2.067)
= 2.067 - (-0.0028) = 2.067 +0.0001519 = 2.0671519.
18.422
Q.6. Write the Newton-Raphson procedure for finding ifN, where N is a real number. Use it
to find 3 J18 correct to 2 decimals, assuming 2.5 as the initial approximation.
3 3
Sol. Let x = ifN => x = N or x - N = 0
Let l(x)=x
3
-N=0.
=> I'(x) =3x
2
By Newton-Raphson Method, x +1 =X _ 1 (xn )
n n f'(x
n
)
X 3 2 3+N 2
n - N _ xn " n =0 1, , ....
=> xn+1 =xn
~ - 3x '
n n
Let N =18.
We shall find the root of 1(x) = x
3
-18 = 0 ,taking 2.5 as the initial approximation.
So, take Xo = 2.5 .
156
2X03 + 18 _ 2(2.5f + 18 =2.62667
=>
.x; = 3X02 - 3x(2.5f
2.x;3 + 18 2(2.62667)3 + 18 =2.620755 .
=>
2
x = 3x/ = 3(2.62667)2 _
Q.7. Find an approximate value of the root of the equation X3 +x-I =0 near x =1, using
the method of false position two times.
Sol. We have f (x) =X3 + x-I =0
f(I)=I+I-I=+1
f (0.5) =(0.5)3 +(0.5) -1 = -0.375
Since f (0.5) f (1) < O,the root lies between a.Sand 1.
Let Xl =0.5 and x
2
=1
.x;f (x2)-xd (.x;)
X3 =
f(x
2
)- f(x
1
)
x = 0.5f(I)-lf(0.5) = 0.5(1)-1(-0.375) =064
=>
3 f(I)- f(0.5) 1+0.375 .
Now f (0.64) = -D.0979 and f (1) = 1
:. Root lies between .64 and 1.
So ,let Xl = 0.64, x
2
= 1
0.64f (1) -1.f (0.64) 0.64(1) -1.(-0.0979)
X = =0.672
3 f(l) f(0.64) 1+0.0979
157
Q.8. From the following table of values of x and Y, obtain dy for x =2 .
ax
x: 2 3 4 5
y: 8 27 64 125
501. The difference table is
:x
Y


2 8
19
3 27 18
37 6
4 64 24
61
5 125
I
I
We know that,
d
Y
] 1 ( 1 2 1 3 1 4 J
[
ax x=Xo =h Yo Yo Yo +...
Here, h = 1, Xo =2, Yo = 8 .
[
d
Y
] 1 ( 1 1)
- =- 19--.18+-.6 =19-9+2=12.
ax x=2 1 2 3
Q.9. From the following table of values of x and y,obtain dy for x 0.30 .
ax
Ix
0.10 0.15 0.20 0.25 0.30
Iy
0.1003 0.1511 0.2027 0.2553 0.3093
Sol. The difference table is
x
0.10
0.15
y
0.1003
0.1511

0.0508
0.0516
,

0.0008
-

0.0002

158
0.2027 0.0010 0.20 0.0002
0.0526 0.0004
0.2553 0.0014 0.25
!
0.0540
0.30 0.3093
i
We know that,
[
d
Y
] =!"(VYn +!..V2Yn +'!'V
3
Yn + !..V
4
Y
n
+ ... )
dx h 2 3 4 X=X
n
Here, h = O.OS,xn = 0.30, Y
n
= 0.3093
[
d
Y
] =_1_(0.OS40+!..(0.0014)+.!.(0.0004)+!..(0.0002))
dx x=O.30 0.05 2 3 4
= 1.097666.
Q.l0. Evaluate I = ~ ~ , correct to three decimal places using trapezoidal rule with
.b l+x
h=O.S.
Sol. For h =0.5: the values of x and yare tabulated below:
! X
Y
I 0.0
1.0000
I 0.5
0.6667
1
1
.
0 0.5000
Trapezoidal rule gives
Cydx= ~ [ Y o +2(Yl + Yz + ... + Yn-l)+ Yn]
~ 1=!..[1.0000+2(0.6667) + 0.5] = 0.70835.
4
159
Long Answer Type Questions:
3
Q.l. Find a real root of the equation f (x) = x - 2x- 5 = O. using method of False position.
3
Sol. We have f (x) =x - 2x - 5 =O.
~ f (2) =-1 and f (3) =If) .
Hence a root lies between 2 and 3.
let .x; = 2 and x
2
= 3
The regula falsi iteration formula is
.x;f (X2) - xzf (.x;)
X3 f(x
2
)-f(.x;)
2(16)-3(-1) = 35 = 2.058823529.
~
X3 = 16-(-1) 17
Now, f (X3) =-0.390799917 1 it follows that the root lies between 2.08126366 and
3.0. Hence, we have
2.08126366(16)-3-(0.147204057) = 2.089639211.
X3 = 16.147204057
Proceeding in this way, we obtain successively:
X
4
=2.092739575, Xs =2.09388371,
X6 = 2.094305452, X; = 2.094460846, ...
The correct value is 2.0945"'1 so that is correct to five significant figures. x
7
Q.2. Find a root of the equation X sin x + cos x =0 using Newton Raphson method.
Sol. We have
f (x) =xsinx+cosx
~ f'(x) =xcosx.
The iteration formula is therefore
xn sin xn + cos xn
xn+1 =x
n

xn cosx
n
With Xo = 1[, the successive iterates are given below
160
n . xn f(x
ll
)
Xn+l
0 3.1416 -1.0 2.8233
1 2.8233 -0.0662 2.7986
2 2.7986 -0.0006 2.7984
-
-
:3
2.7984 0.0 2.7984
Q.3. Find, from the following table, the area bounded by the curves and the x-axis from
x = 7.47 to x = 7.52 .
x
7.47
.7.48
7.49
f(x)
1.93
1.95
1.98
x
7.50
7.51
7.52
f(x)
2.01
2.03
2.06
Sol. We know that
r
.52 ( rrf.52
Required Area= f x)dx= ydx
.47 .47
We are given h=O.Ol,
x
7.47
7.48
7.49
Y
1.93
1.95
1.98
x
7.50
7.51
7.52
Y
2.01
2.03
2.06
The trapezoidal rule gives
In Y d x = ~ [ Y o + 2(Yl + Y2 + ... + YII-J)+ YIIJ
"52 h J
=> ydx=-[Yo+2(Yl+Y2+"'+Y4)+Y5
r
.47 2
= 0.01 [1.93+2(1.95+ 1.98+2.01+ 2.03)+2.06J
2
161
=0.0996
Area= 0.0996 .
Q.4. Derive Simpson's 1/3- rule using the method of undetermined coefficients.
Sol. We assume the formula
+--,:
fhydx=a-IY-I +aoYo +a1Yl'
...(1)
where the coefficients a_I' a
o
and a
l
have to be determined.
For this, we assume that formula (1) is exact when Y(x) = 1, x and x
2

Putting therefore Y(x) = 1, x and x
2
successively in (i), we obtain the relations
and
a_I +aO+a
1
= fh
dx
=2h,
-a_I +a
l
= fhXdx=O
a_I +a
1
="3
2
h
.
Solving (2),(3) and (4) for a_pa
o
and ~ , we obtain
2 4h
a_I ="3=al and a
o
=3'
Hence formula (1) takes the form
fhYdx= ~ (Y-l +4yo +YI)'
which is the Simpson's 1/3- rule.
Q.S. By using Newton-Raphson's method, find the root of
x = 2 correct to three places of decimal.
Sol. Let l(x}=x
4
-x-1O=0.
::} f' (x) = 4x
3
-I,
1(2)=16-2-10=4
1'(2}=32-1=31
Take Xo =2.
By Newton- Raphson's method
162
... (2)
... (3)
...(4)
X4 - x -10 = 0 , which is near to
=xo- I(x
o
) =2- 1(2)
f'(x
o
) 1'(2) 31
1 (1.871) = (1.871)4 -1.871-10 = 12.25 -1.871-10 = 0.379
1'(1.870) = 4(1.871)3 -1-= 4x6.5497 -1 = 25.1988
1(1.871) 0.379
x
2
=1.871- ( ) =1.871- =1.871-0.0150=1.856
I' 1.871 25.1988
1 (1.856) = (1.856t -(1.856) -10 = 11.8662-11.856 = 0.0102
1'(1.856) = 4(1.856)3 -1 = 4x6.4623-1 = 24.8492
x =1.856- 1(1.856) =1.856- 0.0102 =1.856-0.00041=1.85559
3 1'(1.856) 24.8492
1 (1.85559) = (1.85559t -1.85559-10 = 11.855572-11.85559 = 0.00013
1'(1.85559) = 4(1.85559)3 -1 = 4x6.389193927 -1 = 24.55677571
x = 1.85559 1(1.85559) = 1.85559 0.00013
4 1 '(1.85559) 24.55677571
= 1.85559 - 0.00000529 = 1.85558471.
2
Q.6. From the following table of values of x and y,obtain dy and d ;, at x =1.2 .
dx dx
x 1.0 1.2 1.4 1.6 1.8 2.0 2.2
Y
2.7183 3.3201
--------
4.0552
-
4.9530 6.0496 7.3891 9.0250
Sol. The difference table is
x Y

1.0 2.7183
0.6018
1.2 3.3201 0.1333
0.7351 0.0294
1.4 4.0552 0.1627 0.0067
163
1.6 4.9530
0.8978
0.1988
0.0361
0.0080
0.0013
0.0001
1.8
2.0
6.0496
7.3891
1.0966
1.3395
1.6359
0.2429
0.2964
0.0441
0.0535
0.0094
0.0014
2.2 9.0250
Here, h = 0.2,x
o
= 1.2, Yo = 3.3201
Now,
d
Y
] 1 ( 1 2 1 3 1 4 )
[
dx x=Xo =h Yo Yo Yo +...
d
Y
] 1 ( 1 1 1 1 )
- =_. 0.7351--(0.1627)+-(0.0361)--0.0080+-(0.0014)
[
dx =1.2 0.2 2 3 4 5
=3.3205
Also,
-..I _ 4
2
11
d ] 1 (
23
[
dx' =. - h' <I. Yo -<I. Yo +12<I. Yo +...)
2y
-2
1 ) d ]
=--2
(
0.1627-0.0361+-(0.0080)--(0.0014)
11 5
[
dx x=1.2 (0.2) 12 6
= 3.318
Q.7. Calculate the first and second derivatives of the function tabulated in the preceding
example at the pOint x = 2.2 .
Sol. We use the table of differences of previous example.
Here xn =2.2, Y
n
=9.0250 andh=0.2.
dy ] 1 (v 1 V2 1V3 1 V4 )
- =- Y +- Y +- Y +- Y + ...
[
dx h n2 n3 n4 n
x=x"
164
[
d
Y
] = _1[1.6359 +!(0.2964) +.!.(0.0535) +!(0.OO94 ) +.!.(0.0014)]
dx x=Z.Z 0.2 2 3 4 5
=9.0228
Now,
dZy] 1 (V2 V3 11V4 5Vs )
[
dxz =11 Yn+ Yn+12 Yn+(j Yn+'"
X=Xn

= _1_[0.2964+0.0535 = 8.992
dx x=2.Z 0.04 12 6
= 7.3896.
a.8. A solid of revolution is formed by rotating about the x-axis the area between the I-axis,
the line x =0 and x = 1,and a curve through the points with the following coordinates:
x 0.00 0.25 0.50 0.75 1.00
Y
1.0000 0.9896 0.9589 0.9089 0.8415
Estimate the volume of the solid formed, giving the answer to three decimal places.
Sol. If V is the volume of the solid formed, then we know that
V=
1r
1
ldx
We shall find 1ldx using Simpson's rule. For this we need the values of l and these
are given by
,x
i
0.00 1.0000
0.25 0.9793
0.50 0.919
0.75 0.8261
1.00 0.7081
By, Simpson's rule,
r:n ydx +4(Yt + Ys + ... + Yn-l) +2(yz + Y
4
+ ... + Yn-2)+ YnJ
=>
165
ydx = 0.:5 [1.0000+ 4(0.9793+ 0.8261)+2(0.9195) +0.7081J
Ydx = 0.897834
V = Jl' ldx = Jl'(0.89783) 4= 2.8192.
Q.9. Evaluate 1= r_l_dx to three decimal places using Simpson's rule and taking
.b l+x
h =0.125.
Sol. Taking h = 0.125 ,the tabulated values of x and yare:
x y x y
0 1.0 0.625 0.6154
0.125 0.8889 0.750 0.5714
0.250 0.8000 0.875 0.5333
0.375 0.7273 1.0 0.5
i 0.5 0.667
We know that
BY,Simpson's rule,
r:
n
Ydx = +4(Yl + Ys + ... + Yn-I) + 2(Y2 + Y4 + ... + Yn-2) + YnJ
ydx = +4(Yl + Y3 + Ys + Y7)+ 2(Y2 + Y4 + Y6)+ YsJ

0.125[1.0+4(0.8889+0.7273+0.6154+0.5333)+2(0.800)+0.6667+0.5714)+0.5J
.bl+x 3
=0.6932.
The exact value of the given integral is loge 2 = 0.693147 .
Hence the value of given integral may be taken to be equal to 0.693,correct to three
decimal places.
166
Q.10. Determine the root of X4 +X3 -7X2 - x+5= 0 which lies between 2 and 3 correct to
three decimal places.
Sol. Let I(x) =X4 +X3 -7X2 -x+5.
=> 1(2)=-1,1(3)=47.
Since 1 (2) =-1 is nearer to zero than 1(3) =47 ,:. root is near to 2.
So, let's try at 2.1.
Now, 1(2.1)=0.7391
Since 1(2) 1(2.1) < 0,:. the root lies between 2 and 2.1.
Let XI =2;X2 =2.1.
By Regula falsi method, next approximation to the root is given by

X3 = I(X2)-
21 (2.1) -(2.1)1 (2) 2(0.739)-2.l(-1)
=>
X3 = =-'--0-.73.....:.9---=-( 1(2.1)- 1 (2)
=> =2.0576
=>
1(2.0576) = -0.0579
: ..now root lies between 2.1 and 2.0576.
Let Xl =2.0576;x
2
=2.1
By Regula falsi method, next approximation to the root is given by
(2.0576) 1 (2.1) -(2.1) 1 (2.0576)
= 1(2.1)-1(2.0576)
_ (2.0576)(0.739)-2.1(-0.0579)
- 0.739-(-0.0579)
=> =2.0607 .
=> 1(2.0607) =-0.0028
=> Root lies between 2.0607 and 2.1.
I
By Regula falsi method, next approximation to the root is given by
167
--
(2.0607) 1 (2.1)-(2.1) 1 (2.0607)
= 1(2.1)- 1(2.0607)
_ (2.0576) (0.739) - 2.1( -0.0028)
- 0.739-(-0.0028)
=> =2.0609
The root of the given equation correct to three decimal places is 2.0609.
Q.ll. Evaluate "3 eX dx using Simpson's rule.
Sol. Divide the interval (1.1.3) into 6 equal parts of width h =0.05.
The tabulated values of x and yare
x
1.00
1.05
y=e
x
2.718
2.8577
1.10 3.0042
1.15
1.20
1.25
1.30
3.1582
3.3201
3.4903
3.6693
By Simpson's rule,
C +4(Yl + Ys + ... + Yn-l)+2(Y2 + Y4 + ... + Yn-2)+ YnJ
"3 eX dx = [2.718+4(2.8577 +3.1582+3.49031) + 2(3.0042+ 3.3201) + 3.6693J
=0.9510167.
168
1
.30
Q.12. Evaluate tan x dx using Trapezoidal rule by taking h = 0.05 .
.10
Sol. The tabulated values of x and yare
X
~ .
0.10
0.15
0.20
y=tan x
0.1003
0.1511
0.2027
0.25
0.30
0.2553
0.3093
Trapezoidal rule gives
rXn ydx= h[yo +2(Yl + Y2 + ... + Yn-l) + Yn]
JXo 2
=> f).3O tanxdx = h [Yo +2(YI + Y2 + Y3) + Y4]
1
.
.b.10 2
30
0.05[( )]
=> tanxdx=- 0.1003+2 0.1511+0.2027+0.2553 +0.3093
.10 2
=0.040695
169
Old Question Paper
Advanced Engineering Mathematics
(AC/AA 1.1)
Date: 22It, July) 2008- .. Time: 2:00-PMto 5:00-PM . Max. Marks: 100
Assume any missing data If necessary.
MiM'V...
Choose the correct or best alternative In the following.
Each question carries two marks.
1. If Z= f (x+ct) + g (x-ct), then
(a)
Zit =Zxx
(c)
Zit -
_
C
2
Zxx
_ 2
(b)
Zt =Zx
(d)
Zxx - C Ztt
-1 X + Y
113 )''' '"
.
2. If U =Slfi
[113
112 1/2 I thenx-+
'"
y- IS equal to
x +y ax ~
1 1
(a) -cotu (c) --tanu
12 6
1 1
(b) tanu (d) -cotu
12 6
2 2
3. The partial differential equation of the equation 2z = x2 + 4- is
a b
dz dz dz dz
z=-+- z=x-+y
(a) (c)
dX dy dx dy
d
2
Z dz dz
z=--
2z=xa-+ Y
(b)
dXdy
(d)
x dy
4. The solution of (y - Z) P+ ( Z- x) q =x- Y is
(a) f(x+ y + Z)= xyz (c) f{x
2
+ l + Z2, x
2
lz
2
)= 0
2
(b) f{x
2
+ l + Z2)= xyz (d) f(x+ y+ Z)= x + l + Z2
f.l -1 0]
5. The rank of matrix 0 f.l -1 is 2, for f.l equal to
-1 0 f.l
f
(a) any row number (c) 1
(b) 3 (d) 2
170
I I i-x
6. The value of J J Jx dz dx dy is
o l o
(a) 4/35 (c) 8/35
(b) 3/35 (d) 6/35
7. If A is the area under the curve}'=sin x, above x-axis such thaLO::;; X::;;1l' 12, then the
area under the curve y = sin2x, 0::;; x ::;; 1l'/2 is
(a) A (c) Al2
(b) 2A (d) 1+A
8. The figure bounded by one arc of the wave y= sin x and x-axis is revolved about x-axis.
The volume of solid of revolution is
(a)
It
(c) 21l'
2
2
1t
(b) (d) 1t
2
2
9. A problem of statistics is given to three students A, Band C where charges of solving it
are .!.., ~ , and .!.., respectively. The probability that the problem will be solved is
244
3 29
(a) - (c)
32 32
1 3
(b) - (d)
32 4
10. If (J' = 2, x =5, the equation of normal distribution is
1 _(.>:-5) _ (x_5)2
(a) f(x)=-e 8 (c) f(x)=lte 8
It
(X_5)2
1 8
f(x) = 2e--8
(b) (d)
f(x) = 2J21t e
iilllt-"':1
Choose TruelFalse In the following.
Each question carries one mark.
1. Charpit's method is useful to solve a partial differential equation of first order and higher
degree
2. The property of normal curve is that the curve is symmetrical about y-axis. The mean, median
and mode coincide at the origin.
171
1
3. The modulus of each eigen value of matrix is any scalar.
4. r!(x)dx= O=> !(x) =0
5. Simpson's rule gives more accuracy than trapezoidal rule .
.6;iI"'6
Answer any five questions out of eight questions.
Each question carries five marks
If A and B are non singular matrices of the same order then prove that
(ABtl = B-
1
A-I
2. Show by means of an example that in matrices AB=O does not necessarily mean that either
A=O or B=O, where 0 stands for the null matrix.
3. Using the method of iteration, find a real root of the equation
2x-Iog
10
x = 7
which lies between 3.5 and 4
4. Expand!(x, y) =eX log, (1 + y) in a Taylor's series in the neighborhood of origin, retaining
terms upto second degree in x and y
5. Find the maximum or minimum values of
x
3
l(1-x-y), x:;t:O, y:;t:O, x+y:;t:l
6. Apply Newton-Raphson method to obtain a root of the equation
x
3
-3x-5=0
,
starting with Xo =3.
7. A random variable x has the following probability distribution:
x: 0 1 234567
p(x): 0 k 2k 2k 3k 2 7
(i) find k (Ii) Evaluate p(x<6), and p(O<x<5).
8. Solve
y-z z-x x-y
--p+--q=-
yz zx xy
172
!ilWJmgl]
Answer any five questions out of eight questions.
Each question carries ten marks
1 Prove that of all the rectangular parallelepiped of the same volume the cube has the least
surface.
&l &l
-+u=
2
Find the solutions u(x, y) of equation Ox IX if u= 4e-3x when t = 0 by separating
variables.
2
3 Find the volume of the portion of the sphere x
2
+ l + Z2 =a lying inside the
cylinder x
2
+ y2 = ay
4 Find the surface of solid generated by the revolution of asteroid x = acos
3
t, y = asin
3
t,
about y-axis
5. A slider in a machine moves along a fixed straight rod. Its distance x (cm) along the rod is
given below for various values of the time t (seconds). Find the velocity and acceleration of
the slider when t =0.3 sec.
t 0 0.1 0.2 0.3 0.4 0.5 0.6
x : 30.13 31.62 32.87 33.64 33.95 33.81 33.24
6. Compute the inverse of the following by using elementary transformation
2 -6 -2 -3
5 -13 -4 -7
-1 4 1 2
o 1 o 1
7. If [min = Icosmxsin xdx find its reduction formula,
8. Find the probability that at most 5 defective components will be found in a lot of 200 if
experience shows that 3% of such components are defective. Also find the probability of more
than five defective components.
173
Old Question Paper
Advanced Engineering Mathematics
(AC/AA 1.1)
Time: 2:00 PM to 5:00 PM Max. Marks: 100
Assume any missing data If necessary.
I#Nliil.j'Wd
Choose the correct or best alternative In the following.
Each question carries two marks.
0000
J
_y
1. The Value of J-=-dxdY is
Ox y
00
(a) 1 (b) 0 (c) -1 (d)
a
2. The value of J[f(a + x) + f(a x)]dx is equal to
o
a a
(a) Jf(x)dx (c) 2 Jf(x)dx
o o
2a
(b) Jf(x)dx
(d) None of the above
o
12
3. The reduction formula for In =. sin
n
6:tOshall be
[
1 n-l
(a) In =-I
n
_
1
(c) In =-I
n
_
2
n n
2 1
(b) In +1"_1 = - (d)
In = 2n In-2
n
4. z = ea.<+b
y
f{ax - by) is solution of differential equation
az az
az +a az =2z
(a) a ay +b ax = 2z (c)
ay ax
az az az az
(b) a ay +b ax =2abz (d)
b-+a-=2abz
ay ax
174
1 a
2
a
2
.
5. If f1= -log(x2 + y2) then ~ + ~ is equal to
2 ax dy
(a) 2f1 (b) f1 (c) x
2
+l (d) o
6. A system of m linear equations in n unknowns gives the matrix form
AX-:::8
(a) has a solution if m ~ n (c) has no solution if 8=0
(b) has a solution if n ~ m (d) has a solution if m ~ nand B=O
7. If the variance of the Poisson distribution is 2, the value of P ( r ~ 4) is
(a) 0.271 (b) 0.180
(c) 0.090 (d) 0.143
-0.5 0 0]
8. The inverse of the matrix 0 4 0 is
[
o 0 1
0.5 0 0] -2 0 0]
(a) 0 -4 0 (c) o 0.25 0
[ [
o 0 -1 o 0 1
0.5 0 0] 2 0 0]
(b) 0 -4 0 (d) o -0.25 0
[ [
o 0 1 o 0 -1
9. The order and degree of the following partial differential equation
2
au=(au)3 is
axdy az
(a) order 1, degree 3 (c) order 1, degree 2
(b) order 2, degree 1 (d) order 2, degree 3
00
10. The value of Je-xxndx. nbeing a positive integer, is
o
(a) (n+l)! (c) (n-l) !
(b) n! (d) None of the above
175
I44 MIe",:)
Choose True/False in the following.
Each question carries one mark.
1. Inverse of a matrix is unique.
,[1 -2
2. The matrix A =.l - 2 1 ~ 1is an orthogonal matrix.
3 -2 _ 2
-1
3. Newton-Raphson method fails where f'(x). is zero or nearly zero.
l2 l2
r
~ dx
4. r ~ dx=
l+.Jtanx 1+.Jcotx
5. The Simpson's one-third rule in numerical integration needs at least 4 geometrical
points.
@lUII.I,le
Answer any five questions out of eight questions
Each question carries five marks
1. If l, m are positive real numbers then
I Ix1-1ym-1dxdy = nl)nm)
D nm+l +1)
Where D is the domain x ~ 0, y;::: 0 and x+ y S; 1
2. Find the perimeter of the cardioid , = a(l- cos 8).
3. Find the volume of the solid generated by revolving the curve ,2 =a
2
cos
2
8 about the
line
8=tc12
4. Evaluate the integral JJ(x- y f cos
2
(x+ y}dxdy, where R is the rhombus with
R
successive vertices at (tc,O), (2tc,tc), (tc,2tc), and ( O , n - ~
-'C;'
x-y
5. Evaluate cos--dxdy where R is banded by x =0 , y =0 , x + y = 1.
II
R x+y
176
x
6. Find a positive root of the equation xe = 1 lying in (0,1).
2
Z
7. Solve L p+xzq =l
x
8. Evaluate
7C/2
flog(tan x +cot x)dJ;
o
@lMii."'.]
Answer any five questions out of eight questions.
Each question carries ten marks
1. State and prove Euler's theorem for functions of three independent variables.
2 2
2. Form the partial differential equation 2z = x 2 +..;..
a b
3. Using method of separation of variables, find the solution of equation
2
au au if (0) .
- = 2 lUX, = SIn 11X
at ax
4. The temperature T at any point (x,y,z)in space is given by T 400
XYZ
2. Find the
highest temperature at the surface of a unit sphere x
2
+ y2 + Z2 =1
- ~ a 2 - i xy log(x + a)
5. Evaluate (x a)2 dx dy by changing the order of integration
r r
6. Obtain Taylor's expansion of tan -1 Z about (1,1) up to and including the second
x
degree terms, Hence complete f(1.1, 0.9).
7. A solid of revolution is formed by rotating about the x-axis, the area between the
x - axis, the lines x 0 and x = 1 and a curve through the points with the
following Co-ordinates.
x: 0.00 0.25 0.50 0.75 1.00
Y : 1.0000 0.9896 0.9589 0.9089 0.8415
Estimate the volume of the solid formed using Simpson's rule.
8. In a normal distribution, 31 % of the items are under 45 and 8% are over 64. find the
mean and standard deviation of the distribution.
177
l
Old Question Paper
Advanced Engineering Mathematics
(AC/AA 1.1)
22
nd
, June, 20Q!I Time: 2:00 PM to 5:00 PM

Max. Marks: 100
......_.._._._- ----
Assume any missing data if necessary.
44U.hE1
_Choose the correct or best alternative in the following.
Each question carries two marks.
1. The marks obtained were found normally distributed with mean 75 and
variance 100. The percentage of students who scored more than 75 marks, is
(a) 25% (c) 50%
(b) 75% (d) 100%
1 2 3
2.
J J Jdxdydz
-1-2-3
Is
(a) 0 (c) 48
(b) - 48 (d) 6
3. The random variable X has a Poisson distribution of P(X=1) = 0.01487
P(X=2) = 0.04461, then P(X=3) is
(a) 0.08922 (c) -0.08922
(b) 1.235 (d) -1.235
4. The solution of equation z = px +qy + p2 +q2 is
(a) z=a.x+a
2
+b
2
(c) z=ax+by+a
2
+b
2
(b) z=a.x+b (d) Z2 =ax+by
!]
then A (adj A) equals
5. If A =
(c)
(a)
[
10
0 10
0]
[
10 1]
110
0 10]
(d) None of the above
(b)
[
10 0
178
6. A box contains 100 bulbs including 10 defective. The probability that in a sample of 5,
none is defective is
1
(a) 10
5
(c)
(:oJ
(d) 9/10 (b) (i)S
7. If f (x, y) =0, then dy is equal to
dx
(a) Ix (c)
Iy Ix
(b) Iy
(d)
Ix
Ix
Iy
8. The figure bounded by one arc of the wave y= sin x and x-axis is revolved about x
axis. The volume of solid of revolution is
(a) 1[
(c) 21[
2
2
(b) ~ (d)
1[2
2
9. The order of convergence in Newton - Raphson method is
(a) 2 (c) 3
(b) 0 (d) None of the above
10. Lagrange's method is used to solve partial differential equation of
(a) first order and degree 2 (b) first order and degree one
(c) second order and degree 2 (d) second order and degree one.
QfI;t;i.t,'iii=1
Choose TrueJFalse in the following.
Each question carries one mark.
1. If A,B, C are non-singular nxn matrices then (ABCr
1
=A-1B-
1
C-
1
2. The probability of throwing 9 with two dice is 1
9
3. The t- distribution is used to test the significance of coefficient of correlation.
179
tr
4. jcosf(sinx)dx 0
o
5. Out of methods of false position and Newton Raphson method, the role of
convergence is faster for false position.
ijtlfi'i."""
Answer any five questions out of eight questions.
Each question carries five marks
1. Apply Newton-Raphson method to obtain a root of the equation
x
3
-3x-5=O
2. Test for consistency of the following system of equations
2x-3y+5z=1,3x+y-z=2, x+4y 6z=1
and, if consistent, solve them.
3. Find the volume of the solid generated by revolving the curve
2
r2 = a cos
2
8 about the line 8 =!r12
4. Evaluate the following integral by changing the order of integration
1 h_X2 X dydx
J J~ x 2 + l
5. A random variable x has the following probability
x: 0
P(x):
(i) Find k,
1
o
2
k
3
2k
4
2k
(ii) Evaluate P(x<6)
5
3k
6
k
2
72
2k2 7k2+k
6. Solve ~ p+xzq=i
x
180
7. Evaluate
1r/2
J log (tan x + cot x) dx.
o
8. Find the maximum and minimum distances ofthe point (3,4,12) from the sphere
X
2
+y2+Z2=1.
mail.",.]
Answer any five questions out of eight questions.
Each question carries ten marks
1. Solve rz p -.x
2
zq = x
2
Y
3
2. Trace the curve x =acos t, y =a sin
3
t.Also find the surface area of the solid
generated by revolution of the curve about y-axis
3. Find the real root of the equation
x
3
-12.2x
2
+7.45x+42=0
lying between x =11 and x =12using Regula Falsi method.
4.. Find the solution of equation
at 2U 'f (0) .
"I
-=-'2 I U x, =smJ1X
Ii: at
by using separating variables method.
5. If the probability density function of a random variable x is
f(x): kx
a
-
I
(1- xl-! ,0 < X < 1,a >O,p > 0,
= 0, otherwise
find k and mean of x
!t/2
6. if In = Jxnsin x,n > 1
o
Show that In + n(n -1)1._2 = n ( ~ )"-1
7. if XXYYZZ = c,show that at x = y = z
(fz
dXdy =-(xlogexr!
181
L_
8. Find by double integration, the smallest of the areas bounded by the
circle x
2
+1 =9 and the straight line x+y=3
182
Old Question Paper
Advanced Engineering Mathematics
(AC/AA 1.1)
Date: 18
111
, Dec., 2009 Time: 2:00 PM to 5:00 PM Max. Marks: 100
Assume any missing data if necessary.
gigltm.,
Choose the correct or best alternative In the following.
Each question carries two marks.
3
x + l . (b) au
au
1. Let u (x, y ) = + x tan 1 - ,( x, y) (0,0) then x - +
dy equals
x+y x ~
(a) (b) 2u
x3 + 3
( 3 3) -1 Y
(c) Y + 2x tan-
1
L
(d)
2x +y +2xtan ~
x+y x x+y
C
2
2
au =C
2
V
2
U,
= K, This equation is called
at up
(a) Cold equation (b) Warm equation
(c) Heat equation (d) None
3. The rank of a matrix A is the ............... number of linearly independent row
vectors of A.
(a) Minimum (b) Maximum
(c) Two (d) None
4 6 9 6]
4. If A = 6 0 - 2 20, This is called
[
5 -8 1 10
(a) Augmented matrix (b) Segmented matrix
(c) Linear matrix (d) None
5. J J J dxdydz ,This is called
(a) Area (b) Volume
(c) Perimeter (d) A n ~ l e
183
L
6. In rolling a fair die once, what is the probability of obtaining a 5 or a 6?
(a) 1/2 (b) 1/3
(c) 1/4 (d) 1/5
7. X=-n
1
t X J ~ = ~ n l ( Xl +X
2
+ ..... +X
n
) This equation represents
J ~ l
(a) Median (b) Mean
(c) Average (d) None
~
8. The mean f.l, (mu) is defined by f.l= J.if(x)dx for
(a)
(c)
Discrete distribution
Both (a) and (b)
(b)
(d)
Continuous distribution
None of the above
9. D sin =cos, D2 Sin =
(a) -cos
(c) sin
(b)
(d)
-sin
tan
10. If number of rows and columns of matrix are equal, then matrix is called
(a) Rectangular matrix (b) Square matrix
(c) Vector (d) None of the above
MUIt.,,,:)
Choose TrueIFalse In the fOllowing.
Each question carries one mark.
1. Derivative of a function of several variables is the ordinary derivative with respect to
one of the variables when all the rest are hold constant is called Integral.
2. One of the simplest method for finding the root of a nonlinear equation is interval
halving (also known as bisection).
3. A square matrix A = [aka is called Hermitian Matrix if A = A.
4. If matrix A has no inverse, it is called singular matrix.
5. If events A and B are such that (A n B) =P(A)P(B), they are called Permutation.
184
44n.uG
Answer any five questions out of eight questions.
Each question carries five marks.
I
r
1. Find _au
ar
and au
ae
if u = e
rcos
(J cos-(r sinO).
3 -4 -1 2
2.
1
Find the rank of the matrix A = I
5
7
-2
3
5
1
4
9 -3 7 7
3. A bag contains four white and two black balls and a second bag contains three balls
of each colour. A bag is selected at random, and a ball is then drawn at random from
the bag chosen. What is the probability that the ball drawn is white?
4. Evaluate J J ~ xy - y2 dy dx . Where s is a triangle with vertices (0,0), (10,1) and
s
(1, 1).
5. Three machines I, 1\ and III manufacture respectively 0.4, 0.5 and 0.1 of the total
production. The percentage of defective items produced by I, 1\ and III is 2, 4 and 6
percent respectively. An item is chosen at random, what is the probability that it is
effective?
6. Find the arithmetic mean for the following distribution:
3-3 4]
7. If A = 2 - 3 4 ,Find two non singular matrices P and Q such that
[
o -1 1
PAQ =I, Hence find A-
1
8. Find the positive root of the equation xe
x
=1 by Newton Raphson method.
185
maiN"']
Answer any five questions out of eight questions.
Each question carries marks.
2 2 2
. 01[ x+ y ] P . t.h.... t 2 d U.. 2.. d.. U 2 ..d. U -sinu cos 2u
1
.
If
.u= sm Ie ,iove a.x: -2+ .xy --tY 2= 3
VX +-..JY dX dXdy dY 4cos u
2. Evaluate by changing the order of integration J J.x:
2
dx dy, where A is a region in the
A
first quadrant by the hyperbola .xy =16 and the lines y =x, y =0 and x = 8.
3. Solve, with the help of matrices, the simultaneous equations:
x+y+z=3; x+2y+3z=4; x+4y+9z 6
4. The two regression equations of the variables x and yare
x=19.13-0.87 y and y=11.64-0.50x.
Find (i) Mean of Xl s; (ii) Mean of yl s; (iii) The correlation coefficient between x and y
5. There are two groups of subjects one of which consists of 5 science and 3
engineering subjects and the other consists of 3 science and 5 engineering subjects.
An unbiased die is cast. If number 3 or number 5 turns up, a subject is selected at
random from the group, otherwise the subject is selected at random from the second
group. Find the probability that an engineering subject is selected ultimately.
6. Find the value of Median from the following data:
I No. of days for 5 10 15 20 25 30 35 40
which absent (less
than)
No. of students 29 224 465 582 634 644 , 650 653
7. Use method (Elementary matrices method) to compute the inverse of
the matrix
A =[3 -3 41
2 -3 4
o -1 1 J
6dx
8. Evaluate J--2 by using
o 1+x
0) Trapezoidal rule (ii) Simpson's.!..
3
rule
(iii) Simpson's rule by dividing the intelVal in six equal parts.
8
186
ula uuestion Paper
Advanced Engineering Mathematics
(AC/AA 1.1)
Date: 24th, June 2010 Time: 2:00 PM to 5:00 PM Max. Marks: 100
Assume any missing data if necessary.
!iNdit."..
Choose the correct or best alternative In the following.
Each question carries two marks.
1. A matrix is a ............ array of numbers (or functions) enclosed in brackets.
(a) Rectangular (b) Triangular
(c) Circular (d) None of the above
2. In rolling a fair dice once, what is the probability of obtaining even number?
(a) 1/3 (b) 1/4
(c) 1/2 (d) 1/6
3. The mean ,u (mu) is defined by .u = 2>j!(X
j
)for
(a) Discrete distribution (b) Continuous distribution
(c) Both (a) & (b) (d) None of the above
4. The product of any mXn matrix A = lajk Jby any scalar c (number c) is written as
cA then the mx n matrix obtained by multiplying each entry of c by A
(a) cA=lc+ajkJ (b) cA=lc-ajkJ
(c) cA=lcxajkJ (d) None of the above
5. A function f(x, y) is homogeneous of degree n in a region R if and only if
(a) f(2x, y) = 2f(x, y) (b) f(Ax, liy) =lin f(x, y)
(c) f(x,2y) = 2f(x, y) . (d) None of the above
3 3
6. The value of limit Lt x2 - y, is
(x,y) ~ (1,.1) x - Y
(a) 0 (b) 1/2
(c) 3/2 (d) 1
187
L
...~ ":,..;
7. A transformation that transforms a function into another function is called
(a) Operator (b) Separator
(c) Denoter (d) None of the above
2 In 2 Ir 2
8.
S =-L (x. -X) =-l(xI-X) +",+(x
n
X)2 ]
n "-I j=r J n ~ 1 . ......... .
Above equation represents
(a) Standard Deviation (b) Variance
(c) Mean (d) None of the above
9. For mutually exclusive events A and B
(a) P(AUB)=P(A)xP(B) (b) P(AUB)=P(A)-P(B)
(c) P(A UB) =P(A) + P(B) (d) None of the above
1O. 1'1' events A and B are such that P(A fl B) = P(A)P(B) , they are called
(a) Independent events (b) Dependent events
(c) Discrete events (d) None of the above
man."':1
Choose TrueJFalse In the following.
Each question carries one mark.
1. A POE is an equation involving one or more partial derivatives of a function.
2. An ODE together with an initial condition is called an initial value problem.
3. A matrix is a rectangular array of numbers (or functions) enclosed in brackets.
4. If the number of rows and columns of matrix are equal, then it is called Square Matrix.
5. Numerical differentiation formulae can be developed by fitting approximating functions
(e.g., polynomials) to a set of discrete data and differentiating the approximating
function.
188
ClDmD1Ift

Answer any five questions out of eight questions

Each question carries five marks
3
-1 X + Y
3
au au .
1. If u =Jan ,pr.ove that x.-.- ..... + y.-.. -.::: sm2u
x-y ~ ~
2
2. Find the area between the parabolas l =4ax and x =4ay
3. An urn contains 10 black and 10 white balls. Find the probability of drawing two
balls of the same colour.
4. If z be a homogeneous function of degree n, show that
a
2
a
2
a
2
2 z z 2 z
x .-+2xy.--+y .-=n(n-1)z
ax
2
axay ay2
5. Evaluate IIxydxdy over the region in the positive quadrant x + y $; 1.
-
2 3 -1 -1
6.
1
Find the rank of matrix I
3
-1
1
-2
3
-4
-2
6 3 0 -7
7. Three machines I, II and III manufacture respectively 0.4, 0.5 and 0.1 of the total
production. The percentage of defective items produced by I, II and III is 2, 4 and 6
percent respectively. An item is chosen at random, what is the probability that it is
defective?
8. Using the method of False position, find the real root of the equation eX =x up to
three decimal places.
189
@lUll.'''.]
Answer any five questions out of eight questions.
Each question carries ten marks.
2 2 2
1. If U -= sin-I [ x + y ] prove that 2 au 2 au
-fY ' x.ax
2
+ xy axCJy + Y
tr
21r 4" a
2. Evaluate J J Jr2Sin8drd8d9
000
2 4 3 2
6 5 2
3. Find the inverse of the matrix A =I3
2 5 2 -3
4 5 14 14
2 au
sin ucos 2u
u
4. Find the regression line of y on x for the following data:
x 1 3 4 6 8
1 2 4 4 5
Y
I I I I I
Estimate the value of y, when x =10.
9
5. If u = log(x
3
=t- y3 + Z3 - 3xyz), show that
(0
-
00)'
u = + - +
ax ()y az (x+y+
6. Solve, with the help of matrices, the simultaneous equations
x+y+z=3, x+2y+3z=4, x+4y+9z=6
1 2 -1] [3 -1 1]
7. If A = 2 0 3 ; B = 0 0 2
[
o 1 2 4 -3 2
Verify the result (A + BY = A
2
+ AB + BA + B2.
~
8. Evaluate !.Jcose. de by using Simpson's !rd rule.
o 3
190
x- Lh
Area under standard normal curve from 0 to
o z
~ g - ~ .
Table
x
~
0'
0.00 0.01 0.02 0.03 0.04 0.05 0.06 0.07 0.08 0.09
0.0 0.0000 0.0040 0.0080 0.0120 0.0160 0.0199 0.0239 0.0279 0.0319 0.0359
0.1 0.0398 0.0438 0.0478 0.0517 0.0557 0.0596 0.0636 0.0675 0.0714 0.0753
0.2 0.0793 0.0832 0.0871 0.0910 0.0948 0.0987 0.1026 0.1064 0.1103 0.1141
~ : ..r
0.3 0.1179 0.1217 0.1255 0.1293 0.1331 0.1368 0.1406 0.1443 0.1480 0.1517
0.4 0.1554 0.1591 0.1628 0.1664 0.1700 0.1736 0.1772 0.1808 0.1844 0.1879
0.5 0.1915 0.1950 0.1985 0.2019 0.2054 0.2088 0.2123 0.2157 0.2190 0.2224
0.6 0.2257 0.2291 0.2324 0.2357 0.2389 0.2422 0.2454 0.2486 0.2517 0.2549
0.7 0.2580 0.2611 0.2642 0.2673 0.2704 0.2734 0.2764 0.2794 0.2823 0.2852
0.8 0.2881 0.2910 0.2939 0.2967 0.2995 0.3023 0.3051 0.3078 0.3106 0.3133
0.9 0.3159 0.3186 0.3212 0.3238 0.3264 0.3289 0.3315 0.3340 0.3365 0.3389
1.0 0.3413 0.3438 0.3461 0.3485 0.3508 0.3531 0.3554 0.3577 0.3599 0.3621
1.1 0.3643 0.3665 0.3686 0.3708 0.3729 0.3749 0.3770 0.3790 0.3810 0.3830
1.2 0.3849 0.3869 0.3888 0.3907 0.3925 0.3944 0.3962 0.3980 0.3997 0.4015
1.3 0.4032 0.4049 0.4066 0.4082 0.4099 0.4115 0.4131 0.4147 0.4162 0.4177
1.4 0.4192 0.4207 0.4222 0.4236 0.4251 0.4265 0.4279 0.4292 0.4306 0.4319
1.5 0.4332 0.4345 0.4357 0.4370 0.4382 0.4394 0.4406 0.4418 0.4429 0.4441
1.6 0.4452 0.4463 0.4474 0.4484 0.4495 0.4505 0.4515 0.4525 0.4535 0.4545
191
1.7 0.4554 0.4564 0.4573 0.4582 0.4591 0.4599 0.4608 0.4616 0.4625 0.4633
1.8 0.4641 0.4649 0.4656 0.4664 0.4671 0.4678 0.4686 0.4693 0.4699 0.4706
1.9 0.4713 0.4719 0.4726 0.4732 0.4738 0.4744 0.4750 0.4756 0.4761 0.4767
2J) 0 ~ ~ ~ l t 4 n 8 0:4783---0:4788--0:4793 --0.479&---0:4803 - -0.4808----0:-4812- -0:41317 --_.
2.1 0.4821 0.4826 0.4830 0.4834 0.4838 0.4842 0.4846 0.4850 0.4854 0.4857
2.2 0.4861 0.4864 0.4868 0.4871 0.4875 0.4878 0.4881 0.4884 0.4887 0.4890
2.3 0.4893 0.4896 0.4898 0.4901 0.4904 0.4906 0.4909 0.4911 0.4913 0.4916
2.4 0.4918 0.4920 0.4922 0.4925 0.4927 0.4929 0.4931 0.4932 0.4934 0.4936
2.5 0.4938 0.4940 0.4941 0.4943 . 0.4945 0.4946 0.4948 0.4949 0.4951 0.4952
2.6 0.4953 0.4955 0.4956 0.4957 0.4959 0.4960 0.4961 0.4962 0.4963 0.4964
2.7 0.4965 0.4966 0.4967 0.4968 0.4969 0.4970 0.4971 0.4972 0.4973 0.4974
2.8 0.4974 0.4975 0.4976 0.49n 0.4977 0.4978 0.4979 0.4979 0.4980 0.4981
2.9 0.4981 0.4982 0.4982 0.4983 0.4984 0.4984 0.4985 0.4985 0.4986 0.4986
3.0 0.4987 0.4987 0.4987 0.4988 0.4988 0.4989 0.4989 0.4989 0.4990 0.4990
192
-- ----------- ------------------- - - - - - ~ - - - - ---

You might also like